plusthemath의 등록된 링크

 plusthemath로 등록된 티스토리 포스트 수는 432건입니다.

중1 박준혁 학생 구술 테스트 영상 [내부링크]

안녕하세요. 울산 남구 옥동에 위치한 더플러스수학학원장입니다. 울산 더플러스수학학원에서 20여년을 수능 수리영역을, 최근 10여년 수리논술을 강의해 왔습니다. 포항에 있는 자사고 포항제철고 수리논술 출강했습니다. 또 울산 과학고 1기부터 지금까지 대입 심층면접, 과고내신, AP CALCULUS 수업 등 일등급 학생들의 수학수업에 전문가라고 자부할 수 있습니다. 수능수학, 수리논술과 심층면접, CALCULUS에 대한 질문에 답을 해 줄 수 있습니다. 문제는 수학의 깊이! 해답은 자기주도학습! [더플러스수학학원] 수능, 교육청모의고사, 삼사, 경찰대 등의 기출문제 풀이 동영상, 서울대 등 명문대 심층면접문제, 수리논술문제 풀이 동영상 제공 자기주도적 학습 더플러스수학 : plus the dream beyon..

더플러스수학 원장 [내부링크]

안녕하세요. 울산 남구 옥동에 위치한 더플러스수학학원장입니다. 울산 더플러스수학학원에서 20여년을 수능 수리영역을, 최근 10여년 수리논술을 강의해 왔습니다. 포항에 있는 자사고 포항제철고 수리논술 출강했습니다. 또 울산 과학고 1기부터 지금까지 대입 심층면접, 과고내신, AP CALCULUS 수업 등 일등급 학생들의 수학수업에 전문가라고 자부할 수 있습니다. 수능수학, 수리논술과 심층면접, CALCULUS에 대한 질문에 답을 해 줄 수 있습니다. 문제는 수학의 깊이! 해답은 자기주도학습! [더플러스수학학원] 수능, 교육청모의고사, 삼사, 경찰대 등의 기출문제 풀이 동영상, 서울대 등 명문대 심층면접문제, 수리논술문제 풀이 동영상 제공 자기주도적 학습 더플러스수학 : plus the dream beyon..

옥동수학학원 [더플러스수학학원] (예비)울산과고1학년 설명회 및 수업 안내, (예비)고1학년 최상위반 수업안내 [내부링크]

안녕하세요, 더(THE) 플러스수학학원입니다. (예비)과고1학년 설명회와 수업시간안내, (예비)울산과고1학년 및 최상위반 수업안내 1. 겨울학기를 앞두고 저희 더플러스수학학원에서 예비울산과고1학년 학부모님 및 학생들을 대상으로 설명회를 진행하려 합니다. 장소가 협소하여 전화로 사전예약 부탁드립니다. 대상 : (예비)과고1학년 학부모님 및 학생 (현 중3, 학생은 반드시 참여바랍니다.) 날짜 : 12월 5일 화요일 시간 : 오후 7시 설명회 내용 : - 합격 이후 학습 로드맵 및 학습전략 - 과학고 내신 및 기출 분석과 학습방법 전화 예약 : 052-260-9981 또는 052-258-4409 문자 예약 : 052-258-4409 로 학생이름과 함께 문자보내주시면 됩니다. 2. (예비..

[더플러스수학] 과학고 선배가 과고 후배인 예비과고에게 들려주는 이야기 [내부링크]

학생들이 수학문제를 구술로 풀어야 하는이유는?대상2023 수업일정 및 내용과정초등반(5학년) 상담예약중1과정 개강(1:1 또는 그룹)중등반(1,2,3학년) 상담예약고등과정과고준비반(예비중3학년) 상담예약12월말 개강 예정일반고(현1,2학년) 상담예약내신대비과학고(예비1학년) 상담예약12월 5일(화) 개강예정과학고(1,2학년) 상담예약내신대비 접수 및 상담위의 상담예약으로 접수 후 학원등록시 학원비할인 이벤트 대상 (연락처 : 052-260-9981, 052-258-4409 로 상담문의)-수업+구술(주1회) -http://www.더플러스수학.com 게시판에서 학생들의 구술영상을 확인하실 수 있습니다. 회차가 거듭될수록 향상되는 자녀들의 실력을 확인하세요 -구술영상 예시 https://youtu.be/9_a..

학생들이 수학문제를 구술로 풀어야 하는이유는? [더플러스수학 학원생 모집] [내부링크]

학생들이 수학문제를 구술로 풀어야 하는이유는?대상2023 수업일정 및 내용과정초등반(5학년) 상담예약중1과정 개강(1:1 또는 그룹)중등반(1,2,3학년) 상담예약고등과정과고준비반(예비중3학년) 상담예약12월말 개강 예정일반고(현1,2학년) 상담예약내신대비과학고(예비1학년) 상담예약12월 5일(화) 개강예정과학고(1,2학년) 상담예약내신대비 접수 및 상담위의 상담예약으로 접수 후 학원등록시 학원비할인 이벤트 대상 (연락처 : 052-260-9981, 052-258-4409 로 상담문의)-수업+구술(주1회) -http://www.더플러스수학.com 게시판에서 학생들의 구술영상을 확인하실 수 있습니다. 회차가 거듭될수록 향상되는 자녀들의 실력을 확인하세요 -구술영상 예시 https://youtu.be/9_a..

행렬과 연립방정식-가우스소거법 [내부링크]

1.1. 선형연립방정식 선형연립방정식과 그의 해를 구하는 문제는 선형대수학의 가장 중요한 문제 중 하나입니다. 수 천, 수만 개의 미지수를 갖는 연립방정식은 자연과학, 공학, 경제-사회-인문학은 물론 교통문제, 일기예보, 의사결정 둥 수많은 분야에서 만나게 됩니다. 더구나 속도, 가속도와 같이 도함수를 포 함하는 미분방정식도 선형연립방정식 문제로 바꾸어 해결합니다. 선형 대수학에서 선형연립방정식의 해는 첨가행렬을 이용한 Gauss(가우스)소거법이나 행렬식을 이용한 방법으로 구합니다. 이장에서는 선형연립방정식의 해법과 구한 해의 기하학적 의미를 고찰 하고 다양한 선형연립방정식의 응용에 대하여 알아봅시다. 정의 [선형방정식] 미지수 $x_1,~ x_2,~ \ldots,~ x_n$ 에 관한 선형방정식(line..

2023년 과고2학년 2학기 대비 공부 [내부링크]

이번 학기에서는 우연히 수학(하)와 확통에서 공통으로 경우의 수를 다룬다. #울산과고 1학년, 2학년 수업 모두에는 실력정석 "경우의 수" 단원의 연습문제에 #교란순열을 포함되어 있다. 교란순열을 #포함과_배제의_원리(포제의 원리)를 갖고 일반항을 구할 수 있고, 또, #점화식 을 이용하여 교란순열의 개수를 구할 수 있다. 따라서 이 글에서는 교란순열에 대해 알아보고자 한다. 교란순열의 일반항, 점화식, 항등식 등등.... 교란순열 함수 \(\displaystyle f:\left\{1,~2,~3,~\cdots,~n \right\} \rightarrow \left\{1,~2,~3,~\cdots,~n \right\} \)에 대하여 \(\displaystyle f ( 1) \neq 1 \), \(\displa..

과고1학년 2학기 대비 공부 [내부링크]

이번 학기에서는 우연히 수학(하)와 확통에서 공통으로 경우의 수를 다룬다. #울산과고 1학년, 2학년 수업 모두에는 실력정석 "경우의 수" 단원의 연습문제에 #교란순열을 포함되어 있다. 교란순열을 #포함과_배제의_원리(포제의 원리)를 갖고 일반항을 구할 수 있고, 또, #점화식 을 이용하여 교란순열의 개수를 구할 수 있다. 따라서 이 글에서는 교란순열에 대해 알아보고자 한다. 교란순열의 일반항, 점화식, 항등식 등등.... 교란순열 함수 \(\displaystyle f:\left\{1,~2,~3,~\cdots,~n \right\} \rightarrow \left\{1,~2,~3,~\cdots,~n \right\} \)에 대하여 \(\displaystyle f ( 1) \neq 1 \), \(\displa..

[더플러스수학] 교란순열에 대하여 [내부링크]

이번 학기에서는 우연히 수학(하)와 확통에서 공통으로 경우의 수를 다룬다. #울산과고 1학년, 2학년 수업 모두에는 실력정석 "경우의 수" 단원의 연습문제에 #교란순열을 포함되어 있다. 교란순열을 #포함과_배제의_원리(포제의 원리)를 갖고 일반항을 구할 수 있고, 또, #점화식 을 이용하여 교란순열의 개수를 구할 수 있다. 따라서 이 글에서는 교란순열에 대해 알아보고자 한다. 교란순열의 일반항, 점화식, 항등식 등등.... 교란순열 함수 \(\displaystyle f:\left\{1,~2,~3,~\cdots,~n \right\} \rightarrow \left\{1,~2,~3,~\cdots,~n \right\} \)에 대하여 \(\displaystyle f ( 1) \neq 1 \), \(\displa..

[더플러스수학]과학고 학교 문제 중 조금 어려운 문제 [내부링크]

울산과학고 학교 프린트 중에 다음의 문제가 있었다. 학생들이 어려워 해서 한 번 풀어보았습니다. \(\displaystyle n\) 개의 실수 \(\displaystyle a_1 ,~a_2 ,~\cdots,~a_n \)으로 된 집합 \(\displaystyle A\)가 있다. 이 집합 \(\displaystyle A\) 의 원소 \(\displaystyle a_i \)가 그 외의 \(\displaystyle n-1\)개의 원소의 평균보다 크지 않게 하는 이 실수의 조로 된 집합을 모두 구하여라. 정답 없다. (풀이) 집합 \(\displaystyle A\)가 \(\displaystyle n\)개의 원소 \(\displaystyle a_1 ,~a_2 ,~\cdots,~a_n \)로 이루어져 있으므로 \(\..

[옥동수학학원] 울산과학고 제18기 설명회 [내부링크]

2023학년도 #울산과학고등학교_제18기 합격자 발표가 12월6일(화)로 다가왔습니다. 중학교 3년을 보내면서 울산과학고등학교 진학을 꿈꾸었던 친구들 모두에게 좋은 소식이 전해지길 바랍니다. #더_THE_플러스수학전문학원에서는 울산과학고등학교 입학예정 학생들과 학부모님들을 위해 울산과학고등학교 수학 커리큘럼 설명 및 본 학원의 겨울방학 특강 설명회를 다음과 같이 준비하고 있습니다. 일시 : 2022년 12월 6일 (화) 19:00 장소 : 더(THE)플러스수학전문학원 제3강의실 내용 : 울산과학고등학교 수학 커리큐럼 및 겨울방학 학습 계획 설명 참석예약 전화번호: 052-260-9981 *문의 전화 주시면 성실히 답변드리겠습니다!!! 다시 한번 울산과학고등학교 합격자 발표를 기다리는 모든 친구들과 ..

[더플러스수학] 과고 추가문제 [내부링크]

심화수학 수업에서 학교에서 나온 문제입니다. 첫번째는 수리논술 기출일 것 같은데 어디인지는 잘 모르겠고 알고 있는 사람은 댓글을 달아 주세요. 두 번재는 포스텍 면접문제, 세번째는 부산대 수리논술 문제입니다. 풀이를 서술하기는 시간이 많이 걸려 우선 풀이 동영상을 링크 걸겠습니다. 참조하세요. (가) \(\displaystyle f\)가 실수 전체의 집합에서 정의된 함수일 때, 실수 \(\displaystyle a\)에 대하여 극한값 \(\displaystyle \lim_{ h \rightarrow 0} \frac{ f(a+h)-f(a)}{h}\)가 존재하면 함수 \(\displaystyle f\)가 \(\displaystyle x=a\)에서 미분가능하다고 한다. 이 때, 이 값을 함수 \(\displa..

[더플러스수학] 과학고 학교 프린트 [내부링크]

심화수학 수업에서 학교에서 나온 문제입니다. 첫번째는 수리논술 기출일 것 같은데 어디인지는 잘 모르겠고 알고 있는 사람은 댓글을 달아 주세요. 두 번재는 포스텍 면접문제, 세번째는 부산대 수리논술 문제입니다. 풀이를 서술하기는 시간이 많이 걸려 우선 풀이 동영상을 링크 걸겠습니다. 참조하세요. (가) \(\displaystyle f\)가 실수 전체의 집합에서 정의된 함수일 때, 실수 \(\displaystyle a\)에 대하여 극한값 \(\displaystyle \lim_{ h \rightarrow 0} \frac{ f(a+h)-f(a)}{h}\)가 존재하면 함수 \(\displaystyle f\)가 \(\displaystyle x=a\)에서 미분가능하다고 한다. 이 때, 이 값을 함수 \(\displa..

[더플러스수학] 과고 심화수학 프린트 문제 풀이 [내부링크]

심화수학에서 삼각함수의 역함수, 즉 역삼각함수를 정의하기 위해 삼각함수의 정의역과 치역을 제한하여 일대일 대응을 만든다. 각각의 역삼각함수 \(\displaystyle \sin^{-1}x,~\cos^{-1}x ,~\tan^{-1}x ,~ \cot^{-1}x,~\sec^{-1}x ,~\csc^{-1}x\)의 도함수를 구한다. 이를 이용하여 문제를 푼다. 예제5. (1) \(\displaystyle y=\cos ^{-1}(1-2x)\) (2) \(\displaystyle y=\cot ^{-1} \left(\frac{1-x}{1+x}\right)\) https://youtu.be/5OLDxRr4w40 (구독과 좋아요를) 예제6. \(\displaystyle \cos \left(\sin ^{-1}x \right..

[과고1] 수2 방과 후 프린트 [더플러스수학] [내부링크]

다음 제시문을 읽고 물음에 답하시오. (가) 롤의 정리 함수 \(\displaystyle f(x)\)가 닫힌 구간 \(\displaystyle [a,~b]\)에서 연속이고, 열린 구간 \(\displaystyle (a, ~b)\)에서 미분가능할 때, \(\displaystyle f(a)=f(b)\)이면, \(\displaystyle f'(c)=0\)인 \(\displaystyle c\)가 \(\displaystyle a\)와 \(\displaystyle b\) 사이에 적어도 하나 존재한다. (나) 함수 \(\displaystyle f( x),~g(x)\)와 실수 \(\displaystyle k\)에 대하여 \(\displaystyle g(x)=e^{kx} f(x)\) 이면 방정식 \(\displaysty..

[더플러스수학] 과학고 준비 이렇게! 꽃들에게 희망을! 수학 공부하는 학생들에게!.... [내부링크]

[더플러스수학] 과학고 준비 이렇게! 꽃들에게 희망을! 수학 공부하는 학생들에게!.... 꽃들에게 희망을!!!!! 호랑애벌레.... 올라간다. 위에는 아무것도 없다. 허무하다. 주위의 애벌레에게 말을 해도 알아듣지 못한다. 노랑애벌레... 나비가 되어 애벌레 기둥에 먼저 올라가서 없다는 것을 미리 안다. 우리는 어떤 애벌레가 되어야 할까? 우리 학생들과 어머님들은 이학원 저학원 다니면서 학생들이 공부 잘 하게 노력하신다. 그러나 학생들과 어머님들은 이학원이 더 잘가르치는지 저학원이 잘 가르치는지 평가하는 능력만 올라간다. 그러나 학생들의 수학실력은......? 수학 잘하는 방법은 없다. 학원에 없다. 학생에게 있다는 것을 빨리 깨달아야 한다. 우리 학생들은 수업을 너무 많이 듣는다. 수업을 많이 듣는다고..

[수학의 기초] 증명"리만적분가능하면 유계" [더플러스수학] [내부링크]

AP-Calculus과정에서 구간 \(\displaystyle [a,~b]\)에서 리만적분 가능한 함수 \(\displaystyle f\)는 그 구간에서 유계임을 증명하는 것이 나와 학생들이 많이 당황해 하고 있어서 이 글을 작성한다. 먼저 리만적분 가능하다는 것은 https://plusthemath.tistory.com/500 [수학의 기초] 정적분의 정의(2)-상적분, 하적분, 정적분의 성질 앞의 글에서 리만합, 상합, 하합사이의 관계에 대해 말했다면 이젠 이것을 바탕으로 정적분을 정의하자. 앞의 글을 먼저 읽고 이 글을 읽는 것이 이해하기가 편할 것이다. 2021.02.01 - [수학과 공부 plusthemath.tistory.com 에서 알 수 있듯이 다음과 같이 정의한다. 실수 \(\displa..

[수학의 기초] 평균변화율과 이차함수의 성질과 그 역 증명 [더플러스수학] [내부링크]

정적분문제를 점화식을 이용하여 푸는 문제를 찾아 보았다. 문제1. \(\displaystyle m,~n\)이 음이 아닌 정수일 때, 다음을 증명하여라. (1) \(\displaystyle \int_{\alpha}^{\beta}(x-\alpha)^m (\beta-x)^n dx =\frac{m! n! (\beta-\alpha)^m+n+1}{(m+n+1)!}\) (2) \(\displaystyle \int_0^{\frac{\pi}{2}} \sin^{2m+1}x \cos^{2n+1} x dx = \frac{m!n!}{2(m+n+1)!}\) (힌트 : \(\displaystyle \sin^2 x =t\)로 치환)

[정적분과 점화식] beta function [AP-Calculus] [내부링크]

정적분문제를 점화식을 이용하여 푸는 문제를 찾아 보았다. 문제1. \(\displaystyle m,~n\)이 음이 아닌 정수일 때, 다음을 증명하여라. (1) \(\displaystyle \int_{\alpha}^{\beta}(x-\alpha)^m (\beta-x)^n dx =\frac{m! n! (\beta-\alpha)^m+n+1}{(m+n+1)!}\) (2) \(\displaystyle \int_0^{\frac{\pi}{2}} \sin^{2m+1}x \cos^{2n+1} x dx = \frac{m!n!}{2(m+n+1)!}\) (힌트 : \(\displaystyle \sin^2 x =t\)로 치환)

[옥동수학학원][수학의 기초] 다르부 정리 -도함수의 사잇값 정리[더플러스수학학원] [내부링크]

울산과고 3학년 학생들과 AP_Calculus 수업을 할 때, 연속함수에서는 사잇값 정리가 있는데 미분가능한 함수의 도함수에서는 사잇값 정리와 비슷한 성질 - 사잇값 성질 이 있음을 설명해야 하는 상황이 와서 울산 옥동에 있는 수학학원인 더플러스수학 학원에서 다르부 정리라고 말하는 정리를 조사해 보았다. 다르부 정리 닫힌구간 \(\displaystyle I\)에 대하여 함수 \(\displaystyle f:I\rightarrow \mathbb R\) 가 치역이 실수인 미분가능한 함수일 때, 도함수 \(\displaystyle f'\)는 사잇값 성질을 갖는다. 여기서 사잇값 성질이란 다음과 같다. \(\displaystyle a

과고2학년 미적분 학교프린트풀이 [내부링크]

1. 임의의 \(\displaystyle x \)에 대하여 \(\displaystyle x \)의 다항식 \(\displaystyle f ( x) \)가 \(\displaystyle f ( x ^{2} )=x ^{3} f ( x+1)-2x ^{4} +2x ^{2} \) 을 만족할 때, \(\displaystyle f ( 0)=f ( 1)=f ( 2)=0 \)임을 보이고 \(\displaystyle f ( x) \)를 구하여라. (단, 모든 실수 \(\displaystyle x \)에 대하여 \(\displaystyle f ( x)=0 \)인 것은 아니다. 즉 \(\displaystyle f ( x _{0} ) \neq 0 \)인 실수 \(\displaystyle x _{0} \)가 존재한다.) https..

[더플러스수학] 과고 1학년1학기 중간대비 킬러문항(카톡제공)-1,2 [내부링크]

1. 임의의 \(\displaystyle x \)에 대하여 \(\displaystyle x \)의 다항식 \(\displaystyle f ( x) \)가 \(\displaystyle f ( x ^{2} )=x ^{3} f ( x+1)-2x ^{4} +2x ^{2} \) 을 만족할 때, \(\displaystyle f ( 0)=f ( 1)=f ( 2)=0 \)임을 보이고 \(\displaystyle f ( x) \)를 구하여라. (단, 모든 실수 \(\displaystyle x \)에 대하여 \(\displaystyle f ( x)=0 \)인 것은 아니다. 즉 \(\displaystyle f ( x _{0} ) \neq 0 \)인 실수 \(\displaystyle x _{0} \)가 존재한다.) https..

[2022년 3월교육청 모의고사 기하 30번] 포물선의 정의 응용 [더플러스수학] [내부링크]

그림과 같이 꼭짓점이 \(\displaystyle \mathrm{A_1}\) 이고 초점이 \(\displaystyle \mathrm{F_1}\) 인 포물선 \(\displaystyle P_1\)과 \(\displaystyle \mathrm{A_2}\) 이고 초점이 \(\displaystyle \mathrm{F_2}\) 인 포물선 \(\displaystyle P_2\)가 있다. 두 포물선의 준선은 모두 직선 \(\displaystyle \mathrm{F_1 F_2} \)와 평행하고, 두 선분 \(\displaystyle \overline{\mathrm{A_1A_2}},~ \overline{\mathrm{F_1 F_2}} \)의 중점은 서로 일치한다. 두 포물선 \(\displaystyle P_1,~P_2\..

[2022년 3월 교육청모의고사 수학1-13번] 등차수열의 합 [내부링크]

2022년 3월에 친 교육청 모의고사 수학1 수열 단원의 등차수열의 합과 관련된 문제입니다. 수열은 자연수의 집합에서 실수의 집합으로의 함수라는 것을 착안하여 등차수열의 합은 원점을 지나는 2차함수로 보고 그래프를 이용하여 문제를 풀었습니다. 수열은 함수다. 첫째항이 양수인 등차수열 \(\displaystyle \left\{ a_n \right\}\)의 첫째항부터 제\(\displaystyle n\)항까지의 합을 \(\displaystyle S_n \)이라 하자. \(\displaystyle \left| S_3 \right| = \left|S_6 \right| =\left|S_11 \right| -3\) 을 만족시키는 모든 수열 \(\displaystyle \left\{ a_n \right\}\)의 첫째..

[수학의 기초] 정적분의 정의(2)-상적분, 하적분, 정적분의 성질 [내부링크]

앞의 글에서 리만합, 상합, 하합사이의 관계에 대해 말했다면 이젠 이것을 바탕으로 정적분을 정의하자. 앞의 글을 먼저 읽고 이 글을 읽는 것이 이해하기가 편할 것이다. 2021.02.01 - [수학과 공부이야기] - [수학의 기초] 정적분의 정의(1)-리만합, 상합, 하합의 관계 먼저 실수의 성질인 완비성공리를 전제로 논리를 진행한다. 상계, 하계, 상한, 하한에 대해서도 다음 글을 참고하시길 바란다. 2022.03.22 - [수학과 공부이야기] - [수학의 기초]상계-상한, 하계-하한[더플러스수학] 완비성공리 공집합이 아닌 실수의 집합이 아래로 유계이면 상한이 존재한다. 마찬가지로 위로 유계이면 하한이 존재한다. 앞의 글에서 증명한 정리인 "임의의 하합은 모든 상합보다 크지 않다."라는 사실과 실수의 ..

[2022년 3월교육청 모의고사 22번 수학2] 정적분으로 정의된 함수의 응용 [더플러스수학] [내부링크]

2022년 3월 고3 모의고사 22번 수2 킬러문제입니다. 정적분으로 정의된 함수의 미분가능성, 연속성과 3차함수 그래프와의 관계에 대한 문제입니다. 실수 전체의 집합에서 연속인 함수 \(\displaystyle f(x)\)와 최고차항의 계수가 \(\displaystyle 1\)이고 상수항이 \(\displaystyle 0\)인 삼차함수 \(\displaystyle g(x)\)가 있다. 양의 상수 \(\displaystyle a\)에 대하여 두 함수 \(\displaystyle f(x),~g(x)\)가 다음 조건을 만족시킨다. (가) 모든 실수 \(\displaystyle x\)에 대하여 \(\displaystyle x \left| g(x) \right| = \int_{2a}^x (a-t)f(t) dt\..

[2020년3월 가형 미적분 30번] [더플러스수학] [내부링크]

#2020년_3월_가형_모의고사_30번 #미적분킬러문항 문제에 대한 여러가지 풀이 입니다. 최고차항의 계수가 \(\displaystyle 4 \)인 삼차함수 \(\displaystyle f ( x) \)와 실수 \(\displaystyle t \)에 대하여 함수 \(\displaystyle g ( x) \)를 \(\displaystyle g ( x)= \int _ {t} ^ {x} {f ( s)} ds \) 라 하자. 상수 \(\displaystyle a \)에 대하여 두 함수 \(\displaystyle f ( x) \)와 \(\displaystyle g ( x) \)가 다음 조건을 만족시킨다. (가) \(\displaystyle f ' ( a)=0 \) (나) 함수 \(\displaystyle \le..

[옥동수학학원][수학의 기초]울산과고 상계-상한, 하계-하한[더플러스수학] [내부링크]

울산과학고 학생들이 공부하는 AP-Calculus(미적분학)를 학원에서 가르치다 보면 해석학에서 나오는 개념들인 상계, 하계 등등에 대한 개념을 정리할 필요성을 느낀다. 먼저 상한(최소상계)와 최대값의 개념이 혼란스러울 것이다. 예를 들어 유한집합 \(\displaystyle S_1=\left\{ 1,~2,~3,~4 \right\}\)를 생각해 보면, 이 집합 \(\displaystyle S\)의 최댓값은 \(\displaystyle 4\)로 존재한다. 그런데 다음과 같은 무한집합 \(\displaystyle S_2 = [0,~2)\)에서는 최댓값은 존재하지 않는다. 그런데 여기서 \(\displaystyle 2\)는 집합 \(\displaystyle S_2 \)의 원소는 아니지만 또, \(\displa..

[2021년3월 미적분 30번] [킬러문항30번풀이][더플러스수학] [내부링크]

자연수 \(\displaystyle n \)에 대하여 삼차함수 \(\displaystyle f ( x)=x \left ( x-n \right) \left ( x-3n ^{2} \right) \)이 극대가 되는 \(\displaystyle x \)를 \(\displaystyle a _{ n} \)이라 하자. \(\displaystyle x \)에 대한 방정식 \(\displaystyle f ( x)=f \left ( a _{ n} \right) \)의 근 중에서 \(\displaystyle a _{ n} \)이 아닌 근을 \(\displaystyle b _{ n} \)이라 할 때, \(\displaystyle \lim\limits _{n \rightarrow \infty } {\frac {a _{ n} b..

[AP-Calculus] Proof of \(\lim\limits_{x \rightarrow a} f(x)=L \Longleftrightarrow \lim\limits_{h \rightarrow 0}f(a+h)=L\) [더플러스수학] [내부링크]

Prove that \(\displaystyle \lim\limits_{x \rightarrow a} f(x) =L \) if and only if \(\displaystyle \lim\limits_{h \rightarrow }f(a+h)=L\). (Use the precise definition of limits with \(\displaystyle \epsilon-\delta\)) -극한에 대한 엄밀한 정의인 \(\displaystyle \epsilon-\delta\)논법을 이용하여 다음이 서로 동치-필요충분조건임을 보이시오. \(\displaystyle \lim\limits_{x \rightarrow a} f(x) =L \) \(\displaystyle \Longleftrightarrow\) \(\..

[AP-Calculus] 고려대 미적분학 기출 (Spring, 2010) [더플러스수학] [내부링크]

2010학년도 봄, 고려대 미적분학 기출 Calculus Ⅰ Exam 1(Spring, 2010) 1. (15 pts) Let \(\displaystyle f ( x)= {\begin{cases}x ^{2} +1, ~& \rm {if}~ x \leq 0\\x ^{3} +1, ~&\rm{if}~ x>0\end{cases}} \) Use the '\(\displaystyle \epsilon - \delta \) argument' to show that \(\displaystyle f \) is continuous on the real line. 2. (15 pts) Let \(\displaystyle f \) be a polynomial and let {\(\displaystyle x _{0} ,x _{1}..

[더플러스수학] 증가함수(또는 감소함수)의 역함수도 증가함수(또는 감소함수)이다. [내부링크]

정의 증가함수 함수 \(\displaystyle f\,:\,(a,~b) \longrightarrow (c,~d)\)가 다음 조건을 만족할 때를 (순)증가함수-strictly increasing function라고 한다. 임의의 \(\displaystyle x_1 ,~x_2 \in (a,~b)\)에 대하여 \(\displaystyle x_1 < x_2 \) \(\displaystyle \longrightarrow \) \(\displaystyle f(x_1 ) < f(x_2 )\) \(\displaystyle \cdots\cdots \) (i) 또, 다음을 만족하면 단조증가함수(monotonic increasing function) 또는 감소하지 않는 함수(non-decreasing function)이라 ..

[고려대 미적분학 기출] 2018년 2학기 미적분학1 -exam1 [내부링크]

1. Using the precise definition of limit, show that \(\displaystyle \lim\limits_{x \rightarrow 1}\frac{x^2 +1}{x^3 +x}=1\). (12pts) 2. If \(\displaystyle f(x)\) and \(\displaystyle g(x)\) are differentiable functions such that \(\displaystyle f \left( g(x)\right)=\tan x\) and \(\displaystyle f'(x)=1+f^2 (x)\), \(\displaystyle \left ( -\frac{\pi}{2}

[카이스트 미적분기출] 2008 Midterm Exam of Calculus I [더플러스수학] [내부링크]

1. Let \(\displaystyle f(x) = 2x + cos x \). (a) (8 points) Verify that \(\displaystyle f\) has an inverse. (b) (8 points) Find \(\displaystyle (f ^{−1} ) ( \pi)\). 2. (15 points) Let \(\displaystyle f(x) = \sqrt{4x+1}\). Use the formal \(\displaystyle \epsilon-\delta\) definition of limit with given \(\displaystyle \epsilon =1\) to explain that \(\displaystyle f \) is continuous at \(\displayst..

[수학의 기초] 삼각부등식 [더플러스수학] [내부링크]

과학고 AP-Calculus 수업을 할 때, 특히 극한을 \(\displaystyle \epsilon-\delta\) 논법을 이용하여 증명할 때, 많이 나온다. 물론 이 내용은 수학 하에서 부등식의 증명단원에서 절댓값을 포함한 부등식을 증명할 때, 예로 나온다. 먼저 중학교 1학년에서 배운 절댓값의 정의에서 시작하자. 절댓값 실수 \(\displaystyle x \)에 대하여 절댓값 \(\displaystyle x \) 즉, \(\displaystyle \left| x \right|\)는 원점으로부터 실수 \(\displaystyle x \)까지의 거리를 나타낸다. 예를 들어 \(\displaystyle \left| -5 \right|\)는 수직선에서 원점 \(\displaystyle 0 \)로부터 점 ..

[카이스트 방학숙제2] winter 2022 assignment 2 [더플러스수학] [내부링크]

Problem 1 Suppose that a function \(\displaystyle f : \mathbb{R} \rightarrow \mathbb{R}\) satisfies the following conditions for all real values \(\displaystyle x\) and \(\displaystyle y\): (i) \(\displaystyle f(x + y) = f(x) · f(y)\). (ii) \(\displaystyle f(x) = 1 + xg(x)\), where \(\displaystyle \lim\limits_{x \rightarrow 0} g(x) = 1\) Show that the derivative f′(x) exists at every value of x ..

[카이스트 방학숙제1] winter 2022 assignment 1 [더플러스수학] [내부링크]

카이스트 방학 숙제1- bridge program – winter 2022 assignment 1 Problem 1 Which of the following statements are true, and which are false? If true, try to give a convincing argument; if false, give a counter-example (that is, an example confirming the falsehood). (a) If \(\displaystyle \lim\limits_{x \rightarrow c} f(x)\) exists but \(\displaystyle \lim\limits_{x \rightarrow c} g(x)\) does not exist, the..

[수학의 기초] 함수의 극한의 엄밀한 정의(1) $\epsilon-\delta$, $\displaystyle p \rightarrow q$와 $\displaystyle \sim p ~or~ q$와 그 부정 [내부링크]

과학고 AP 수업을 할 때, 극한이 \(\displaystyle \epsilon-\delta\)로 정의되는데 이 속에 조건 \(\displaystyle p \rightarrow q\)과 모든(\(\displaystyle \Large \forall\))과 어떤(\(\displaystyle \Large \exists\))등이 포함되어 있다. 즉, 극한의 엄밀한 정의 \(\displaystyle \lim\limits_{x \rightarrow c} f(x)=L\) For all \(\displaystyle \epsilon >0\), there exists some \(\displaystyle \delta =\delta (\epsilon)\) such that for all \(\displaystyle x\),..

[수학의 기초] 디리클레 자 함수(Dirichlete ruler function)-Thomae function [내부링크]

과학고에서 배우는 AP-Calculus에서 나오는 디리클레 자함수(Dirichlete Ruler function)에 대해 알아보자. 토마스 Calculus 12판에서 나오는 디리클레 자함수를 인용하면 아래와 같다. The Dirichlet ruler function If \(\displaystyle x \) is a rational number, then \(\displaystyle x\) can be written in a unique way as a quotient of integers \(\displaystyle \frac{m}{n}\) where \(\displaystyle n >0\) and \(\displaystyle m\) and \(\displaystyle n \) have no commo..

[수학의 기초] 평균값의 정리(1) [내부링크]

평균값의 정리에 대하여 알아보고 롤의 정리를 이용하여 평균값의 정리를 증명해 보자. 롤의 정리 함수 \(\displaystyle f(x)\)가 닫힌구간 \(\displaystyle [a,~b]\)에서 연속이고 열린구간 \(\displaystyle (a,~b)\)에서 미분가능할 때, \(\displaystyle f(a)=f(b)\)이면 \(\displaystyle f'(c)=0\) 인 \(\displaystyle c\)가 열린구간 \(\displaystyle (a,~b\)에 적어도 하나 존재한다. (증명) 함수 \(\displaystyle f(x)\)가 닫힌구간 \(\displaystyle [a,~b]\)에서 연속이므로 최대 최소의 정리 의 정리를 이용하여 증명한다. 먼저 함수\(\displaystyle ..

[수학의 기초] 평균값의 정리와 구간단속 [내부링크]

\(\displaystyle 53.7 \mathrm{km}\)의 울산-포항간 고속도로가 개통되어 이동시간이 \(\displaystyle 30\)분 시대가 열렸다. 어느 날 울산에 살던 성현이가 포항에 있는 대학교 포스텍(\(\displaystyle \mathrm{POSTECH}\))에 있는 '상학'이라는 학교 선배를 만날 겸 대학을 탐방하기 위해 울산-포항간 고속도로를 이용하여 울산에서 출발하여 포항에 \(\displaystyle 30\)분만에 도착했다. 도착한 직후 교통경찰관인 준현와 성현이의 대화내용을 적어 본다. 물론 가상의 대화이다. 교통경찰 준현 : 속도를 위반하셨군요! 운전자 성현 : 무슨 말씀이세요? 저는 규정속도를 절대로 넘은 적이 없어요! 교통경찰 준현 : 당신은 오늘 아침 \(\disp..

[더플러스수학] 2022학년도 수리논술 파이널 개강 [내부링크]

#11월18일! 고3 그리고 다시 도전하는 수험생 여러분, 오늘로 #2022학년도_대학수학능력시험 이 얼마남지 않았네요... 작년부터 이어져 온 지긋지긋한 COVID-19 pandemic 소용돌이 속에서 정말 고생 많으셨습니다... 2022년 봄, 대학 캠퍼스에서 여러분의 꿈을 활짝 펼쳐 나가시길 힘껏 응원합니다! 수능이 끝나더라도 #수리논술 전형에 지원한 수험생 여러분들께서는 길게는 2주일가량을 더 수험생 신분을 유지하셔야 하네요...ㅠㅠ... #더플러스수학학원 에서는 2022학년도 각 대학 #수리논술일정 에 맞춰 다음과 같이 #수리논술FINAL수업을 준비했습니다! #수리논술FINAL_일정 #개강_11월22일_월요일 #시간_13시부터_17시 #부산대_경북대_의학계열반_자연계열반_별도운영 #11월20일_..

[더플러스수학] 고급수학 교과서 질문 복소평면 이항방정식 풀이 [내부링크]

과고2학년 학생들이 고급수학 교과서에서 질문해서 문제와 나의 풀이를 적어본다. 고급수학 교과서 심화문제 \(\displaystyle \zeta \)가 방정식 \(\displaystyle z^n = \omega \)(\(\displaystyle \neq 0 \))의 임의의 한 해라 하자. 또, \(\displaystyle x^n = 1 \)의 근(\(\displaystyle 1 \)의 \(\displaystyle n \)제곱근 중 하나인 \(\displaystyle \omega _ {n} \))에 \(\displaystyle \zeta \)를 곱해서 \(\displaystyle z ^ {n} = \omega \)의 모든 해를 만들 수 있음을 보여라. 즉, 해집합 \(\displaystyle S \)은 \(..

[더플러스수학] 사인법칙-벡터에 의한 증명 [내부링크]

사인법칙 삼각형 \(\displaystyle \mathrm {ABC}\)의 세 변 \(\displaystyle a,~b,~c\)와 세 내각 \(\displaystyle A,~B,~C\)에 대하여 \(\displaystyle \frac{a}{\sin A}= \frac{b}{\sin B}=\frac{c}{\sin C}= 2R \) (\(\displaystyle R\)은 외접원의 반지름)(\(\displaystyle R\)은 삼각형 \(\displaystyle \mathrm {ABC}\)의 외접원의 반지름) 외접원의 반지름이면 떠오르는 공식은? 답:사인법칙 더플러스수학홈페이지 연습합니다. 성공했을까요? https://youtu.be/IR9vYMHL-cY

[더플러스수학] 코시-슈바르츠 부등식, 삼각부등식-벡터에 의한 증명 [내부링크]

코시-슈바르츠 부등식 \(\displaystyle \mathbb{R}^n \)에 속하는 두 벡터 \(\displaystyle \overrightarrow {u} =(u_1 ,~u_2 ,~\cdots,~u_n) \), \(\displaystyle \overrightarrow {v} =(v_1 ,~v_2 ,~\cdots,~v_n \)에 대하여 \(\displaystyle \left| \overrightarrow {u} \cdot \overrightarrow { v} \right| \leq \left| \overrightarrow { u} \right| \left| \overrightarrow { v} \right| \) 이다. 이것을 성분으로 표현하면 \(\displaystyle \left|u_1 v_1 +..

[더플러스수학] 아폴로니우스의 원-벡터에 의한 증명 [내부링크]

아폴로니우스의 원 평면 위에서 서로 다른 두 정점 \(\displaystyle \mathrm { A,~B} \)으로부터 거리의 비가 \(\displaystyle m:n \) (\(\displaystyle m \neq n \))인 점의 자취는 선분 \(\displaystyle \mathrm { AB} \)를 \(\displaystyle m:n \)으로 내분하는 점과 \(\displaystyle m:n \)으로 외분하는 점을 지름의 양끝으로 하는 원이다. https://youtu.be/kERcL5srzyw 2020.05.17 - [수학과 공부이야기] - [수학의 기초] 아폴로니우스의 원으로 가는 길(1)-삼각형에서 각이등분선의 성질 증명 [수학의 기초] 아폴로니우스의 원으로 가는 길(1)-삼각형에서 각이등분..

[더플러스수학] 메네라우스의 정리와 그 역 정리-벡터에 의한 증명 [내부링크]

삼각형 \(\displaystyle \mathrm { ABC} \)와 직선 \(\displaystyle l \)이 존재하여 삼각형의 변 \(\displaystyle \mathrm { AB,~BC,~CA} \) 또는 그 연장선과 직선 \(\displaystyle l \)의 교점을 각각 \(\displaystyle \mathrm { P,~Q,~R} \)이라 할 때, 다음이 성립한다. \(\displaystyle \mathrm { \frac {\overline {RB} } {\overline {AR} } \times \frac {\overline {PC} } {\overline {BP} } \times \frac {\overline {QA} } {\overline {CQ} } = \mathrm { 1} } \..

[더플러스수학] 2019년 과고2학년 2학기 중간고사[ 울산과고] [내부링크]

1. 다음을 계산하시오. (5점) (1) $\displaystyle \left ( \matrix {1 & 2\\3 & 4} \right) + \left ( \matrix {3 & 4\\5 & 6} \right) $ (1점) (2) $\displaystyle \left ( \matrix {1 & 2\\0 & 1} \right) \left ( \matrix {1 & 2\\1 & 3} \right) $ (2점) (3) $\displaystyle \left ( \matrix {1 & 3 & 0\\2 & -1 & 2\\0 & 1 & 3} \right) ^ {2} $ (2점) *해설: (1) $\displaystyle \left ( \matrix {4 & 6\\8 & 10} \right) $ (2) $\displa..

[더플러스수학] 2020년 2학년 2학기 중간고사 기출[울산과고] [내부링크]

답안지의 각 장마다 학년, 반, 번호, 이름을 표기하고, 해당란에 답안을 작성하시오. 모든 문항에는 부분점수가 있으므로, 과정을 반드시 적으시오. 모든 문항은 서술형입니다. 1. 행렬 \(\displaystyle A \)가 성분이 \(\displaystyle a _ {ij} \)인 \(\displaystyle m \times n \)행렬일 때, \(\displaystyle A=[a _ {ij} ] _ {m \times n} \)이라 한다. 2. 정칙행렬 : 역행렬이 존재하는 행렬 1. 두 행렬 \(\displaystyle A= \left ( \matrix {1 & 0 & -1\\0 & 2 & 0\\-1 & 0 & 1} \right) \), \(\displaystyle B= \left ( \matr..

[더플러스수학] 울산과고 고급수학 프린트 1 [내부링크]

1. 다음 사상 중에서 선형사상인 것을 찾아라. (1) \(\displaystyle T~:~\mathbb{R}^3 \longrightarrow \mathbb{R}^2 ,~T(x_1 ,~x_2 ,~x_3 )=(3x_1+2x_3 ,~x_2 )\) (2) \(\displaystyle T~:~\mathbb{R}^2 \longrightarrow \mathbb{R}^2 ,~T(x_1 ,~x_2 )=( \left|x_1 \right| ,~x_2 )\) (3) \(\displaystyle T~:~\mathbb{R}^2 \longrightarrow \mathbb{R}^2 ,~T(x_1 ,~x_2 )=(x_1 +1 ,~x_1 +x_2 )\) (4) \(\displaystyle T~:~\mathbb{R_3}[x] \long..

[더플러스수학] 수반행렬과 역행렬, 크래머 정리 -2 [내부링크]

연립일차방정식의 풀이 방법에는 '가우스소거법', '역행렬'과 '크래머 공식' 등이 있다. 우리는 앞에서 수반행렬을 통해 역행렬을 구하는 방법을 보았다. 이것을 이용하여 크래머 공식을 유도하고자 한다. 간단한 연립방정식을 예를 들어 해를 구하고 그 과정에서 크래머 공식이 어떻게 쓰이는지 한 번 보자. \(\displaystyle \begin{cases} x+2y=7 \\ 2x+3y=1\end{cases}\) 이것을 행렬로 나타내면 \(\displaystyle \begin{pmatrix} 1&2\\2&3\end{pmatrix}\begin{pmatrix} x\\y\end{pmatrix}=\begin{pmatrix} 7\\1\end{pmatrix}\) \(\displaystyle A=\begin{pmatrix}..

[더플러스수학] 수반행렬과 역행렬, 크래머 정리 -1 [내부링크]

행렬식을 이용하여 역행렬을 구하는 과정을 알아보자. 정의1. 정사각행렬 \(\displaystyle A=(a_{ij})_{n \times n}\)에서 \(\displaystyle A\)의 각 원소 \(\displaystyle a_{ij}\)의 여인수 \(\displaystyle A_{ij}\)로 이루어진 행렬 \(\displaystyle \left(A_{ij} \right) _{n \times n}\)을 \(\displaystyle A\)의 여인수행렬 이라 한다. 또, \(\displaystyle A\)의 여인수 행렬의 전치행렬을 \(\displaystyle A\)의 수반행렬이라고 하고 \(\displaystyle adj A\)로 나타낸다. 즉, \(\displaystyle adj A = \left(A_..

[평가원기출]고3 2022학년도 9월 평가원 22번 [내부링크]

22. 최고차항의 계수가 \(\displaystyle 1\)인 삼차함수 \(\displaystyle f(x)\)에 대하여 함수 \(\displaystyle g(x)= f(x-3)\times \lim_{h \rightarrow 0+}\frac{\left|f(x+h) \right| -\left|f(x-h) \right|}{h} \) 가 다음 조건을 만족시킬 때, \(\displaystyle f(5)\)의 값을 구하시오. [\(\displaystyle5\)점] (가) 함수 \(\displaystyle g(x)\)는 실수 전체의 집합에서 연속이다. (나) 방정식 \(\displaystyle g(x)=0\)은 서로 다른 네 실근 \(\displaystyle \alpha_1 ,~\alpha_2 ,~\alpha_3 ..

[평가원기출]고3 2022학년도 9월 평가원 기하 30번 [내부링크]

좌표평면에서 세 점 \(\displaystyle \mathrm{A}(3,~-1),~\mathrm {B}(0,~2),~\mathrm{C)}(1,~0)\)에 대하여 두 점 \(\displaystyle \mathrm{P,~Q}\)가 \(\displaystyle \left| \overrightarrow {\mathrm{AP}} \right|=1,~\left| \overrightarrow {\mathrm{BQ}} \right|=2,~\overrightarrow {\mathrm{AP}}\bullet \overrightarrow{\mathrm{OC}} \geq \frac{\sqrt2}{2}\) 를 만족시킬 때, \(\displaystyle \overrightarrow {\mathrm{AP}}\bullet \overr..

[더플러스수학] 고급수학1 det(AB)=det(A)det(B) [내부링크]

이 글에서는 \(\displaystyle n\)차 정사각행렬 \(\displaystyle A,~B\)에 대하여 \(\displaystyle det(AB)=det(A)det(B) \)임을 증명해 보도록 하자. 먼저 행렬 \(\displaystyle A\)가 비가역행렬(특이행렬)일 때와 가역행렬일 때로 나누어서 증명한다. 먼저 비가역행렬일 때의 증명에 앞어서 다음의 보조정리를 증명하자. 보조정리1. 행렬 \(\displaystyle A\)가 비가역행렬이면 행렬 \(\displaystyle AB\)도 비가역행렬이다. 주의하자. 행렬식의 성질 \(\displaystyle det(AB)=det(A)det(B) \)을 이용하여 위의 보조정리를 증명할 수 있다. 그러나 여기서는 쓰면 안된다. 왜냐하면 \(\displ..

[더플러스수학] 고급수학1-행렬식(2) [내부링크]

2021.08.23 - [수학과 공부이야기/선형대수학] - [더플러스수학] 고급수학1-행렬식(1) [더플러스수학] 고급수학1-행렬식(1) 행렬식에 대해 알아보자. 먼저 행렬은 수를 직사각형 모양으로 배열한 것이고 행렬식은 정사각행렬에 대해서만 정의되고 하나의 실숫값이다. 다르게 말하면 행렬식은 정사각행렬에서 실수로 plusthemath.tistory.com 앞의 글에서 행렬식의 정의에 대하여 알아 보았다. 여기서는 행렬식의 성질을 고찰해 보자. \(\displaystyle n\)차 정사각행렬 \(\displaystyle A\)의 행렬식(determinant)는 \(\displaystyle A\)의 임의의 어느 한 행이나 열에 대해 여인수 전개해도 똑같으면 그 전개한 값을 행렬 \(\displaystyle..

[더플러스수학] 가우스 소거법 - 기본 행 연산, 기본 행렬 [내부링크]

연립방정식을 풀 때, 문자를 소거하느라 단순 계산 과정이 엄청 많다는 것을 경험한 적이 있을 것이다. 특히 연립방정식 중에서 미지수의 개수가 식의 개수보다 많은 연립방정식이면 역행렬이 존재하지 않아서 해를 구하기가 힘들다. 또, 역행렬이 존재한다 하더라도 행렬의 크기가 커 역행렬 구하기가 힘들 때, 우리는 가우스 소거법을 많이 쓴다. 또, 컴퓨터 프로그램으로 짜기도 쉬어 가우스 소거법을 이용하여 해를 구하는 과정이 쉬워진다. 가우스 소거법에 대하여 알아보자. 다음과 같은 \(\displaystyle 3\)개의 미지수를 가진 연립일차방정식을 생각해보자. \(\displaystyle \begin{cases} a_{11}x +a_{12}y+a_{13}z=b_{1} \\a_{21}x +a_{22}y+a_{23}..

[더플러스수학] 고급수학1-행렬식(1) [내부링크]

행렬식에 대해 알아보자. 먼저 행렬은 수를 직사각형 모양으로 배열한 것이고 행렬식은 정사각행렬에 대해서만 정의되고 하나의 실숫값이다. 다르게 말하면 행렬식은 정사각행렬에서 실수로 가는 일종의 함수라고 생각할 수 있다. 정의1. 행렬식(determinant of \(\displaystyle A\)) \(\displaystyle n\)차의 정사각행렬 \(\displaystyle A=(a_{ij})_{n \times n}\)의 행렬식을 \(\displaystyle \left| A \right| \) 또는 \(\displaystyle det(A)\)로 나타내며 다음과 같이 귀납적으로 정의한다. (1) \(\displaystyle n=1\)일 때, \(\displaystyle \left | a_{11} \righ..

[더플러스수학] 필요충분조건-특이행렬과 고윳값 0 [내부링크]

문제의 상황 고급수학 연습문제 중 "행렬 \(\displaystyle A\)가 역행렬을 갖지 않을 필요충분조건은 행렬 \(\displaystyle A\)의 고윳값 중 \(\displaystyle 0\)이 있다." 영어로 "Show that a matrix \(\displaystyle A\) is singular if and only if \(\displaystyle 0\) is an eigenvalue of \(\displaystyle A\)" (증명) \(\displaystyle (\Longleftarrow )\) (증명1) 행렬 \(\displaystyle A\)가 고윳값 \(\displaystyle 0\)을 갖는다고 가정하자. 고윳값의 정의에 의해 \(\displaystyle A \vec x = \..

[더플러스수학] 대칭행렬과 교대행렬 [내부링크]

정의1. 전치행렬(transpose matrix) 행렬 \(\displaystyle A=(a_{ij})_{m \times n} \in M_{\textcolor{red}{m \times n} }(\mathbb{R})\)에 대하여 행과 열을 바꾼 새로운 행렬 \(\displaystyle B=(b_{ij})_{n \times m} \in M_{ \textcolor {blue}{n \times m} }(\mathbb{R})\)을 행렬 \(\displaystyle A \)의 전치행렬이라 하고 \(\displaystyle A^T \)로 쓴다. 여기서 \(\displaystyle b_{\textcolor{red}{ij}} =a_{\textcolor{red}{ji}} \)이다. 노트) \(\displaystyle M_..

[더플러스수학] (AB)^T=B^T A^T 증명 (전치행렬의 성질) [내부링크]

\(\displaystyle (AB)^T =B^T A^T \)을 증명해보자. 먼저 행렬 \(\displaystyle A\)를 \(\displaystyle l \times m \)행렬, 행렬 \(\displaystyle B \)를 \(\displaystyle m \times n \) 행렬이라 하자. 행렬 \(\displaystyle AB\)는 \(\displaystyle l \times n \) 행렬이다. 즉 \(\displaystyle A=(a_{ij})_{l \times m}\), \(\displaystyle B=(b_{ij})_{m \times n}\) 으로 놓을 수 있다. 그러면 행렬 \(\displaystyle (AB)^T\)는 \(\displaystyle n \times l \) 행렬이다. 행렬..

[더플러스수학] 수학공부법-친구를 가르쳐라! [내부링크]

https://169254.tumblr.com/post/92633286949 머리 나쁘면 공부해도 소용이 없을까? 그런데 아무리 노력해도 타고난 재능을 못 당한다고 주장하는 도발적인 논문이 발표됐습니다. 미국 미시건 주립대 심리학과 자크 햄브릭 교수팀의 논문인데요. 연습이 얼마나 실력을 향상시키 169254.tumblr.com 요약하면 «학생들은 보통 공부를 하다가 알 것 같다는 기분이 들면 그 내용은 그만 공부하고 다음 내용으로 넘어가게 된다. 문제는 그 기분이 별로 정확하지 않다는 것이다. 그래서 실제로는 잘 모르는데도 충분히 공부를 안하게 된다. 쉬운 부분을 대충 공부하고 넘어가서 남는 시간을 학생들은 어려운 부분을 이해하는데 쏟아 붓는다. 그런데 어려운 부분은 공부의 효율이 낮기 때문에 시간만 많..

[더플러스수학] 3차함수의 접선의 갯수 [내부링크]

3차함수의 접선이 개수 삼각함수 \(\displaystyle f(x)=ax^3 +bx^2 +cx+d~( a > 0)\)와 변곡점에서의 접선을 기준으로 평면 위의 점에서 삼차함수에 그을 수 있는 접선의 개수는 아래의 그림과 같다. 이것을 증명하기 위해 삼차함수의 변곡점을 \(\displaystyle (\alpha,~\beta )\) 라 할 때, \(\displaystyle f''(x)=6a(x-\alpha)\) 라 두면 적분을 통해 \(\displaystyle f'(x)=3a(x-\alpha)^2 + b\) \(\displaystyle f(x)=a(x-\alpha)^3 + b(x-\alpha)+ \beta\) 로 표현할 수 있다. 여기서 \(\displaystyle f(x)\)를 \(\displaystyl..

[더플러스수학] 케일리-해밀턴 정리의 증명 - 고윳값, 고유벡터 이용 [내부링크]

과학고 학생들과 고급수학1 수업을 하는 과정에서 고윳값, 고유벡터, 특성 다항식, 케일리 해밀턴 정리를 만나게 되었다. 이 내용에 대하여 가볍게 정리하고, \(\displaystyle 2\)차 정사각형렬 \(\displaystyle A\)가 \(\displaystyle 2\)개의 서로 다른 고윳값과 고유벡터를 가질 때, 케일리 해밀턴 정리가 성립함을 증명해 보겠다. 정의 고윳값, 고유벡터, 특성다항식, 특성방정식 \(\displaystyle 2\)차 정사각형렬 \(\displaystyle A\)와 실수 \(\displaystyle \lambda\)에 대하여 \(\displaystyle A \vec x =\lambda \vec x \) 를 만족하는 \(\displaystyle \vec 0\)이 아닌 벡터 ..

[더플러스수학] 카탈란 수 -활용(5) [내부링크]

이제 카탈란 수를 이용하여 경우의 수를 구하는 문제를 찾아보자.문제1. 한국과 일본이 야구시합을 해서 한국이 $ 5 $대 $ 4 $로 이겼다. 한국이 일본에 리드를 당하지 않고서 즉, 이기거나 비길 수는 있어도 역적은 당하지 않은 채 시합에서 이길 경우의 수를 구하여라.(풀이)더보기 한국이 일본에 \(5\) 대 \(4\)로 이기는 총 경우의 수는 위의 그림에서 \((0,~0)\)에서 \((5,~4)\)에 도달하는 경우의 수\(\displaystyle \frac{9!}{5! \times 4!}={}_9 \mathrm{C}_5 \)이다. 이 경우의 수 중 역전을 허용하지 않으면서 \((5,~4)\)에 도달하려면 위의 경우에서 역전을 허용하면서 \( (5,~4)\)에 도달하는 경우의 수를 빼 주면 된다.위의 ..

[더플러스수학] 카탈란 수 - 생성함수(4) [내부링크]

2021.08.03 - [수학과 공부이야기] - [더플러스수학] 카탈란 수 - 점화식(3) [더플러스수학] 카탈란 수 - 점화식(3) 카탈란 수란 무엇인가?에 대한 글 2021.08.02 - [수학과 공부이야기] - [더플러스수학] 카탈란 수 (1) 카탈란 수의 일반항에 대한 설명글 2021.08.03 - [수학과 공부이야기] - [더플러스수학] 카탈란 수 - plusthemath.tistory.com 2021.08.03 - [수학과 공부이야기] - [더플러스수학] 카탈란 수 - 일반항(2) [더플러스수학] 카탈란 수 - 일반항(2) 2021.08.02 - [수학과 공부이야기] - [더플러스수학] 카탈란 수 (1) [더플러스수학] 카탈란 수 (1) 이산수학 최경식(경문사)에서의 예에서 시작해보자. 요금이..

[더플러스수학] 카탈란 수 - 점화식(3) [내부링크]

카탈란 수란 무엇인가?에 대한 글 2021.08.02 - [수학과 공부이야기] - [더플러스수학] 카탈란 수 (1) [더플러스수학] 카탈란 수 (1) 이산수학 최경식(경문사)에서의 예에서 시작해보자. 요금이 \(\displaystyle 5,000\)원인 어떤 영화가 보고싶어 \(\displaystyle 6\)명이 극장에 들어가려고 한다. 그 중 \(\displaystyle 3\)명은 \(\displaystyle.. plusthemath.tistory.com 카탈란 수의 일반항에 대한 설명글 2021.08.03 - [수학과 공부이야기] - [더플러스수학] 카탈란 수 - 일반항(2) [더플러스수학] 카탈란 수 - 일반항(2) 2021.08.02 - [수학과 공부이야기] - [더플러스수학] 카탈란 수 (1) ..

[더플러스수학] 카탈란 수 - 일반항(2) [내부링크]

2021.08.02 - [수학과 공부이야기] - [더플러스수학] 카탈란 수 (1) [더플러스수학] 카탈란 수 (1) 이산수학 최경식(경문사)에서의 예에서 시작해보자. 요금이 \(\displaystyle 5,000\)원인 어떤 영화가 보고싶어 \(\displaystyle 6\)명이 극장에 들어가려고 한다. 그 중 \(\displaystyle 3\)명은 \(\displaystyle.. plusthemath.tistory.com 앞 편의 글에서 카탈란 수 \(\displaystyle C_n \)가 무엇인지 또, 실생활에서는 어떻게 활용되는지에 대해 알아 보았다. 이 편에서는 카탈란 수 \(\displaystyle C_n \)의 일반항이 어떤 과정을 거쳐서 \(\displaystyle C_n = \frac{1}..

[더플러스수학] 2020학년도 성균관대 수시모집 논술우수전형 논 술 시 험 (자 연 2) [내부링크]

[ 수학 1 ] 다음 ~ 를 읽고 [수학 1 -ⅰ] ~ [수학 1 -ⅳ]를 문항별로 풀이와 함께 답하시오. 쌍곡선 \(\displaystyle x^2 -2y^2 =-1\)의 윗부분을 \(\displaystyle C_1 \)으로 하고, 아랫부분을 \(\displaystyle C_2 \)라고 하자. 쌍곡선 밖의 한 점 \(\displaystyle \mathrm{P}\)에서 곡선 \(\displaystyle C_1 \)에 그은 접선의 접점을 \(\displaystyle \mathrm{Q}\), 점 \(\displaystyle \mathrm{P}\)에서 곡선 \(\displaystyle C_2 \)에 그은 접선의 접점을 \(\displaystyle \mathrm{R}\)로 표기하자. \(\displaystyle..

[더플러스수학] 2020학년도 성균관대 수시모집 논술우수전형 논 술 시 험 (자 연 1) [내부링크]

[ 수학 1 ] 다음 ~ 을 읽고 [수학 1 -ⅰ] ~ [수학 1 -ⅲ]을 문항별로 풀이와 함께 답하시오. 표본공간 \(\displaystyle S\) 에서 각각의 근원사건이 일어날 가능성이 모두 같을 때, 사건 \(\displaystyle A\) 가 일어날 수학적 확률은 다음과 같다. \(\displaystyle \mathrm {P}(A)=\frac{n(A)}{n(S)} \) 사건 \(\displaystyle A\)가 일어났을 때의 사건 \(\displaystyle B\)의 조건부확률은 \(\displaystyle \mathrm{P}(B \vert A)=\frac{\mathrm {P}(A \cap B)}{\mathrm{P}(A)}\) 이다. (단, \(\displaystyle \mathrm{P}(A)>..

2021학년도 서울대 일반전형 구술문제-공과대학, 농업생명과학대학 [내부링크]

공과대학 | 농업생명과학대학 문제 1. 두 함수 \(\displaystyle g_1 (x)\)와 \(\displaystyle g_2 (x)\)가 아래와 같이 주어져 있다. \(\displaystyle g_1 (x)=\begin{cases} 0&(-1 \leq x < 0)\\1&(0 \leq x \leq 1)\end{cases}\) \(\displaystyle g_2 (x)=\sin(4\pi x)~(0 \leq x \leq 1)\) 합성함수 \(\displaystyle h(x)=(g_1 \circ g_2 )(x)\)에 대하여 다음 질문에 답하시오. 1-1. 함수 \(\displaystyle y=h(x) ~(0 \leq x \leq 1\) 의 그래프와 이차함수 \(\displaystyle y=-6x(x-b)..

2021학년도 서울 일반전형 면접 및 구술고사[수학]-수리통계, 수학교육 [내부링크]

문제 1. 음이 아닌 정수들의 집합을 \(\displaystyle X \) 라고 하고, 음이 아닌 실수들의 집합을 \(\displaystyle Y \)라고 하자. 두 함수 \(\displaystyle f :X\rightarrow Y \)와 \(\displaystyle g :Y \rightarrow X \)에 대해 아래 조건을 생각하자. (조건 1) \(\displaystyle n \in X,~y \in Y\) 에 대하여 \(\displaystyle f(n) \leq y \Longleftrightarrow n \leq g(y)\)이다. 1-1. 함수 \(\displaystyle f :X\rightarrow Y \), \(\displaystyle g :Y \rightarrow X \)가 (조건 1)을 만족할..

2021학년도 서울 일반전형 면접 및 구술고사[수학]-경제,자유전공 [내부링크]

문제 1. 다항식 \(\displaystyle g(x)=x^4 +x^3 +x^2 +x+1 \)에 대하여 다음 물음에 답하시오. 1-1. \(\displaystyle x^5 \)을 \(\displaystyle g(x) \)로 나눈 나머지를 구하시오. 1-2. 자연수 \(\displaystyle n \)에 대하여 \(\displaystyle f_n (x)=(x^3 +x^2 +3)^n \)이라 하자. \(\displaystyle f_n (x) \)를 \(\displaystyle g (x)\)로 나눈 나머지를 \(\displaystyle r_n (x)= a_n x^3 +b_n x^2 +c_n x+d \) (단, \(\displaystyle a_n ,~b_n ,~c_n ,~d_n \)은 정수) 라고 쓰자. 모든 ..

2021학년도 서울 일반전형 면접 및 구술고사[수학]-경영,경제 [내부링크]

2021학년도 서울 일반전형 면접 및 구술고사[수학]-사회과학대학 경제학부|경영대학|농업생명과학대학 농경제사회학부| 생활과학대학 소비자아동학부(소비자학전공), 의류학과|자유전공학부(인문) 문제 1. 두 함수 \(\displaystyle g_1 (x)\)와 \(\displaystyle g_2 (x)\)가 아래와 같이 주어져 있다. \(\displaystyle g_1 (x)=\begin{cases} 0&(-1 \leq x < 0)\\1&(0 \leq x \leq 1)\end{cases}\) \(\displaystyle g_2 (x)=\sin(4\pi x)~(0 \leq x \leq 1)\) 합성함수 \(\displaystyle h(x)=(g_1 \circ g_2 )(x)\)에 대하여 다음 질문에 답하시오. ..

2020학년도 서울 일반전형 면접 및 구술고사[수학]-자연,공대 [내부링크]

문제 1. 좌표공간에서 \(\displaystyle 0\) 이상의 정수 \(\displaystyle n\) 에 대하여 평면 \(\displaystyle \alpha_n ,~\beta_n \)을 다음과 같이 정의하자. (i) 평면 \(\displaystyle \alpha_n \) 은 점 \(\displaystyle (1,~0,~1)\)을 지나고 \(\displaystyle xy\)평면과의 교선의 방정식이 \(\displaystyle x+y=n,~z=0 \) 이다. (ii) 평면 \(\displaystyle \beta_n \) 은 점 \(\displaystyle (0,~0,~1)\)을 지나고 \(\displaystyle xy\)평면과의 교선의 방정식이 \(\displaystyle x-y=n,~z=0 \) 이..

2020학년도 서울 일반전형 면접 및 구술고사[수학]-경제,자유전공 [내부링크]

문제 1. 곡선 \(\displaystyle C \) 와 직선 \(\displaystyle l\)이 점 \(\displaystyle \mathrm{A}\)에서 만나고, 점 \(\displaystyle \mathrm{A}\)에서의 곡선 \(\displaystyle C\)에 대한 접선이 직선 \(\displaystyle l\)과 수직일 때 \(\displaystyle C\) 와 \(\displaystyle l\)이 점 \(\displaystyle \mathrm{A}\)에서 수직으로 만난다고 한다. 곡선 \(\displaystyle y=x^3\)을 \(\displaystyle T\) 라고 하자 1-1. 좌표평면 위의 한 점 \(\displaystyle (a,~b)\)를 지나는 직선 \(\displaystyle..

2020학년도 서울 일반전형 면접 및 구술고사[수학]-경영,경제 [내부링크]

2020학년도 서울 일반전형 면접 및 구술고사[수학] -사회과학대학 경제학부|경영대학|농업생명과학대학 농경제사회학부|생활과학대학 소비자아동학부(소비자학전공), 의류학과|자유전공학부 문제 1. 자연수 \(\displaystyle n\)에 대하여 다음의 조건을 만족하는 원 \(\displaystyle A_n\) 을 생각해보자. (i) \(\displaystyle A_1\)의 중심은 \(\displaystyle (0,~0)\)이고 반지름은 \(\displaystyle 4\) 이다. (ii) \(\displaystyle A_n\)의 중심은 \(\displaystyle \left( \lim\sum_{i=1}^{n-1} \frac{15}{2^i},~0\right)\)이고 반지름은 \(\displaystyle \fr..

[신정수학학원][옥동수학학원]##[더플러스수학] 카탈란 수 (1) [내부링크]

이산수학 최경식(경문사)에서의 예에서 시작해보자. 요금이 \(\displaystyle 5,000\)원인 어떤 영화가 보고싶어 \(\displaystyle 6\)명이 극장에 들어가려고 한다. 그 중 \(\displaystyle 3\)명은 \(\displaystyle 5,000\)원짜리를 가지고 있고 나머지 \(\displaystyle 3\)명은 \(\displaystyle 10,000\)원짜리를 가지고 있다. 매표소에는 잔돈이 준비되어 있지 않다. 모두 입장할 수 있도록 일렬로 서는 방법의 수를 구한다. 단, 사람들은 구별하지 않는다. 즉, 같은 돈을 가진 사람이면 누가 앞에 서든지 문제삼지 않는다. 풀이) 사람들을 구별하지 않으므로 \(\displaystyle 5,000\)짜리를 가진 사람을 \(\dis..

[더플러스수학] 롱테일 급수 [내부링크]

(가) 다음과 같이 자연수의 거듭제곱의 역수로 이루어진 무한급수의 합을 구하는 수학 문제는 아주 오래 됐다. \(\displaystyle S _ {p} = \sum\limits _ {n=1} ^ {\infty } \frac {1} {n ^ {p} } = \frac {1} {1 ^ {p} } + \frac {1} {2 ^ {p} } + \frac {1} {3 ^ {p} } + \cdots ~~~ ( p=1,~2,~3 \cdots ) \) \(\displaystyle p=1 \)일 때는 이른바 ‘조화급수’를 얻는데, 이때의 합은 1350년경 이래 발산하는 것으로 알려졌다. 이 결과는 중세 프랑스의 수학자이자 철학자인 오렘(Nicole Oresme, 1320-82)이 처음으로 밝혔다. (중략) 그런데 \(\d..

한양대학교 2021학년도 논술전형 자 연 계 열 (오후 1) [내부링크]

[문제 1] 다음 제시문을 읽고 물음에 답하시오. (50점) 학생 \(\displaystyle A\)와 \(\displaystyle B \) 가 다음과 같이 야구방망이를 휘둘러서 공을 치는 놀이를 한다. (1) 공을 쳐서 날아간 거리가 \(\displaystyle 50 \mathrm{m} \) 이상인 경우 \(\displaystyle 2\) 점, 공을 쳐서 날아간 거리가 \(\displaystyle 50 \mathrm{m} \) 미만인 경우 \(\displaystyle 1 \mathrm{m} \) 점, 공을 치지 못한 경우 \(\displaystyle 0 \) 점을 얻는다. (2) 학생 \(\displaystyle A\)와 \(\displaystyle B \) 가 다음과 같은 확률로 공을 친다. 1. 학..

한양대학교 2021학년도 논술전형 자 연 계 열 (오전) [내부링크]

[문제 1] 다음 물음에 답하시오. (50점) 1. 곡선 \(\displaystyle y=e^x ~(0 \leq x \leq \ln t )\)와 \(\displaystyle y\)축, 직선 \(\displaystyle y=t\)로 둘러싸인 도형을 밑면으로 하는 두 입체도형 \(\displaystyle A\) 와 \(\displaystyle B\)가 있다. 도형 \(\displaystyle A\)는 \(\displaystyle y\)축에 수직인 평면으로 자른 단면이 모두 정사각형이고, 도형 \(\displaystyle B\) 는 \(\displaystyle x\)축에 수직인 평면으로 자른 단면이 모두 정삼각형이다. 도형 \(\displaystyle A \)의 부피를 \(\displaystyle V(t) \..

한양대학교 2021학년도 논술전형 자연계열(의예과) [내부링크]

[문제 1] (가)의 핵심 내용을 ‘~가설’(전체 글자 수 10자 이내)의 형식으로 정리하고 그렇게 명명한 이유 를 밝힌 다음, 그 가설과 관련하여 (나)가 의미하는 바를 논하고, 이를 바탕으로 (다)가 시사하는 바를 설 명하시오. (600자, 50점) (가) "인간과 동물의 감정 표현"(1872)에서 찰스 다윈은 모든 인간의 웃음 짓기와 눈살 찌푸리기는 진화적 적응의 일환으로 하는 행동이라고 주장했다. 서로에게 자신의 감정을 정확하고 신속하게 전달하는 것이야말로 인류 생존에 중요한 일이었기 때문에 얼굴이 마음의 게시판으로 발전했다는 것이다. 일반적으로 다윈의 이러한 견해는 얼굴에 표현된 기본적인 감정의 의미를 인간 이 판독해내는 능력 또한 생물학적으로 설명될 수 있다는 주장으로 받아들여진다. (나) \..

2021학년도 연세대 수리논술 자연2 [내부링크]

[문제 1] 한 개의 주사위를 \(\displaystyle 3\)번 던져 나온 눈의 수를 차례로 \(\displaystyle a,~b,~c\)라 하자. 이차방정식 \(\displaystyle x^2 +y^2 +ax+by+6=0 \)이 원을 나타낼 때, 방정식 \(\displaystyle x+2y+c=0 \)이 나타내는 직선이 이 원의 넓이를 이등분할 확률을 구하시오. [10점] [문제 2] 방정식 \(\displaystyle x_1 +x_2 +x_3 = 5\)를 만족시키는 양의 정수해를 과 같이 나타냈을 때, 숫자 \(\displaystyle 2 \)가 나오는 횟 수는 \(\displaystyle 6\)이다. 자연수 \(\displaystyle n \)에 대하여 방정식 \(\displaystyle x_1..

2021학년도 연세대 수리논술 자연1 [내부링크]

[문제 1] \(\displaystyle 100\)명의 학생 중 \(\displaystyle k\)명을 선정하여, 두 명을 회장, 다른 다섯 명을 부회장, 나머지는 위원으로 임명하는 경우의 수가 최대가 되도록 하는 모든 \(\displaystyle k\)의 값을 구하시오. (단, \(\displaystyle 10 \leq k \leq 100\)) [10점] [제시문] 실수 전체의 집합에서 정의된 연속함수 \(\displaystyle g(x)\)가 다음 조건을 만족시킬 때, 다음 물음에 답하시오. (가) \(\displaystyle g(2020)=1\) (나) 임의의 실수 \(\displaystyle a,~b\)에 대하여 \(\displaystyle g(a+b)+g(a-b)=2g(a) \cos b \pi\..

[전자책] 수리논술 심층면접대비 미적분 제1부 [더플러스수학] [내부링크]

수리논술 심층면접 전자책 출간했습니다! 서울대 카이스트 심층면접준비하는 학생! 연세대 등 수리논술을 준비하는 학생들이 보면 좋은 책! 각 문제별 저자 직강 동영상이 링크로 연결된 책! 자기주도적으로 공부할 수 있는 책입니다! 수리논술 심층면접대비 미적분 제1부 권도형 저 서울대, 카이스트, 연세대 등 각 대학별 수리논술, 수학 심층면접을 어떻게 준비할까? 고민이 많는 학생들에게 드리는 전자책! 풀이 동영상이 있는 전자책! 수능수학! 수리논술! 서로 다르게 준비해야 할까? 아니다. 문제는 수학의 깊이!수능 수학 일등급을 맞고자 하는 학생은 반드시 수리논술 문제, 즉 서술형 주관식 문제를 반드시 풀어야 한다. 또 수능의 합답형 문제 ㄱ, ㄴ, ㄷ 선택하는 문제는 수리논술 문제이다. 이 문제의 풀이과정을 서술..

2022학년도 평가원 6월 킬러문제 기하 29-30번 [내부링크]

더플러스수학 킬러문항 문제 및 풀이 동영상 29. 포물선 \(\displaystyle y ^ {2} =8x \)와 직선 \(\displaystyle y=2x-4 \)가 만나는 점 중 제\(\displaystyle 1 \)사분면 위에 있는 점을 \(\displaystyle \mathrm {A} \)라 하자. 양수 \(\displaystyle a \)에 대하여 포물선 \(\displaystyle ( y-2a) ^ {2} =8 ( x-a) \)가 점 \(\displaystyle \mathrm { A} \)를 지날 때, 직선 \(\displaystyle y=2x-4 \)와 포물선 \(\displaystyle ( y-2a) ^ {2} =8 ( x-a) \)가 만나는 점 중 \(\displaystyle \mathr..

[더플러스수학] 함수 \(\displaystyle f \)가 일대일대응이면 역함수 \(\displaystyle f^{-1}\)도 일대일대응이다. [내부링크]

정리. 함수 \(\displaystyle f:X \rightarrow Y \)가 일대일대응(즉, 일대일함수이고, 치역과 공역이 같은 함수)이면, \(\displaystyle f ^ {-1} :Y \rightarrow X \)도 일대일대응이다. 이 때, \(\displaystyle f ^ {-1} \)을 \(\displaystyle f \)의 역함수(inverse function)이라고 한다. (증명) \(\displaystyle f \)가 함수이므로 함수 \(\displaystyle f \)의 정의역 \(\displaystyle X \)의 모든 원소의 함숫값이 집합 \(\displaystyle y \)에 존재하므로 \(\displaystyle f ^ {-1} \)의 공역 \(\displaystyle X ..

[옥동 수학학원] 2021년 1학기 울산과학고 중간고사 [내부링크]

T.S ELIOT이 '황무지'라는 시에서 지칭한 잔인한 4월 (시에서는 생명과 관련한 이야기지만ㅎㅎ)이라 함은 대한민국 중고등학생들의 중간고사 기간을 미리 예견해서 일까요?ㅎㅎ 하지만 우리 #더플러스수학학원 친구들은 기분 좋게 5월을 맞이하고 있을 거라 믿습니다. #2021년_1학년_1학기_중간고사 문제를 보는 순간 눈에 익은 문제들이 많이 있었을 거예요. 중간고사 문제를 분석하면서 우리 친구들이 매 수업 전에 시행했던 #TEST 문제와 유사한 문항이 많아 우리 친구들의 이번 중간고사 결과에 큰 기대를 가지고 있습니다^^ #연조립제법 문항, #소수관련 #부정방정식 문항, #중간고사_파이널_수업에서 원장님께서 보여주셨던 #드모르간법칙_증명과 #대칭차집합의 증명 문제, 특히 #부분집합_원소_합에 관한 ..

[옥동 수학학원] 2021년 울산과학고 중간고사 대비 1 [내부링크]

https://m.blog.naver.com/plusthemath/222305635164[옥동 수학학원] 2021년 울산과학고 중간고사 대비 1개학 한지도 벌써 1달이 훌쩍 넘어 이제 중간고사가 얼마 남지 않았네요~#울산과학고등학교_1_2_3학년 친구...blog.naver.com개학 한지도 벌써 1달이 훌쩍 넘어 이제 중간고사가 얼마 남지 않았네요~ #울산과학고등학교_1_2_3학년 친구들 마음도 조금씩 조급해지는 듯합니다. 지금까지 차곡차곡 쌓아온 문제해결 능력을 최대한 끌어올려야 할 시기가 되었어요... 원장님 이하 #더THE플러스수학학원 선생님들은 항상 우리 친구들이 최선의 결과를 얻을 수 있게 노력합니다. 우리 학생들에게 공지한 것처럼 #주말_정규수업 이외에 #울산과학고_중간고사대비 #4월9일(금..

[옥동수학학원][수학의 기초] 정적분의 정의(1)-리만합, 상합, 하합의 관계 [내부링크]

미적분 수업을 하면서 정적분의 정의를 고등학교과정을 넘어 대학에서는 어떻게 정의하고 있는지를 학생들에게 알려 줄 필요가 있어서 이우출판사의 미적분학(저자 김정수, 박을용, 이을용, 윤옥경 등등) 1988년 책-오래된 책 ㅋㅋ 본인의 대학교 1학년교재-을 참조하여 정리해 나가고자 한다. 증명은 최대한 본인이 이해한 방식으로 적을 것이다. 그러므로 오류가 있다면 그것은 나로 인한 것이다. 고등학교에서는 구간 \(\displaystyle I=[a,~b] \) 를 \(\displaystyle n \) 등분-균등분할하여 구분구적법으로 정적분을 정리하지만 대학에서는 균등분할하지 않고 임의로 분할하여 정적분을 정리해 간다. 먼저 구간의 분할에서 출발하자. 정의1. 구간의 분할, 분할의 크기(norm of partit..

한 문제의 두 이야기 1 - 수학상 5-18번 [내부링크]

(구독과 좋아요) $\displaystyle a,~b $가 유리수일 때, 모든 자연수 $ \displaystyle n $에 대하여 $$ \displaystyle( a+b \sqrt {5} ) ^ {n+2} = ( a+b \sqrt {5} ) ^ {n+1} + ( a+b \sqrt {5} ) ^ {n} $$이 성립하도록 $\displaystyle a,~b $의 값을 정하여라. 단, $\displaystyle b \neq 0 $이다. (풀이1) 더보기 정답 $\displaystyle {a= \frac {1} {2} ,~b=\pm \frac {1} {2} } $ $\displaystyle a $는 유리수이고 $\displaystyle b \neq 0 $이므로 $ \displaystyle( a+b \sqrt {..

[더플러스수학학원] 2021년 1월 4주 차 예비 과고 1학년 수업 [내부링크]

어느새 1월 마지막 주 수업을 마쳤습니다. 2월 한 달 열심히 준비해서 3월부터 시작될 과학고 정규 수업 시 우리 학생들의 능력이 압도적으로 발휘되기를 바랍니다^^ 이번 1월 4주 TEST 결과, 이전 TEST와 같이 우리 친구들이 일반적인 연산(일반적이라고 하지만 어려웠죠?^^) 문제는 선생님도 놀랄 만큼 해결을 잘했답니다. 하지만 '증명하시오.'라는 문제에는 아직 적응이 덜 된 것인지 명쾌한 답안을 제시 못하는 경우가 있었어요. 물론 증명 문제는 많은 연습이 필요한 부분이기도 합니다. 우리 #더플러스수학학원 친구들이 좀 더 깊이 있는 학습을 통해 본인이 알고 있는 개념을 적재적소에 사용할 수 있기를 바랍니다. http://m.blog.naver.com/plusthemath/222224412590

[수학의 기초] [정적분의 정의] 2008학년도 연세대 모의논술 [더플러스수학] [내부링크]

2008학년도 연세대 모의논술 문제를 통해 정적분의 정의를 이용하여 곡선의 길이를 구하는 공식을 여러가지 방식으로 유도해 본다. youtu.be/QuhLNX7rK4A(구독과 좋아요) 2008학년도 연세대 모의논술 제시문을 읽고 물음에 답하시오. (40점) 함수 $\displaystyle f ( x) $의 도함수 $\displaystyle f ' ( x) $가 닫힌구간 $\displaystyle \left [ a,~b \right ] $에서 연속이고, $\displaystyle y=f ( x) $의 그래프가 [그림 1]과 같을 때, 다음 물음에 답하시오. [문제 1-1] 곡선 $\displaystyle y=f ( x) $ 위의 점 $\displaystyle ( a,~f ( a)) $부터 점 $\displa..

[수학의 기초] 카발리에리의 원리-정적분의 활용 [내부링크]

https://youtu.be/FnwgXBIVTOA(구독과 좋아요) 카발리에리의 원리(Cavalieri's Principle) 같은 높이를 갖는 입체들이 각 높이에서 동일한 넓이를 가진다면 똑같은 부피를 갖는다. 절단면의 넓이함수 $\displaystyle A ( x) $와 구간 $\displaystyle [a,~b] $가 두 입체들에 대해서 똑같은 조건이기 때문에 이 원리는 부피의 정의로부터 두 입체의 부피가 명백히 같다는 것을 말한다. 다음 문제를 읽고 물음에 답하여라.(카발리에리의 원리) (1) 곡선으로 둘러싸인 높이가 같은 두 개의 도형 $\displaystyle \mathrm {M,~N} $이 있다. 일정한 직선 $\displaystyle \mathrm {XY} $에 평행인 직선을 그렸을 때 $..

[울산과고기출] 에르미트 항등식과 그 증명방법들(2)[더플러스수학] [내부링크]

전편에서 가우스기호의 성질을 이용하여 에르미트 항등식을 증명한데 이어 2020/12/31 - [수학과 공부이야기] - [울산과고기출] 에르미트 항등식과 그 증명방법들(1)[더플러스수학] https://youtu.be/nAyjopeVhAU(구독과 좋아요!!) 이제는 주기함수임을 증명하고 이를 이용하여 에르미트 항등식을 증명하자. $$\displaystyle \left [ x \right ] + \left [ x+ \frac {1} {n} \right ] + \left [ x+ \frac {2} {n} \right ] + \cdots + \left [ x+ \frac {n-1} {n} \right ] = \left [ nx \right ] $$ 임을 보이시오. (단, $\displaystyle [x] $ 는..

[울산과고기출] 에르미트 항등식과 그 증명방법들(1)[더플러스수학] [내부링크]

$$\displaystyle \left [ x \right ] + \left [ x+ \frac {1} {n} \right ] + \left [ x+ \frac {2} {n} \right ] + \cdots + \left [ x+ \frac {n-1} {n} \right ] = \left [ nx \right ] $$ 임을 보이시오. (단, $\displaystyle [x] $ 는 $\displaystyle x $ 를 넘지 않는 가장 큰 정수이다.)[6점] [2017 과고1 1학기 중간 주11] 위의 항등식을 샤를 에르미트가 만든 항등식으로 그 이름을 따서 '에르미트 항등식'이라고 한다. https://youtu.be/nAyjopeVhAU(구독과 좋아요!!) 크게 두가지 방법으로 증명하는데 (i) 가우스..

[더플러스수학] \(\displaystyle x^n\) 미분 증명(실수까지) [내부링크]

\(\displaystyle \frac{d}{dx}x^n =n x^{n-1} ~ (n은~실수)\) https://youtu.be/G-uDnkUR6JQ (구독과 좋아요) 이것을 (i) \(n\)이 자연수일 때, (ii) \(n\)이 정수일 때, (iii) \(n\)이 유리수일 때, (iv) \(n\)이 실수일 때의 순으로 증명하자. 증명하는 과정에서 미분공식이 각각 필요하다. (i) \(n\)이 자연수 증명할 때, 필요한 미분공식은 곱미분법이다. 즉 함수 \(f(x),~g(x)\)가 각각 미분가능하면 \(f(x)\times g(x)\)도 미분가능하고, 도함수는 $$\left\{f(x)\times g(x)\right\}'=f'(x)g(x)+f(x)g'(x)$$ 또, 증명방법으로 수학적 귀납법이 필요하다. 이..

[더플러스수학] 과고합격자반 [내부링크]

합격을 축하합니다. 또 다른 한편 새로운 시작을 위해 준비해야 할 것이 많네요. 과학고1학년 1학기 중간고사가 과고생활을 결정합니다. 과고1기부터 학생을 가르쳐온 더플러스수학학원과 함께 준비합시다. 유투브에서 더플러스수학을 검색해보세요. 실력정석 연습문제 별해와 저의 모습을 보실 수 있습니다

[더플러스수학학원] 울산과고 1학년 2학기 기말대비-미적분 극한~평균값정리 서술형 문제 [울산과학고] [내부링크]

1. 함수 $\displaystyle f ( x) $가 임의의 실수 $\displaystyle x,~y $에 대하여 $$\displaystyle f ( x+y)=f ( x)+f ( y)+xy $$ 를 만족시킨다. 다음을 보여라. (1) 함수 $\displaystyle f ( x) $가 $\displaystyle x=0 $에서 연속이면 $\displaystyle f ( x) $는 모든 실수에서 연속임을 보여라. (2) $\displaystyle f ' ( 0)=1 $이라 할 때, $\displaystyle f ( x) $가 모든 실수에서 미분가능함을 보이고 도함수 $\displaystyle f ' ( x) $를 구하여라. 정답 및 풀이 더보기 (1) \(\displaystyle f ( x+y)=f ( x)..

과고2학년 2학기 프린트 [내부링크]

2차곡선 1. 점 $\displaystyle \mathrm { P} \left ( 2,~a \right ) $에서 원 $\displaystyle x ^ {2} +y ^ {2} =1 $에 두 개의 접선을 긋고, 두 접점의 중점을 $\displaystyle \mathrm { Q} $라 한다. 점 $\displaystyle \mathrm { P }\left ( 2,~a \right ) $가 직선 $\displaystyle x=2 $ 위의 $\displaystyle y > 0 $인 부분을 움직일 때, 점 $\displaystyle \mathrm { Q} $가 그리는 도형의 방정식을 구하여라. 풀이 더보기 풀이1) 위의 그림처럼 점들을 정의하자. 직선 $\displaystyle\mathrm{\overleftrig..

[수학의 기초] 지수함수는 모두 아래로 볼록, 로그함수는 위로볼록, 아래로볼록 모두 있는 이유? [내부링크]

과고 학생의 질문? 학교 과제로 지수함수는 모두 아래로 볼록인 함수이지만 왜 그 역함수는 위로 볼록인 함수와 아래로 볼록인 함수가 있는지 그 이유를 알고 싶어 질문함. 젠센 부등식을 이용하여 보이는 것이 힌트임. 먼저 젠센 부등식에 대하여 알아 보자. 함수 $\displaystyle f$가 아래로 볼록인 함수이면 다음이 성립한다. 정의역에 속하는 임의의 $\displaystyle a,~b $에 대하여 $$\displaystyle f \left(\frac{a+b}{2} \right) \leq \frac{f(a)+f(b)}{2}$$ 부등호가 반대로 되면 위로볼록함수이다. 이것에 대한 자세한 설명은 아래의 링크를 따라가 보세요. 2019/10/23 - [수학과 공부이야기] - [수학의 기초] 곡선의 볼록성 정..

[성균관대 수리논술] 2020학년도 성균관대 수시 논술 자연계 2교시 [내부링크]

https://youtu.be/Z7CtH4HR_So(구독과 좋아요!!) [ 수학 1 ] 다음 ~ 를 읽고 [수학 1 -ⅰ] ~ [수학 1 -ⅳ]를 문항별로 풀이와 함께 답하시오. 쌍곡선 $\displaystyle x ^ {2} -2y ^ {2} =-1 $의 윗부분을 $\displaystyle C _ {1} $으로 하고, 아랫부분을 $\displaystyle C _ {2} $라고 하자. 쌍곡선 밖의 한 점 $\displaystyle \rm P $에서 곡선 $\displaystyle C _ {1} $에 그은 접선의 접점을 $\displaystyle \rm Q $, 점 $\displaystyle \rm P $에서 곡선 $\displaystyle C _ {2} $에 그은 접선의 접점을 $\displaystyle..

[한양대수리논술] 2020학년도 한양대 자연계열 논술(오후1)[더플러스수학] [내부링크]

https://youtu.be/z5N_JG5fHWw(구독과 좋아요!!) [문제 1] 다음 물음에 답하시오. (50점) 1. 주사위를 $\displaystyle n $번 던질 때 $ 3 $의 눈이 나오는 횟수가 $\displaystyle 2 $의 배수일 확률을 구하시오. 2. 주사위를 $\displaystyle n $번(단, $\displaystyle n \geq 3 $) 던질 때 $\displaystyle 3 $의 눈이 나오는 횟수가 $\displaystyle k $이면 $\displaystyle 100k \left ( k-1 \right ) \left ( k-2 \right ) $원의 상금을 지급한다고 하자. 상금의 기댓값을 구하시오. 3. 주사위를 $\displaystyle n $번 던질 때 $\di..

[연세대논술]2020학년도 연세대학교 수시모집 논술시험-의학계열(오후) [내부링크]

https://youtu.be/6tirxblk4rs(구독과 좋아요!!) [문제 1] 합성함수가 정의될 수 있는 범위에서 함수 $\displaystyle f ( x) $에 대한 합성함수를 다음과 같이 나타내자. $\displaystyle \left ( f \circ f \right ) \left ( x \right ) =f ^ {} \left ( x \right ) $, $\displaystyle \left ( f \circ f \circ f \right ) \left ( x \right ) =f ^ {} \left ( x \right ) $, $\displaystyle \cdots $, $\displaystyle ( \underbrace {f \circ f \circ \cdots \circ f}_{n개}..

[연세대논술]2020학년도 연세대학교 수시모집 논술시험 오전 [내부링크]

https://youtu.be/RF95YP4OYM0(구독과 좋아요!!) [문제 1] 그림과 같이 $\displaystyle \mathrm { \overline {AC}} =1 $, $\displaystyle \overline {\mathrm { BC }} =a $ ($\displaystyle a>0 $)이고 $\displaystyle \mathrm { \angle BCA}= \frac {\pi } {2} $인 삼각형 $\displaystyle \mathrm { ABC }$가 있다. 자연수 $\displaystyle n $에 대하여 선분 $\displaystyle \mathrm { CA} $를 $\displaystyle n $등분한 각 분점을 점 $\displaystyle \mathrm { C }$에서 가..

[더플러스수학] 과고2학년 2학기 고급수학 기출문제 [내부링크]

1. 다음을 계산하시오. (5점) (1) $\displaystyle \left ( \matrix {1 & 2\\3 & 4} \right) + \left ( \matrix {3 & 4\\5 & 6} \right) $ (1점) (2) $\displaystyle \left ( \matrix {1 & 2\\0 & 1} \right) \left ( \matrix {1 & 2\\1 & 3} \right) $ (2점) (3) $\displaystyle \left ( \matrix {1 & 3 & 0\\2 & -1 & 2\\0 & 1 & 3} \right) ^ {2} $ (2점) 정답 및 해설 더보기 (1) $\displaystyle \left ( \matrix {4 & 6\\8 & 10} \right) $ (2) $..

[수학의 기초] 생성함수에 대하여 (1) [더플러스수학] [내부링크]

수학1의 수열에서 수학적 귀납법 단원 중 수열의 점화식이 나오는 문제를 풀 때, 점화식 마다 풀이 방법을 외워야 해서 학생들이 많이 힘들어 합니다. 물론 이 과정은 교과에 빠졌지만 상위권학생은 풀이 과정을 이해하고 외워야 합니다. 이 문제를 다른 관점에서 해결하고자 "생성함수"(generating function)에 대해 가볍게 알아보고 이를 이용하여 점화식을 한번 풀어 보겠습니다. 일반적으로 수열 $\displaystyle \left\{ a_n \right\}~(n=0,~1,~2,~\cdots)$에 대하여 $$\displaystyle g(x)=a_0 +a_1 x+a_2 x^2 + \cdots+a_n x^n +\cdots =\sum_{\textcolor{red}{n=0}}^{\infty}a_n x^n$$..

4회-2020학년도 울산과고1학년 2학기 중간대비 [내부링크]

사잇값 정리와 넓이의 3등분 좌표평면 위에 있는 도형을 넓이가 똑같은 두 개의 도형으로 나누는 $\displaystyle y $축과 평행한 직선이 있는가? 또 넓이가 똑같은 3개의 도형으로 나누는 $\displaystyle y $축과 평행한 직선들이 있는가? https://youtu.be/1se7M7epQYI(구독과 좋아요)

[수학의 기초] 사잇값정리의 응용-넓이의 3등분선의 존재 [내부링크]

사잇값 정리와 넓이의 3등분 좌표평면 위에 있는 도형을 넓이가 똑같은 두 개의 도형으로 나누는 $\displaystyle y $축과 평행한 직선이 있는가? 또 넓이가 똑같은 3개의 도형으로 나누는 $\displaystyle y $축과 평행한 직선들이 있는가? https://youtu.be/1se7M7epQYI(구독과 좋아요)

[더플러스수학] 2013학년도 서울시립대 수리논술 (A) [내부링크]

2013학년도 수시모집 일반전형 논술고사 문제지 (자연계열A) 2012. 11. 20(화) 13:30 ~ 15:30 ※ 풀이과정을 반드시 기술할 것. 기술의 형식과 내용은 평가의 중요한 요소임. https://youtu.be/_-jlS67zTfM (구독과 좋아요를) [문제 1] 그림과 같이 좌표평면에 움직이는 점 A가 있다. 시각 $\displaystyle t \, \left( 0 < t < \frac {\pi } {4} \right)$에서 점 A의 좌표는 $\displaystyle ( 2\sin t,~0) $이다. 제 2사분면 위의 점 B는 직선 $\displaystyle y=-x $위에 있고, $\displaystyle \overline {\mathrm{AB}} = \sqrt {2} $이다. 선분 $..

[과학고내신대비] 2020학년도 1학년 2학기 중간대비 3 [내부링크]

기저변환행렬에 대한 정확한 이해!!! 기저 $\displaystyle \vec p , ~\vec q $의 집합을 $\displaystyle A$, 기저 $\displaystyle \vec {p'} , ~\vec {q'} $의 집합을 $\displaystyle B$라고 할 때, 기저 $\displaystyle A$를 $\displaystyle B$로 바꾸는 행렬 즉 기저변환행렬 $\displaystyle \left[ P \right]_{A}^{B}$은 다음과 같다. 기저 $\displaystyle A$에서의 좌표 $\displaystyle (x,~y)$를 기저 $\displaystyle B$에서의 좌표 $\displaystyle (x',~y') $로 바꾸는 행렬을 의미한다. 앞 글의 독자님의 질문을 예로 ..

[수학의 기초] 기저변환행렬 (2) [더플러스수학] [내부링크]

기저변환행렬에 대한 정확한 이해!!! 기저 $\displaystyle \vec p , ~\vec q $의 집합을 $\displaystyle A$, 기저 $\displaystyle \vec {p'} , ~\vec {q'} $의 집합을 $\displaystyle B$라고 할 때, 기저 $\displaystyle A$를 $\displaystyle B$로 바꾸는 행렬 즉 기저변환행렬 $\displaystyle \left[ P \right]_{A}^{B}$은 다음과 같다. 기저 $\displaystyle A$에서의 좌표 $\displaystyle (x,~y)$를 기저 $\displaystyle B$에서의 좌표 $\displaystyle (x',~y') $로 바꾸는 행렬을 의미한다. 앞 글의 독자님의 질문을 예로 ..

[평가원기출]2021학년도 9월 평가원 가형 21번-킬러문항[더플러스수학] [내부링크]

닫힌구간 $\displaystyle [-2 \pi ,~2 \pi ] $에서 정의된 두 함수 $$\displaystyle f ( x)=\sin kx+2 ,~ g ( x)=3\cos 12x $$ 에 대하여 다음 조건을 만족시키는 자연수 $\displaystyle k $의 개수는? [$\displaystyle 4 $점] 실수 $\displaystyle a $가 두 곡선 $\displaystyle y=f ( x) $, $\displaystyle y=g ( x) $의 교점의 $\displaystyle y $좌표이면 $$\displaystyle \left\{ \,x\,|\,f ( x)=a\, \right\} \subset \left\{ \,x\,|\, g ( x)=a\, \right\} $$이다. ① $\displ..

[수리논술] 사이클로이드의 등시성과 최단강하곡선 [내부링크]

[제시문] [가] 사이클로이드(cycloid)란 한 원이 일직선 위를 굴러갈 때, 이 원의 원둘레 위의 한 점이 그리는 자취이다. 예를 들어 자전거 바퀴의 한 점에 전구를 설치하고 자전거를 일정한 속력으로 움직일 때 전구가 그리는 자취는 사이클로이드가 된다. 사이클로이드는 바퀴라는 뜻의 그리스어로 갈릴레이가 붙였다고 한다. 사이클로이드는 매우 특이한 물리적 성질을 지니고 있어 흥미로운 대상이다. [나]1583년 성당에서 예배를 드리던 갈릴레이는 ‘천장에 매달린 진자의 주기가 진폭에 상관없이 일정하다’는 ‘진자의 등시성(等時性)’을 발견했다. 하지만 정확하게 이야기한다면 등시성은 진자의 진폭이 매우 작을 경우에만 성립한다. 일반적으로 진폭이 커지면 주기도 증가하기 때문에 진자의 등시성은 성립하지 않는데 정..

[더플러스수학] 2회-2020학년도 울산과고1학년 2학기 중간대비 [내부링크]

1. $\displaystyle 0 $이 아닌 세 실수 $\displaystyle a,~b,~c $와 양수 $\displaystyle x $가 다음 조건을 만족시킨다. $$\displaystyle \frac {1} {ab} + \frac {1} {bc} + \frac {1} {ca} = \frac {3} {abc} $$ $$\displaystyle a\log _ {15} x=b\log _ {10} x=c\log _ {6} x=1 $$ 이 때 $\displaystyle \sqrt {x} $의 값을 구하시오. [$\displaystyle 8.0 $점] https://youtu.be/uhW-3hO2_SU *해설: [정답] $\displaystyle {\sqrt {x} = \root {3} \of {30} } ..

[더플러스수학] [1회] 2020학년도 과고1학년 2학기 중간대비 [내부링크]

답안지에 학년, 계열, 반, 번호, 이름을 표기하고 해당란에 바르게 표한 후 답안을 작성하시오. 답안지에는 정답 외의 예비마킹을 하지 않도록 주의 바랍니다. 1. $$\displaystyle 6\sin ^ {2} \theta -\sin \theta \cos \theta -2\cos ^ {2} \theta =0 $$일 때, $\displaystyle \sin \theta $의 값을 구하시오. (단, $\displaystyle \frac {\pi } {2} < \theta < \pi $) [4.3점] [2008 과고1 1학기 기말 11 주관식변형] https://youtu.be/iKJnXK9UAsM(좋아요와 구독을...) 정답 및 풀이를 보려면 아래를 클릭하세요. 더보기 *해설: $\displaysty..

[더플러스수학] 2020학년도 카이스트 면접문제-일반전형, 학교장추천 [내부링크]

2020학년도 카이스트 심층면접-일반전형 https://youtu.be/OTYTHagYhzY(구독과 좋아요!!) [문제1] 자연수 $\displaystyle n $에 대하여 두 함수 $\displaystyle f ( x)=\sin ^ {3} ( nx) $와 $\displaystyle g ( x)=\cos ^ {3} ( nx) $를 생각하자. (5점) (1) 구간 $\displaystyle [0,~2 \pi ] $에서 두 평면 곡선 $\displaystyle y=f ( x) $와 $\displaystyle y=g ( x) $가 만나는 교점의 $\displaystyle x $좌표의 최솟값 $\displaystyle a $와 최댓값 $\displaystyle b $를 구하시오. [$\displaystyle 1..

단원 2. 삼각함수 - 문제 (1) [내부링크]

안녕하세요, 더(THE) 플러스수학학원입니다.권도형원장님의 이 개설됨을 알려드립니다. 과고 1학년 특강반- 개강 : 2020년 8월 17일 월요일- 수업 : 오전 9:00~12:0 (3시간)- 교재 : 실력정석 수학1, 미적분 -수업 동영상 홈페이지(더플러스수학 홈페이지)에 게시 한달 간 볼 수 있슴 - 혜택 : 1) 페이스북의 광고를 통해 더플러스수학 유투브채널 구독, 블로그 비밀댓글로 신규수강신청한 것 확인시 - 문화상품권 1매 학생에게 증정 2) 기존 학생도 더플러스수학 유투브채널 구독, 블로그 비밀댓글로 수강신청한 것 확인시 - 문화상품권 1매 학생에게 증정 3) 기존 학생이 친구 추천, 학생 수강신청시 기존학생과 수강신청 학생에게 문화상품권 각각 1매 증정 * 최대 2매 받을 수 있음(기존학..

단원 2. 삼각함수 - 예제 (1) [내부링크]

안녕하세요, 더(THE) 플러스수학학원입니다.권도형원장님의 이 개설됨을 알려드립니다. 과고 1학년 특강반- 개강 : 2020년 8월 17일 월요일- 수업 : 오전 9:00~12:0 (3시간)- 교재 : 실력정석 수학1, 미적분 -수업 동영상 홈페이지(더플러스수학 홈페이지)에 게시 한달 간 볼 수 있슴 - 혜택 : 1) 페이스북의 광고를 통해 더플러스수학 유투브채널 구독, 블로그 비밀댓글로 신규수강신청한 것 확인시 - 문화상품권 1매 학생에게 증정 2) 기존 학생도 더플러스수학 유투브채널 구독, 블로그 비밀댓글로 수강신청한 것 확인시 - 문화상품권 1매 학생에게 증정 3) 기존 학생이 친구 추천, 학생 수강신청시 기존학생과 수강신청 학생에게 문화상품권 각각 1매 증정 * 최대 2매 받을 수 있음(기존학..

단원 1. 수열의 극한 -문제 (1) [내부링크]

안녕하세요, 더(THE) 플러스수학학원입니다.권도형원장님의 이 개설됨을 알려드립니다. 과고 1학년 특강반- 개강 : 2020년 8월 17일 월요일- 수업 : 오전 9:00~12:0 (3시간)- 교재 : 실력정석 수학1, 미적분 -수업 동영상 홈페이지(더플러스수학 홈페이지)에 게시 한달 간 볼 수 있슴 - 혜택 : 1) 페이스북의 광고를 통해 더플러스수학 유투브채널 구독, 블로그 비밀댓글로 신규수강신청한 것 확인시 - 문화상품권 1매 학생에게 증정 2) 기존 학생도 더플러스수학 유투브채널 구독, 블로그 비밀댓글로 수강신청한 것 확인시 - 문화상품권 1매 학생에게 증정 3) 기존 학생이 친구 추천, 학생 수강신청시 기존학생과 수강신청 학생에게 문화상품권 각각 1매 증정 * 최대 2매 받을 수 있음(기존학..

단원 1. 수열의 극한 -예제 (1) [내부링크]

안녕하세요, 더(THE) 플러스수학학원입니다.권도형원장님의 이 개설됨을 알려드립니다. 과고 1학년 특강반- 개강 : 2020년 8월 17일 월요일- 수업 : 오전 9:00~12:0 (3시간)- 교재 : 실력정석 수학1, 미적분 -수업 동영상 홈페이지(더플러스수학 홈페이지)에 게시 한달 간 볼 수 있슴 - 혜택 : 1) 페이스북의 광고를 통해 더플러스수학 유투브채널 구독, 블로그 비밀댓글로 신규수강신청한 것 확인시 - 문화상품권 1매 학생에게 증정 2) 기존 학생도 더플러스수학 유투브채널 구독, 블로그 비밀댓글로 수강신청한 것 확인시 - 문화상품권 1매 학생에게 증정 3) 기존 학생이 친구 추천, 학생 수강신청시 기존학생과 수강신청 학생에게 문화상품권 각각 1매 증정 * 최대 2매 받을 수 있음(기존학..

[더플러스수학 여름방학특강] 과고1학년 단기 방학 특강반 모집 [내부링크]

안녕하세요, 더(THE) 플러스수학학원입니다.권도형원장님의 이 개설됨을 알려드립니다. 과고 1학년 특강반- 개강 : 2020년 8월 17일 월요일- 수업 : 오전 9:00~12:0 (3시간)- 교재 : 실력정석 수학1, 미적분 -수업 동영상 홈페이지(더플러스수학 홈페이지)에 게시 한달 간 볼 수 있슴 - 혜택 : 1) 페이스북의 광고를 통해 더플러스수학 유투브채널 구독, 블로그 비밀댓글로 신규수강신청한 것 확인시 - 문화상품권 1매 학생에게 증정 2) 기존 학생도 더플러스수학 유투브채널 구독, 블로그 비밀댓글로 수강신청한 것 확인시 - 문화상품권 1매 학생에게 증정 3) 기존 학생이 친구 추천, 학생 수강신청시 기존학생과 수강신청 학생에게 문화상품권 각각 1매 증정 * 최대 2매 받을 수 있음(기존학..

과고3학년 수행평가 문제 [내부링크]

세트1 1. 세 꼭짓점이 포물선 $\displaystyle y=x ^ {2} $ 위에 있는 정삼각형을 한 개 찾아 그 넓이를 구하시오. 풀이) 원점을 지나고 기울기가 $\pm \tan 60^{\circ}$인 직선과 $y=x^2$과 만나는 점이 정삼각형을 이룬다. 2. 미분가능한 함수 $\displaystyle f ( x),~g ( x) $는 다음 조건을 만족한다. 적분값 $\displaystyle \int _ {0} ^ {\pi } { \frac {3f \left ( x \right ) g ' \left ( x \right ) -f ' \left ( x \right ) g \left ( x \right )} {\left\{ f \left ( x \right ) \right\} ^ {2} g \left (..

[더플러스수학] [3회] 2020과고 1학년 1학기 기말고사 대비 [내부링크]

답안지에 학년, 계열, 반, 번호, 이름과 과목코드번호를 표기하고 해당란에 바르게 표한 후 답안을 작성하시오. 1. $\displaystyle \mathrm { A}= \left\{\, x \,|\,x>0\, \right\} $일 때, $\displaystyle \mathrm { A} $에서 $\displaystyle \mathrm { A} $로의 함수 $\displaystyle f,~g $를 $$\displaystyle f ( x)=x ^ {2} -x,~g ( x)= \sqrt {2x-1} $$라 한다. 이 때, $\displaystyle \left ( f \circ ( g \circ f) ^ {-1} \circ f ^ {-1} ) ( 6) \right . $의 값을 구하여라.[4.7점] [2008..

[옥동수학학원][수학의 기초] 테일러 정리 증명-평균값 정리의 일반화[더플러스수학] [내부링크]

테일러 정리(Taylor Theorem)(1) 함수 $f(x)$가 닫힌 구간 $[a,~b]$에서 연속이고 열린 구간 $(a,~b)$에서 $n$번 미분가능하면 $$\displaystyle \begin{align} f(b) &= \sum_{k=0}^{n-1} \frac{(b-a)^k}{k!} f^{(k)}(a) + \frac{(b-a)^n}{n!} f^{(n)}(c)\\&= f(a)+ (b-a)f'(a)+ \cdots+ \frac{(b-a)^{n-1}}{(n-1)!}f^{(n-1)}(a)+\frac{(b-a)^n}{n!}f^{(n)}(c) \end{align}$$ 을 만족하는 $c \in(a,~b)$가 적어도 하나 존재한다.아래의 두번째 방법으로 증명하기 위해서는 함수 $f$의 $n$계 도함수인 $f^{(n..

[더플러스수학] 2020년 과고2학년 1학기 기말 프린트 풀이 [내부링크]

심화수학 II 보충자료-경우의 수, 확률파트 정리 1. 다음 경우의 수를 구하시오. [$\displaystyle \mathbb2017 $학년도 서울대 일반전형 면접 및 구술고사] (1) $\displaystyle 9 $개의 좌석이 일렬로 배치되어 있는 롤러코스터에 3명의 학생을 다음의 조건 (*)를 만족하도록 태우는 경우의 수를 구하시오. (*) 연이은 두 좌석에 학생이 앉은 경우에는 앞좌석에 앉은 학생의 키가 더 작다. 단, $\displaystyle 3 $명이 모두 탑승하며, 어느 두 명의 학생도 키가 같지 않다고 가정한다. (2) $\displaystyle m $이 $\displaystyle n $보다 큰 자연수일 때, $\displaystyle m $개의 좌석이 일렬로 배치되어 있는 롤러코스터에 ..

2020년 과고2학년 기말 프린트[기하 증명] [내부링크]

포물선 증명은 대수적인 식을 이용해도, 논증기하를 이용해도 좋습니다 [정리1] 포물선 위의 점 $\displaystyle \mathrm { P} $에서의 접선은 $\displaystyle \mathrm { \angle CPF }$를 이등분한다. ($\displaystyle \mathrm { F} $는 초점, $\displaystyle \mathrm { C} $는 점 $\displaystyle \mathrm { P }$에서 준선 $\displaystyle l $에 내린 수선의 발) [정리2] 원점 $\displaystyle \mathrm { O }$를 꼭짓점으로 하고 초점을 $\displaystyle \mathrm { F} $, 준선을 $\displaystyle l $인 포물선이 있다. 이 포물선 위의 점..

[질문] $x^2+y^2+z^2 \geq k(xy+yz+zx)$를 만족하는 $k$의 범위?[더플러스수학] [내부링크]

[질문] $x^2+y^2+z^2 \geq k(xy+yz+zx)$를 만족하는 $k$의 범위를 구하자. 2019년 울산과고 1학년 1학기 기말고사 2번 문제입니다. https://youtu.be/OU20_sl-YbE

[수학의 기초] $x^2+y^2+z^2 \geq xy+yz+zx$ 여러가지 증명 [더플러스수학] [내부링크]

고등학교 수학에서, 특히 고1과정에서 굉장히 중요한 공식이 $$\textcolor{red}{x^2+y^2+z^2 \ge xy+yz+zx}$$이다. 이것을 다양하게 증명해 보자. 1) 일반적인 부등식의 증명 $$\begin{align} x^2+y^2+z^2 -(xy+yz+zx) &= \frac{1}{2} \left\{ 2x^2 +2y^2 +2z^2 -2xy-2yz-2zx\right\} \\&= \frac{1}{2} \left\{ \left(x^2 -2xy+y^2 \right) +\left(y^2 -2yz+z^2 \right)+\left(z^2 -2zx+x^2 \right)\right\}\\&= \frac{1}{2} \left\{ (x-y)^2 +(y-z)^2 +(z-x)^2\right\} \geq 0 \e..

(2회) 2019년 울산과고 1학년 1학기 기말고사 수학 [내부링크]

$n$개의 동전을 가지고 있는 $A$와 무한한 동전을 가지고 있는 $B$가 서로 게임을 한다. 한 게임마다 각자 $1$개의 동전을 걸고 게임에서 이긴 사람이 모두 가진다. $A$가 한 게임에서 이길 확률은 $p$이며 $A$와 $B$가 같은 게임을 무한히 반복한다고 하자. 이 때, $A$가 모든 동전을 잃을 확률을 아래와 같음을 증명하시오. $\displaystyle p \leq \frac{1}{2}$이면 $1$ $\displaystyle p > \frac{1}{2}$이면 $\displaystyle \left( \frac{1-p}{p}\right)^n$ $P_n$을 $A$가 현재 $n$개의 동전을 가지고 게임에 참가하여 모든 동전을 잃을 확률이라 하자. 또, $B$가 무한개의 동전을 가지고 있는 것으로 하..

(1회) 2020학년도 울산과고 1학년 1학기 기말대비 1 [내부링크]

$n$개의 동전을 가지고 있는 $A$와 무한한 동전을 가지고 있는 $B$가 서로 게임을 한다. 한 게임마다 각자 $1$개의 동전을 걸고 게임에서 이긴 사람이 모두 가진다. $A$가 한 게임에서 이길 확률은 $p$이며 $A$와 $B$가 같은 게임을 무한히 반복한다고 하자. 이 때, $A$가 모든 동전을 잃을 확률을 아래와 같음을 증명하시오. $\displaystyle p \leq \frac{1}{2}$이면 $1$ $\displaystyle p > \frac{1}{2}$이면 $\displaystyle \left( \frac{1-p}{p}\right)^n$ $P_n$을 $A$가 현재 $n$개의 동전을 가지고 게임에 참가하여 모든 동전을 잃을 확률이라 하자. 또, $B$가 무한개의 동전을 가지고 있는 것으로 하..

[더플러스수학]2019년 울산과고2 심화수학 기출 20번 -gambler's ruin [내부링크]

$n$개의 동전을 가지고 있는 $A$와 무한한 동전을 가지고 있는 $B$가 서로 게임을 한다. 한 게임마다 각자 $1$개의 동전을 걸고 게임에서 이긴 사람이 모두 가진다. $A$가 한 게임에서 이길 확률은 $p$이며 $A$와 $B$가 같은 게임을 무한히 반복한다고 하자. 이 때, $A$가 모든 동전을 잃을 확률을 아래와 같음을 증명하시오. $\displaystyle p \leq \frac{1}{2}$이면 $1$ $\displaystyle p > \frac{1}{2}$이면 $\displaystyle \left( \frac{1-p}{p}\right)^n$ $P_n$을 $A$가 현재 $n$개의 동전을 가지고 게임에 참가하여 모든 동전을 잃을 확률이라 하자. 또, $B$가 무한개의 동전을 가지고 있는 것으로 하..

[수학의 기초] 확률과 통계 경우의 수 구하는 특이한 방법들('menage problem'을 해결하기 위해)-1 [내부링크]

1. $1$부터 $n$까지의 서로 다른 정수 중에서 이웃하지 않는 서로 다른 $k$를 뽑는 방법의 수는 $$ \textcolor{red}{\displaystyle {}_{n-k+1} \mathrm{C}_{k}}$$ (증명) 서로 다른 $k$개의 수를 $x_1 ,~x_2 ,~\cdots,~x_k$라 두면 이 수들이 서로 이웃해서는 안되므로 $i=1,2,\cdots,{k-1}$인 $i$에 대하여 $x_i$ 와 $x_{i+1}$ 사이에는 적어도 $1$개의 숫자가 들어가야 한다. 그런데 $x_1$ 앞과 $x_k$ 뒤에는 $0$개 이상 들어 가도 된다. $n$개 중 $k$개의 수가 아닌 것의 개수는 $n-k$개이다. 이 $n-k$개를 모두 같은 빈 $\boxed{~~}$ 로 표시하자. 그러면 위의 그림에서 볼 수..

2020년 과고2 확률과 통계 - 비둘기집 원리 프린트 [내부링크]

1. 한 변의 길이가 $\displaystyle 2 $인 정사각형에 $\displaystyle 5 $개의 점이 있으면, 두 점 사이의 거리가 $\displaystyle \sqrt {2} $이하인 두 점이 반드시 존재함을 보여라. https://youtu.be/iW0qIHKZMV8(구독과 좋아요) 오른쪽의 그림처럼 한 변이 $\displaystyle 2 $인 정사각형을 $\displaystyle 1 \times 1 $인 한 변의 길이가 정사각형 넷로 나누자. 이 $\displaystyle 4 $개의 정사각형에 $\displaystyle 5 $개의 점을 넣으면 정사각형 중에 두 개의 점이상을 가진 정사각형이 존재한다. 이 정사각형의 내부의 임의의 두점 사이의 거리는 $\displaystyle \sqrt {2..

[질문과 답] $z^{10} =\bar{z}$인 실수가 아닌 복소수에 대하여.... [내부링크]

(질문) 실수가 아닌 복소수 $\displaystyle z $에 대하여 $\displaystyle z ^ {10} - \overline {z} =0 $일 때, $\displaystyle 1+z+z ^ {2} + \cdots +z ^ {21} +z ^ {22} $의 값을 구하시오. https://youtu.be/M3PYqdiaf0Q 정답 및 풀이를 보려면 아래를 클릭하세요. 더보기 [정답] $\displaystyle 1 $ [풀이] $\displaystyle z ^ {10} = \overline {z} $에서 양변에 $\displaystyle \overline {z} ^ {10} =z $ 를 곱하면 $\displaystyle \left ( z \overline {z} \right ) ^ {10} =z \o..

[질문과 답] $(x+2)^2$으로 나눈 나머지, $(x+3)^2$으로 나눈 나머지.... [내부링크]

[질문] 최고차항의 계수가 $\displaystyle 2 $인 삼차다항식 $\displaystyle f ( x) $를 $\displaystyle \left ( x+2 \right ) ^ {2} $으로 나누었을 때의 나머지가 $\displaystyle 3x-2 $이고, $\displaystyle x+1 $로 나누었을 때의 나머지는 $\displaystyle 4 $이다. $\displaystyle f ( x) $를 $\displaystyle \left ( x+2 \right ) ^ {3} $으로 나누었을 때의 나머지를 $\displaystyle R ( x) $라고 했을 때, $\displaystyle R ( 2) $의 값을 구하시오. https://youtu.be/mKH3GrmPEbw [답] 상수 $\dis..

과고1학년수업 중 올리지 못한 동영상 [내부링크]

삼차의 다항식 $\displaystyle f ( x) $가 다음 조건을 만족시킨다. $\displaystyle f ( x) $를 $\displaystyle \left ( x+1 \right ) ^ {3} $으로 나눈 나머지를 $\displaystyle R ( x) $라 하자. $\displaystyle R ( 0)=R ( 1) $일 때, $\displaystyle R ( 2) $의 값을 구하시오. $\displaystyle f ( -1)=3 $ $\displaystyle f ( x) $를 $\displaystyle \left ( x+1 \right ) ^ {2} $으로 나눈 몫과 나머지는 같다. (풀이) 다음 링크를 보세요. https://youtu.be/sP0Gd44-RTo 다항식을 표현하는 방법에 대..

[질문과 답]삼차식을 $(x+1)^2$으로 나눈 몫과 나머지 같다. [내부링크]

삼차의 다항식 $\displaystyle f ( x) $가 다음 조건을 만족시킨다. $\displaystyle f ( x) $를 $\displaystyle \left ( x+1 \right ) ^ {3} $으로 나눈 나머지를 $\displaystyle R ( x) $라 하자. $\displaystyle R ( 0)=R ( 1) $일 때, $\displaystyle R ( 2) $의 값을 구하시오. $\displaystyle f ( -1)=3 $ $\displaystyle f ( x) $를 $\displaystyle \left ( x+1 \right ) ^ {2} $으로 나눈 몫과 나머지는 같다. (풀이) 다음 링크를 보세요. https://youtu.be/sP0Gd44-RTo 다항식을 표현하는 방법에 대..

[수학의 기초] 아폴로니우스의 원으로 가는 길(1)-삼각형에서 각이등분선의 성질 증명 [내부링크]

아폴로니우스의 원 이에 대한 증명을 위해 먼저 각 이등분선의 성질을 아래의 3가지의 관점에서 증명해 보겠다. 중학교 2학년 2학기에 나오는 삼각형의 각 이등분선의 성질을 중학교 과정의 (1) 유클리드 기하의 관점에서 (2) 고1의 해석기하의 관점에서 (3) 기하에서 벡터의 관점에서 증명하고자 한다. 먼저 증명하고자 하는 것을 명제로 표현하면 다음과 같다. 삼각형 $\displaystyle \mathrm { ABC} $에서 $\displaystyle \angle A $의 내(외)각의 이등분선이 변 $\displaystyle \mathrm { \overline {BC} }$와 만나는 점을 $\displaystyle \mathrm { P }$라 하면 $$\displaystyle \mathrm { \overli..

인문계고3 수업 [내부링크]

1. $\displaystyle x $에 대한 이차식 $\displaystyle x ^ {2} +ax+bc $와 $\displaystyle x ^ {2} +bx+ac $의 최대공약수가 $\displaystyle x $에 대한 일차식일 때, 다음 물음에 답하시오. (단, $\displaystyle abc \neq 0 $이다.) [총 $\displaystyle 8 $점] (1) $\displaystyle x $에 관한 이차식 $\displaystyle x ^ {2} +ax+bc $와 $\displaystyle x ^ {2} +bx+ac $의 최소공배수를 구하고, 그 과정을 서술하시오. [$\displaystyle 4 $점] (2) $\displaystyle \frac {\left ( a ^ {3} +b ^ ..

2018년 경기과학고 1-1학기 중간고사 [내부링크]

1. $\displaystyle x $에 대한 이차식 $\displaystyle x ^ {2} +ax+bc $와 $\displaystyle x ^ {2} +bx+ac $의 최대공약수가 $\displaystyle x $에 대한 일차식일 때, 다음 물음에 답하시오. (단, $\displaystyle abc \neq 0 $이다.) [총 $\displaystyle 8 $점] (1) $\displaystyle x $에 관한 이차식 $\displaystyle x ^ {2} +ax+bc $와 $\displaystyle x ^ {2} +bx+ac $의 최소공배수를 구하고, 그 과정을 서술하시오. [$\displaystyle 4 $점] (2) $\displaystyle \frac {\left ( a ^ {3} +b ^ ..

[킬러문항][더플러스수학] 2020년 3월 교육청 30번(4월24일시행) [내부링크]

30. 최고차항의 계수가 $4$ 인 삼차함수 $f(x)$ 와 실수 $t$ 에 대하여 함수 $g(x)$ 를 $$g(x)=\int_t^x f(s)ds$$ 라 하자. 상수 $a$에 대하여 두 함수 $f(x)$ 와 $g(x)$ 가 다음 조건을 만족시킨다. (가) $f'(a)=0$ (나) 함수 $\left| g(x)-g(a) \right|$ 가 미분가능하지 않은 $x$ 의 개수는 $1$ 이다. 실수 $t$ 에 대하여 $g(a)$ 의 값을 $h(t)$ 라 할 때, $h(3)=0$ 이고 함수 $h(t)$ 는 $t=2$ 에서 최댓값 $27$ 을 가진다. $f(5)$의 값을 구하시오. [$4$점] https://youtu.be/bqNYjfLhEOY

2019학년도 상산고 1학기 중간고사 및 풀이 [내부링크]

1. $\displaystyle x ^ {3} +y ^ {3} +z ^ {3} +xy \left ( x+y \right ) +yz \left ( y+z \right ) +zx \left ( z+x \right ) $을 두 다항식의 곱으로 인수분해하시오. [3.2점] 정답 및 풀이를 보려면 아래를 클릭하세요. 더보기 $\displaystyle \left ( x ^ {2} +y ^ {2} +z ^ {2} \right ) \left ( x+y+z \right ) $ https://youtu.be/pbDXIQFyqgQ 2. 네 다항식 $\displaystyle A=x ^ {2} -2xy+3y ^ {2} ,B=y ^ {2} +3xy-2x ^ {2} $, $\displaystyle C=xy-4y ^ {2} +3x ..

[삼사기출] 2017학년도 나형 14번-일대일대응 [내부링크]

두 집합 $\displaystyle A= \left\{ 1,~2,~3,~4 \right\} $, $\displaystyle B= \left\{ 2,~3,~4,~5 \right\} $에 대하여 두 함수 $\displaystyle f~:~A \rightarrow B $, $\displaystyle g~:~B \rightarrow A $가 다음 조건을 만족시킨다. (가) $\displaystyle f ( 3)=5 $, $\displaystyle g ( 2)=3 $ (나) 어떤 $\displaystyle x \in B $에 대하여 $\displaystyle g ( x)=x $이다. (다) 모든 $\displaystyle x \in A $에 대하여 $\displaystyle ( f \circ g \circ f) ..

[더플러스수학]2019학년도 서울대 심층면접 1 [서울대구술면접] [내부링크]

문제 1. 1-1. 좌표평면 위의 두 점 $\displaystyle \mathrm { A} $와 $\displaystyle \mathrm { B }$의 좌표는 각각 $\displaystyle \left ( -10,~2 \right ) $와 $\displaystyle \left ( 10,~2 \right ) $이며, 점 $\displaystyle \mathrm { C} $와 점 $\displaystyle \mathrm { D }$는 $\displaystyle x $축 위를 움직이고 있다. $\displaystyle \mathrm { \overline {AC} + \overline {CD} + \overline {DB} }$가 최소가 되는 점 $\displaystyle \mathrm { C} $와 점 $\d..

[수학의 팁] 3차함수의 극대극소의 차 [더플러스수학] [내부링크]

수학2 극대극소편에서 3차함수 극대극소문제를 풀 때 알고 있으면 좋은 팁을 하나 소개하고 그것을 증명하도록 하겠다. 3차함수 $f(x)=ax^3 +bx^2 +cx +d$가 극대, 극소를 $x=\alpha,~x=\beta$에서 갖는다고 하면 $$\textcolor{red} {\mathrm{(극댓값)과~ (극솟값)의 ~차} =\frac { \left| a \right| }{2} \left| \beta-\alpha \right|^3 }$$ 증명을 보려면 아래를 클릭하세요. 더보기 (증명) 3차함수 $\displaystyle f(x)=ax^3 +bx^2 +cx +d$가 극대, 극소를 $\displaystyle x=\alpha,~x=\beta$에서 가지므로 $\displaystyle f'(x)=0$의 근은 $\d..

과고1학기 중간고사대비 심화문제 모음(2) [내부링크]

1. 다음 제시문을 읽고 물음에 답하시오. 다항식 $\displaystyle f ( x) $와 $\displaystyle g ( x) $에 대해 $\displaystyle f ( x) $라는 다항식을 $\displaystyle g ( x) $로 나누었을 때 몫을 $\displaystyle Q ( x) $, 나머지를 $\displaystyle R ( x) $라고 하면 다음과 같은 성질을 만족한다. (단, $\displaystyle f ( x) $의 차수가 $\displaystyle g ( x) $의 차수보다 크다.) $$\displaystyle \frac {f ( x)} {g ( x)} = \frac {g ( x)Q ( x)+R ( x)} {g ( x)} =Q ( x)+ \frac {R ( x)} {g (..

[수학의 기초] 함수에 대하여(3) - 일대일대응 역함수 [내부링크]

Definition1.(일대일 대응) Let $X,~Y$ be sets, and let $f~ :~ X \longrightarrow Y$ be a function. We say that $f$ is bijective if $f$ is both injective and surjective. bijective는 일대일 대응이으로 치역과 공역이 같은 함수이면서 동시에 $1-1$ 함수이다. Proposition 2. Let $X,~Y$ be sets, and let $f~ :~ X \longrightarrow Y$ be a function. Then $f$ is bijective if and only if $\forall y \in Y$, $\exists! x \in X$ such that \(f(x) = y\..

[수학의 기초] 함수에 대하여(2) - 함수의 종류 [내부링크]

Injectivity(단사) and Surjectivity(전사) injective와 surjective에 대한 용어가 교과과정이 바뀜에 따라 계속 바뀌어져 학생들이 많이 힘들어 한다. 예를들어 단사함수, 1-1함수, 전사함수, 위로의 함수, 치역과 공역이 같은 함수, 1-1대응, 전단사함수 등등 같은 용어가 서로 다르게 표현되어 많이 헷갈린다. 이 이용을 정리해보자. Definition 1.(일대일함수, 단사함수) Let $X,~Y$ be sets, and let $f ~:~ X \rightarrow Y$ be a function. We say that $f$ is injective(단사함수) (sometimes called one-to-one(일대일함수)) if for all $x_1 ,~x_2 \i..

[수학의 기초] 함수에 대하여(1) - 함수의 정의와 합성 [내부링크]

Definition 1(함수의 정의). Let $X$ and $Y$ be sets. A function $f$ from $X$ to $Y$ is an object that, for each element $x \in X$, assigns an element $y \in Y$ . We use the notation $f ~:~ X \rightarrow Y$ to denote a function as described. We write $f(x) = y$ or $f ~: ~x \mapsto y $ to denote that the element in $Y$ assigned to $x$ is $y$. We call $X$ the domain(정의역) of $f$, and we call $Y$ the codom..

과학고 2020년도 1-1 중간고사대비(18회) [내부링크]

1. $\displaystyle k $가 자연수일 때, 모든 실수 $\displaystyle x $에 대하여 $$\displaystyle p ( p ( x))= \left\{ p ( x) \right\} ^ {k} $$ 을 만족하고 계수가 실수인 상수가 아닌 다항식 $\displaystyle p ( x) $를 모두 구하여라. 정답 및 풀이를 보려면 아래를 클릭하세요. 더보기 정답 및 풀이 $\displaystyle p ( x)=a _ {n} x ^ {n} +a _ {n-1} x ^ {n-1} + \cdots +a _ {1} x+a _ {0} $ ($\displaystyle a _ {0} ,~a _ {1} ,~ \cdots ,~a _ {n} $는 실수, $\displaystyle a _ {n} \neq 0..

과고1학기 중간고사대비 심화문제 모음 [내부링크]

1. $\displaystyle k $가 자연수일 때, 모든 실수 $\displaystyle x $에 대하여 $$\displaystyle p ( p ( x))= \left\{ p ( x) \right\} ^ {k} $$ 을 만족하고 계수가 실수인 상수가 아닌 다항식 $\displaystyle p ( x) $를 모두 구하여라. 정답 및 풀이를 보려면 아래를 클릭하세요. 더보기 정답 및 풀이 $\displaystyle p ( x)=a _ {n} x ^ {n} +a _ {n-1} x ^ {n-1} + \cdots +a _ {1} x+a _ {0} $ ($\displaystyle a _ {0} ,~a _ {1} ,~ \cdots ,~a _ {n} $는 실수, $\displaystyle a _ {n} \neq 0..

과고 시험대비 [내부링크]

실수 $ a $와 함수 $ f \left ( x \right ) =\ln \left ( x ^ {4} +1 \right ) -c $ ($ c>0 $인 상수)에 대하여 함수 $ g \left ( x \right ) $를 $$ g \left ( x \right ) = \int _ {a} ^ {x} {f \left ( t \right ) dt} $$ 라 하자. 함수 $ y=g \left ( x \right ) $의 그래프가 $ x $축과 만나는 서로 다른 점의 개수가 $ 2 $가 되도록 하는 모든 $ a $의 값을 작은 수부터 크기순으로 나열하면 $ \alpha _ {1} ,~ \alpha _ {2} ,~ \cdots ,~ \alpha _ {m} $ ($ m $은 자연수)이다. $ a= \alpha _ {1}..

과고 시험 [내부링크]

실수 $ a $와 함수 $ f \left ( x \right ) =\ln \left ( x ^ {4} +1 \right ) -c $ ($ c>0 $인 상수)에 대하여 함수 $ g \left ( x \right ) $를 $$ g \left ( x \right ) = \int _ {a} ^ {x} {f \left ( t \right ) dt} $$ 라 하자. 함수 $ y=g \left ( x \right ) $의 그래프가 $ x $축과 만나는 서로 다른 점의 개수가 $ 2 $가 되도록 하는 모든 $ a $의 값을 작은 수부터 크기순으로 나열하면 $ \alpha _ {1} ,~ \alpha _ {2} ,~ \cdots ,~ \alpha _ {m} $ ($ m $은 자연수)이다. $ a= \alpha _ {1}..

2020학년도 울산과고 1학년 1학기 중간대비(12회) [내부링크]

실수 $ a $와 함수 $ f \left ( x \right ) =\ln \left ( x ^ {4} +1 \right ) -c $ ($ c>0 $인 상수)에 대하여 함수 $ g \left ( x \right ) $를 $$ g \left ( x \right ) = \int _ {a} ^ {x} {f \left ( t \right ) dt} $$ 라 하자. 함수 $ y=g \left ( x \right ) $의 그래프가 $ x $축과 만나는 서로 다른 점의 개수가 $ 2 $가 되도록 하는 모든 $ a $의 값을 작은 수부터 크기순으로 나열하면 $ \alpha _ {1} ,~ \alpha _ {2} ,~ \cdots ,~ \alpha _ {m} $ ($ m $은 자연수)이다. $ a= \alpha _ {1}..

2020학년도 울산과고 1학년 1학기 중간대비(11회) [내부링크]

실수 $ a $와 함수 $ f \left ( x \right ) =\ln \left ( x ^ {4} +1 \right ) -c $ ($ c>0 $인 상수)에 대하여 함수 $ g \left ( x \right ) $를 $$ g \left ( x \right ) = \int _ {a} ^ {x} {f \left ( t \right ) dt} $$ 라 하자. 함수 $ y=g \left ( x \right ) $의 그래프가 $ x $축과 만나는 서로 다른 점의 개수가 $ 2 $가 되도록 하는 모든 $ a $의 값을 작은 수부터 크기순으로 나열하면 $ \alpha _ {1} ,~ \alpha _ {2} ,~ \cdots ,~ \alpha _ {m} $ ($ m $은 자연수)이다. $ a= \alpha _ {1}..

과학고 2017년도 1학년 1학기 중간고사(10회) [내부링크]

실수 $ a $와 함수 $ f \left ( x \right ) =\ln \left ( x ^ {4} +1 \right ) -c $ ($ c>0 $인 상수)에 대하여 함수 $ g \left ( x \right ) $를 $$ g \left ( x \right ) = \int _ {a} ^ {x} {f \left ( t \right ) dt} $$ 라 하자. 함수 $ y=g \left ( x \right ) $의 그래프가 $ x $축과 만나는 서로 다른 점의 개수가 $ 2 $가 되도록 하는 모든 $ a $의 값을 작은 수부터 크기순으로 나열하면 $ \alpha _ {1} ,~ \alpha _ {2} ,~ \cdots ,~ \alpha _ {m} $ ($ m $은 자연수)이다. $ a= \alpha _ {1}..

[킬러문항] 2018학년도 가형 6월 30번 [더플러스수학] [내부링크]

실수 $ a $와 함수 $ f \left ( x \right ) =\ln \left ( x ^ {4} +1 \right ) -c $ ($ c>0 $인 상수)에 대하여 함수 $ g \left ( x \right ) $를 $$ g \left ( x \right ) = \int _ {a} ^ {x} {f \left ( t \right ) dt} $$ 라 하자. 함수 $ y=g \left ( x \right ) $의 그래프가 $ x $축과 만나는 서로 다른 점의 개수가 $ 2 $가 되도록 하는 모든 $ a $의 값을 작은 수부터 크기순으로 나열하면 $ \alpha _ {1} ,~ \alpha _ {2} ,~ \cdots ,~ \alpha _ {m} $ ($ m $은 자연수)이다. $ a= \alpha _ {1}..

[수학의 기초] 함수의 극대와 극소 정의 [내부링크]

거의 모든 교과서에 나오는 극대, 극소의 정의는 아래와 같다. 극대와 극소의 정의에 대해 좀더 자세히 알아보자. 정의1. 극대와 극소 1) 함수 $f(x)$가 $x=a$를 포함하는 어떤 열린구간에 속하는 모든 $x$에 대하여 $$f(x) \leq f(a)$$이면 함수 $f(x)$는 $x=a$에서 극대라 하며, $f(a)$를 극댓값이라고 한다. 2) 함수 $f(x)$가 $x=a$를 포함하는 어떤 열린구간에 속하는 모든 $x$에 대하여 $$f(x) \geq f(a)$$이면 함수 $f(x)$는 $x=a$에서 극소라 하며, $f(a)$를 극솟값이라고 한다. 이 때, 극댓값과 극솟값을 통틀어 극값이라 한다. 이것을 영어로 표현해 보자. Definition : Local Maximum and Minimum A fu..

[더플러스수학] 2007년 과학고1 1학기 중간고사 (9회) [내부링크]

함수 $ f ( x) $에 대하여 옳은 것만을 에서 있는 대로 고른 것은? [4점][2009년 6월] ㄱ. $ f ( x)=x ^ {2} ~ $이면 $ \lim\limits _ {x \rightarrow 0} {} {\large\frac {e ^ {f ( x)} -1} {x}} =0~ $이다. ㄴ. $ \lim\limits _ {x \rightarrow 0} {\large \frac {e ^ {x} -1} {f ( x)}} =1~ $이면 $ \lim\limits _ {x \rightarrow 0} {\large \frac {3 ^ {x} -1} {f ( x)} }=\ln 3~ $이다. ㄷ. $ \lim\limits _ {x \rightarrow 0} {} f ( x)=0~ $이면 $ \lim\limits..

[평가원기출] 2010학년도 가형 6월 29번 [내부링크]

함수 $ f ( x) $에 대하여 옳은 것만을 에서 있는 대로 고른 것은? [4점][2009년 6월] ㄱ. $ f ( x)=x ^ {2} ~ $이면 $ \lim\limits _ {x \rightarrow 0} {} {\large\frac {e ^ {f ( x)} -1} {x}} =0~ $이다. ㄴ. $ \lim\limits _ {x \rightarrow 0} {\large \frac {e ^ {x} -1} {f ( x)}} =1~ $이면 $ \lim\limits _ {x \rightarrow 0} {\large \frac {3 ^ {x} -1} {f ( x)} }=\ln 3~ $이다. ㄷ. $ \lim\limits _ {x \rightarrow 0} {} f ( x)=0~ $이면 $ \lim\limits..

[더플러스수학] 과학고 2015년도 1학년 1학기 중간지필고사(8회) [내부링크]

실수 전체의 집합에서 이계도함수를 갖는 함수 $ f ( x) $에 대하여 점 $ \mathrm A ( a,~f ( a)) $를 곡선 $ y=f ( x) $의 변곡점이라 하고, 곡선 $ y=f ( x) $ 위의 점 $ \mathrm A $에서의 접선의 방정식을 $ y=g ( x) $라 하자. 직선 $ y=g ( x) $가 함수 $ f ( x) $의 그래프와 점 $ \mathrm B ( b,~f ( b)) $에서 접할 때, 함수 $ h ( x) $를 $ h ( x)=f ( x)-g ( x) $라 하자. 에서 항상 옳은 것을 모두 고른 것은? (단, $ a \neq b $이다.) [4점] [2007학년도 수능] ㄱ. $ h ' ( b)=0 $ ㄴ. 방정식 $ h ' ( x)=0 $은 $ 3 $개 이상의 실근을..

[더플러스수학] 2007학년도 수능가 29번 [내부링크]

실수 전체의 집합에서 이계도함수를 갖는 함수 $ f ( x) $에 대하여 점 $ \mathrm A ( a,~f ( a)) $를 곡선 $ y=f ( x) $의 변곡점이라 하고, 곡선 $ y=f ( x) $ 위의 점 $ \mathrm A $에서의 접선의 방정식을 $ y=g ( x) $라 하자. 직선 $ y=g ( x) $가 함수 $ f ( x) $의 그래프와 점 $ \mathrm B ( b,~f ( b)) $에서 접할 때, 함수 $ h ( x) $를 $ h ( x)=f ( x)-g ( x) $라 하자. 에서 항상 옳은 것을 모두 고른 것은? (단, $ a \neq b $이다.) [4점] [2007학년도 수능] ㄱ. $ h ' ( b)=0 $ ㄴ. 방정식 $ h ' ( x)=0 $은 $ 3 $개 이상의 실근을..

[더플러스수학] 2017년 가 교육청 7월 30번 [내부링크]

상수항을 포함한 모든 항의 계수가 유리수인 이차함수 $ f ( x) $가 있다. 함수 $ g ( x) $가 $$ g ( x)= \left | f ' ( x) \right | e ^ {f ( x)} $$ 일 때, 함수 $ g ( x) $는 다음 조건을 만족시킨다. (가) 함수 $ g ( x) $는 $ x=2 $에서 극솟값을 갖는다. (나) 함수 $ g ( x) $의 최댓값은 $ 4 \sqrt {e} $이다. (다) 방정식 $ g ( x)=4 \sqrt {e} $의 근은 모두 유리수이다. $ \left | f ( -1) \right | $의 값을 구하시오. [4점] https://youtu.be/TlgOb62KUmM 정답 및 풀이를 보려면 아래를 클릭하세요. 더보기 정답 71 [출제의도] 미분을 활용하여 함..

[더플러스수학] 2015년 4월 21번 [내부링크]

함수 $ f \left ( x \right ) = { \begin {cases} \left ( x-2 \right ) ^ {2} e ^ {x} +k _ { {} _ { {} _ {} } } & ~ \left ( x \geq 0 \right ) _ { {} _ { {} _ {} } } \\-x ^ {2 ^ { {} ^ { {} ^ {} } } } & ~ \left ( x

[수학의 기초] 도함수의 모순-Derivative paradox [내부링크]

https://youtu.be/9vKqVkMQHKk 도함수의 모순

[수학의 기초] 인수분해 응용 $a+b+c=0$이면 $a^3+b^3+c^3=3abc$ [내부링크]

$\ast $ $a+b+c=0$이면 $a^3+b^3+c^3=3abc$ (증명) $a+b+c=0$이라 가정하자. 그러면 $$a^3 +b^3 +c^3 -3abc=(a+b+c)(a^2 +b^2 +c^2 -ab-bc-ca)=0$$ 따라서 $$a^3 +b^3 +c^3 =3abc$$ 예제 실력정석 수상 2-4의 (2)번 다음식을 인수분해하여라. 한번풀어보세요. $$ ( a-x) ^ {3} + ( b-x) ^ {3} - ( a+b-2x) ^ {3} $$ 정답 및 풀이를 보려면 아래를 클릭하세요. 더보기 (풀이) $$\begin{align} ( a-x) ^ {3} + ( b-x) ^ {3} - ( a+b-2x) ^ {3}=( a-x) ^ {3} + ( b-x) ^ {3} + ( -a-b+2x) ^ {3}\end{ali..

[수학의 기초] 일차식 이차식 기저를 이용한 평균값의 정리 증명 [내부링크]

https://plusthemath.tistory.com/295 [수학의 기초] 일차식 기저, 차원, 표준기저-1 직선의 표현 중2에서 시작하자. 두 점 $(1,~2),~(3,~-4)$을 지나는 직선을 어떻게 구할까? 먼저 직선은 $y=ax+b$ 꼴로 나타낼 수 있으므로 이 식에 $(1,~2),~(3,~-4)$를 대입하면 $$\begin{align}2&=a+b\\-4&=3a+b.. plusthemath.tistory.com 앞에서 일차다항식이 기저가 $1,~x$인 표준기저를 가짐을, 또 $a \neq b$일 때, $x-a,~x-b$를 기저로 가짐을 보였다. 이제 이것과 롤의 정리를 이용하여 평균값의 정리를 증명해 보자. 또, 평균값의 정리를 2계도함수까지 확장해보자. 롤의 정리 $f$가 $[a,~b]$에..

과학고1학년 중간고사 대비6 [내부링크]

https://youtu.be/1LcKkT19nw8 공식 삼차함수 $f(x)=(x-\alpha)(x-\beta)(x-\gamma)$ 에 대하여 $$\int_{\alpha}^{\beta} a(x-\alpha)(x-\beta)(x-\gamma)dx= \frac{a}{6} (\beta-\alpha)^3 \left(\textcolor{blue}{ \gamma- \frac{\alpha+\beta}{2}}\right)$$ 이다. 이것을 증명하기 위해 다음을 먼저 보이자. $$\int_{a}^{b} a(x-\alpha) \left(x- \frac{\alpha+\beta}{2} \right)(x-\beta)dx=0~~\cdots\cdots~(\mathrm{i})$$ (증명) $t=x-\frac{\alpha+\beta}..

[수학의 기초] 삼차함수 적분 공식[팁] [내부링크]

https://youtu.be/1LcKkT19nw8 공식 삼차함수 $f(x)=(x-\alpha)(x-\beta)(x-\gamma)$ 에 대하여 $$\int_{\alpha}^{\beta} a(x-\alpha)(x-\beta)(x-\gamma)dx= \frac{a}{6} (\beta-\alpha)^3 \left(\textcolor{blue}{ \gamma- \frac{\alpha+\beta}{2}}\right)$$ 이다. 이것을 증명하기 위해 다음을 먼저 보이자. $$\int_{a}^{b} a(x-\alpha) \left(x- \frac{\alpha+\beta}{2} \right)(x-\beta)dx=0~~\cdots\cdots~(\mathrm{i})$$ (증명) $t=x-\frac{\alpha+\beta}..

과학고1학년 중간고사 대비5 [내부링크]

원의 방정식에서 극선을 시작합니다. 예를 통해 원에서 극선을 정의할께요. 원의 극선 원 $x^2+y^2=1$과 원 밖의 점 $\mathrm {P} (2,~3)$을 생각하자. 다음 그림처럼 점 $\mathrm P$에서 원에 접선을 그으면 접선이 두 개 나오는데 그 접점을 $\mathrm A ,~\mathrm B$라 할 때, 두 점 $\mathrm A ,~\mathrm B$를 지나는 직선 $l$을 점 $\mathrm P$의 극선(polar line)이라고 하고 점 $\mathrm P$를 극(점)-pole이라고 합니다. 여기서 직선 $l$의 방정식이 마치 점 $\mathrm P$를 접점인 것처럼 원 $x^2+y^2=1$에 잘못 접선을 구한 직선이 극선 $l$이다. 즉 직선 $l$이 $$\textcolor{re..

[수학의 기초] 이차곡선과 극선-1 [내부링크]

원의 방정식에서 극선을 시작합니다. 예를 통해 원에서 극선을 정의할께요. 원의 극선 원 $x^2+y^2=1$과 원 밖의 점 $\mathrm {P} (2,~3)$을 생각하자. 다음 그림처럼 점 $\mathrm P$에서 원에 접선을 그으면 접선이 두 개 나오는데 그 접점을 $\mathrm A ,~\mathrm B$라 할 때, 두 점 $\mathrm A ,~\mathrm B$를 지나는 직선 $l$을 점 $\mathrm P$의 극선(polar line)이라고 하고 점 $\mathrm P$를 극(점)-pole이라고 합니다. 여기서 직선 $l$의 방정식이 마치 점 $\mathrm P$를 접점인 것처럼 원 $x^2+y^2=1$에 잘못 접선을 구한 직선이 극선 $l$이다. 즉 직선 $l$이 $$\textcolor{re..

[수학의 기초] 2013과고 중간고사 문제 - 함수방정식과 주기함수 [내부링크]

22. 정의역과 공역이 실수 전체 집합인 함수 $ f ( x) $가 다음 두 조건을 만족한다. (가) $ f ( a)=0 $ (나) 임의의 실수 $ x $에 대하여, $$ f ( x+y)+f ( x-y)=2f ( x)f ( y) $$ 이 때, 다음을 증명하시오. [2013 과고1 1학기 중간 서술형7] https://youtu.be/lPr8-E8kOA4 (1) $ f ( x) $는 주기함수이다. [2점] 단, 주기함수란 모든 실수 $x$에 대하여 $$f(x+p)=f(x)$$를 만족하는 $0$이 아닌 실수 $p$가 존재할 때, 함수 $f$는 주기함수라 하고, 실수 $p$ 중 최소의 양수 $p$를 주기라고 한다. (2) $ f ( x) $는 우함수이다. [2점] 우함수란, 모든 실수 $x$에 대하여 $$f(..

[옥동수학학원][더플러스수학]##[수학의 기초] 일차식 기저, 차원, 표준기저-1 [내부링크]

직선의 표현 중2에서 시작하자. 두 점 $(1,~2),~(3,~-4)$을 지나는 직선을 어떻게 구할까? 먼저 직선은 $y=ax+b$ 꼴로 나타낼 수 있으므로 이 식에 $(1,~2),~(3,~-4)$를 대입하면 $$\begin{align}2&=a+b\\-4&=3a+b\end{align}$$ 위의 두 식을 연립하면 $a=-3,~b= 5$이므로 $$y=-3x+5$$ 이다. 여기서 나의 고민은 "두 점을 지나는 직선"을 "세 점을 지나는 2차함수"로, "네 점을 지나는 3차함수로"로 계속 올라 간다면 계속 연립방정식을 풀어야 한다는 것이다. 꼭 일차함수는 $y=ax+b$로, 이차함수는 $y=ax^2 +bx+c$로, 삼차함수는 $y=ax^3 +bx^2 +cx+d$로 놓아야 하는가? 이 부분은 나만의 고민? 그럴리..

과학고1학년 중간고사 대비4 [내부링크]

2. 삼각행렬의 고윳값은 그 행렬의 대각원소들임을 보여라. https://youtu.be/1XNEOX-QZ9E YouTube www.youtube.com 3. $ A $를 $ n \times n $행렬이라 할 때, $ A $가 특이행렬이기 위한 필요충분조건은 $ \lambda =0 $이 $ A $의 고유값임을 증명하여라. https://youtu.be/z6ySn83qVE4 YouTube www.youtube.com 4. $ A $가 비특이행렬이고 $ \lambda $가 $ A $의 고윳값이라 하자. $ \large{\frac {1} {\lambda }} $가 $ A ^ {-1} $의 고윳값임을 증명하라. https://youtu.be/HgGecSWsUtI YouTube www.youtube.com 5. ..

과학고1학년 중간고사 대비3 [내부링크]

2. 삼각행렬의 고윳값은 그 행렬의 대각원소들임을 보여라. https://youtu.be/1XNEOX-QZ9E YouTube www.youtube.com 3. $ A $를 $ n \times n $행렬이라 할 때, $ A $가 특이행렬이기 위한 필요충분조건은 $ \lambda =0 $이 $ A $의 고유값임을 증명하여라. https://youtu.be/z6ySn83qVE4 YouTube www.youtube.com 4. $ A $가 비특이행렬이고 $ \lambda $가 $ A $의 고윳값이라 하자. $ \large{\frac {1} {\lambda }} $가 $ A ^ {-1} $의 고윳값임을 증명하라. https://youtu.be/HgGecSWsUtI YouTube www.youtube.com 5. ..

[더플러스수학]과학고2학년 고급수학 2학기기말대비 프린트 [내부링크]

2. 삼각행렬의 고윳값은 그 행렬의 대각원소들임을 보여라. https://youtu.be/1XNEOX-QZ9E YouTube www.youtube.com 3. $ A $를 $ n \times n $행렬이라 할 때, $ A $가 특이행렬이기 위한 필요충분조건은 $ \lambda =0 $이 $ A $의 고유값임을 증명하여라. https://youtu.be/z6ySn83qVE4 YouTube www.youtube.com 4. $ A $가 비특이행렬이고 $ \lambda $가 $ A $의 고윳값이라 하자. $ \large{\frac {1} {\lambda }} $가 $ A ^ {-1} $의 고윳값임을 증명하라. https://youtu.be/HgGecSWsUtI YouTube www.youtube.com 5. ..

[수학의 기초] 우함수 기함수 미분 [내부링크]

우함수 미분하면 기함수? 기함수 미분하면 우함수? 증명은? 우함수란 모든 실수 $x$에 대하여 $$f(-x)=f(x)$$를 만족하는 함수이다. 영어로는 우함수(even-function)으로 여기서 even은 짝수를 의미한다. 즉 $y=x^{짝수}$인 함수는 $y$축에 대칭이므로 이 함수를 일반화하여 $y$축에 대칭인 함수를 우함수라고 한다. 마찬가지로 기함수(odd-function)도 $$f(-x)=-f(x)$$를 만족하는 함수이다. "우" 짝수(even), "기" 홀수(odd) 우함수 미분은 기함수 증명 함수 $\displaystyle f$가 미분가능하며 기함수이면 그 도함수 $f'$는 우함수 $$f'(-x)=f'(x)$$이다. (증명) 여기서 $$f'(-x)$$와 $$(f(-x))'$$을 구별해야 한..

[중간고사 대비2] 2013학년도 과고1 1학기 중간고사 [내부링크]

https://m.facebook.com/story.php?story_fbid=2244428215854713&id=1585733961724145

컴퓨터 단축기 모음 [내부링크]

https://m.facebook.com/story.php?story_fbid=2244428215854713&id=1585733961724145

[시험대비] 2019과고 1학년 2학기 기말고사대비문제(미적분,수1) [내부링크]

1. 다음 식의 값을 구하여라.[2.5점] $$ \cos ^ {2} \theta +\cos ^ {2} \left ( \theta + \frac {\pi } {3} \right ) -\cos \theta \cos \left ( \theta + \frac {\pi } {3} \right ) $$ [2011 과고1 심화수학 2학기 중간7] 2. $ x $의 방정식 $ 25x ^ {2} -ax+12=0 $의 두 근이 $ \sin 2 \theta ,~\cos 2 \theta $일 때, $ \tan \theta $의 값의 합을 구하여라.[2.5점] [2011 과고1 심화수학 2학기 중간8] 3. $ \alpha ,~ \beta $가 $ \tan \alpha -\tan \beta =1 $을 만족시킨다. $ \alph..

[서술형 대비] 과학고 서술형 대비 미적분 증명 [내부링크]

$ f ( x)=x ^ {3} +4x ^ {2} -8x+k,~g ( x)=x ^ {2} +x-2 $일 때, $ f ( x) $의 세 근을 $ \alpha ,~ \beta ,~ \gamma $라 하자. $ g ( \alpha )g ( \beta )g ( \gamma )=28 $일 때, $ k $의 값을 구하는 풀이과정과 답을 쓰시오. [4점] https://youtu.be/Mudum5GePIQ 2019/10/11 - [과학고/1학년1학기] - 2018학년도 1학년 1학기 중간고사 2018학년도 1학년 1학기 중간고사 plusthemath.tistory.com

[내부링크]

$ f ( x)=x ^ {3} +4x ^ {2} -8x+k,~g ( x)=x ^ {2} +x-2 $일 때, $ f ( x) $의 세 근을 $ \alpha ,~ \beta ,~ \gamma $라 하자. $ g ( \alpha )g ( \beta )g ( \gamma )=28 $일 때, $ k $의 값을 구하는 풀이과정과 답을 쓰시오. [4점] https://youtu.be/Mudum5GePIQ 2019/10/11 - [과학고/1학년1학기] - 2018학년도 1학년 1학기 중간고사 2018학년도 1학년 1학기 중간고사 plusthemath.tistory.com

15단원 [내부링크]

$ f ( x)=x ^ {3} +4x ^ {2} -8x+k,~g ( x)=x ^ {2} +x-2 $일 때, $ f ( x) $의 세 근을 $ \alpha ,~ \beta ,~ \gamma $라 하자. $ g ( \alpha )g ( \beta )g ( \gamma )=28 $일 때, $ k $의 값을 구하는 풀이과정과 답을 쓰시오. [4점] https://youtu.be/Mudum5GePIQ 2019/10/11 - [과학고/1학년1학기] - 2018학년도 1학년 1학기 중간고사 2018학년도 1학년 1학기 중간고사 plusthemath.tistory.com

14단원 [내부링크]

$ f ( x)=x ^ {3} +4x ^ {2} -8x+k,~g ( x)=x ^ {2} +x-2 $일 때, $ f ( x) $의 세 근을 $ \alpha ,~ \beta ,~ \gamma $라 하자. $ g ( \alpha )g ( \beta )g ( \gamma )=28 $일 때, $ k $의 값을 구하는 풀이과정과 답을 쓰시오. [4점] https://youtu.be/Mudum5GePIQ 2019/10/11 - [과학고/1학년1학기] - 2018학년도 1학년 1학기 중간고사 2018학년도 1학년 1학기 중간고사 plusthemath.tistory.com

13단원 [내부링크]

$ f ( x)=x ^ {3} +4x ^ {2} -8x+k,~g ( x)=x ^ {2} +x-2 $일 때, $ f ( x) $의 세 근을 $ \alpha ,~ \beta ,~ \gamma $라 하자. $ g ( \alpha )g ( \beta )g ( \gamma )=28 $일 때, $ k $의 값을 구하는 풀이과정과 답을 쓰시오. [4점] https://youtu.be/Mudum5GePIQ 2019/10/11 - [과학고/1학년1학기] - 2018학년도 1학년 1학기 중간고사 2018학년도 1학년 1학기 중간고사 plusthemath.tistory.com

12단원 [내부링크]

$ f ( x)=x ^ {3} +4x ^ {2} -8x+k,~g ( x)=x ^ {2} +x-2 $일 때, $ f ( x) $의 세 근을 $ \alpha ,~ \beta ,~ \gamma $라 하자. $ g ( \alpha )g ( \beta )g ( \gamma )=28 $일 때, $ k $의 값을 구하는 풀이과정과 답을 쓰시오. [4점] https://youtu.be/Mudum5GePIQ 2019/10/11 - [과학고/1학년1학기] - 2018학년도 1학년 1학기 중간고사 2018학년도 1학년 1학기 중간고사 plusthemath.tistory.com

[더플러스수학] 2018 울산과고 중간 8번 문제 [내부링크]

$ f ( x)=x ^ {3} +4x ^ {2} -8x+k,~g ( x)=x ^ {2} +x-2 $일 때, $ f ( x) $의 세 근을 $ \alpha ,~ \beta ,~ \gamma $라 하자. $ g ( \alpha )g ( \beta )g ( \gamma )=28 $일 때, $ k $의 값을 구하는 풀이과정과 답을 쓰시오. [4점] https://youtu.be/Mudum5GePIQ 2019/10/11 - [과학고/1학년1학기] - 2018학년도 1학년 1학기 중간고사 2018학년도 1학년 1학기 중간고사 plusthemath.tistory.com

[수학의 기초] 함수의 정의와 함수의 종류 -2 [내부링크]

함수 $f~:~X~\longrightarrow ~Y$에 대하여 정의 일대일 함수 $X$의 서로 다른 원소에 $Y$의 서로 다른 원소가 대응되는 함수를 일대일 함수라고 한다. 즉 집합 $X$의 임의의 원소 $x_1 ,~x_2$에 대하여 $x_1 \neq x_2$이면 $f(x_1 ) \neq f(x_2 )$이다. $f(x_1 ) = f(x_2 )$이면 $x_1 = x_2$이다. De nition A function f is one-to-one or injective if and only if $f(x) =f(y)$ implies $x = y$ for all $x,~y$ in the domain $X$ of $f$. Formally: $$\forall x,~y \in X, f(x) = f(y) ~\longri..

[수학의 기초] 로그함수의 정의(대학에서)-3 [내부링크]

https://plusthemath.tistory.com/267 [수학의 기초] 로그함수의 정의(대학에서)-2 https://plusthemath.tistory.com/263 [수학의 기초] 로그함수의 정의(대학에서)-1 고등학교 수학에서는 로그함수($\ln x$)를 정의하는 절차는 다음과 같다. 1. 중1에서 자연수에서 지수법칙을 정의하기 $\textcol.. plusthemath.tistory.com 전편에 있어 이제 자연상수 $e$를 정의할 수 있게 되었다. 정의 자연상수 $e$ $e$는 $\ln e=1$인 수이다. 수를 소수 $20$번째 자리까지 계산하면 $$e= 2.71828182845904523536$$ 이다. 이 수는 순환하지 않는 무한소수 즉, 무리수이다. $e$가 무리수임에 대한 증명은 ..

[수학의 기초] 함수의 정의와 함수의 종류 -1 [내부링크]

정의 함수 공집합이 아닌 두 집합 $X,~Y$가 존재하여 집합 $X$의 각 원소에 집합 $Y$의 원소가 하나씩 대응할 때 이 대응을 $X$에서 $Y$로의 함수 라 하고 문자 $f$를 써서 $f~:~X ~\longrightarrow~Y$ 또는 $\require{AMScd}\begin{CD} X @>{f}>> Y\end{CD}$ 또, 함수 $f$에 의하여 $X$의 원소 $x$에 $Y$의 원소 $y$가 대응하는 것을 $f~:~x ~\longrightarrow~y$ 또는 $\require{AMScd}\begin{CD} x @>{f}>> y\end{CD}$, $y=f(x)$ 등으로 나타내고, $y$는 $x$의 함수이다 라고 한다. 이때 $y$를 $f$에 의한 $x$ 의 함숫값이라 하고 $f(x)$ 로 나타낸다. ..

[인하대수리논술] 2019학년도 인하대 수리논술 오후 [내부링크]

[문제 1] (30점) 다음 제시문을 읽고 물음에 답하시오. (가) $ 0

연습 [내부링크]

https://youtu.be/SA6YXrtr7ic https://youtu.be/mtmlNf1npLs https://youtu.be/mtmlNf1npLs

[수학의 기초] 로그함수의 정의(대학에서)-2 [내부링크]

https://plusthemath.tistory.com/263 [수학의 기초] 로그함수의 정의(대학에서)-1 고등학교 수학에서는 로그함수($\ln x$)를 정의하는 절차는 다음과 같다. 1. 중1에서 자연수에서 지수법칙을 정의하기 $\textcolor{red}{m,~n}$ 은 자연수, $a>0,~a \ne 1$인 실수에 대해 $$\begin{align} &a^m \t.. plusthemath.tistory.com 이 과정의 한계지점 (1) 지수보다 수열을 나중에 배움 (2) 수열의 수렴은 미적분과정에서 배움 (3) 단조수렴정리 "증가수열이고 위로 유계면 수렴한다."는 대학과정 혹은 고급수학과정임 전편에서 본 위와 같은 한계를 극복하기 위해 대학에서는 먼저 모든 학생들이 이과면 미적분을 배웠기 때문에 $..

[상경계열논술]2020학년도 한양대 상경계열 모의논술2 [내부링크]

https://tv.kakao.com/v/404116490 [문제 2번] 다음 제시문을 읽고 물음에 답하시오. (50점) (가) $f(x)=\sqrt{|x-1|-2}$ (나) $f(x)=\frac{1}{2}x^2 ,~g(x)=\sqrt {2|x|}$ (다) $f(x)=\sqrt{ ||x-1|-1|},~g(x)=-x^2 +1$ 1. 제시문 (가)에서 주어진 함수 $f(x)$에 대하여 $-5 \leq x \leq 7$와 $0 \leq y \leq f(x)$를 만족하는 정수 순서쌍 $(x,~y)$의 개수를 구하시오. 2. 제시문 (나)에서 주어진 두 함수 $f(x)$와 $g(x)$의 그래프로 둘러싸인 도형의 넓이를 구하시오. 3. 제시문 (다)에서 주어진 두 함수 $f(x)$와 $g(x)$의 그래프가 $n$개..

[상경계열논술] 2020학년도 한양대 상경계열 모의논술1 [내부링크]

https://tv.kakao.com/v/404115266 [문제 2번] 다음 제시문을 읽고 물음에 답하시오. (50점) 전체집합 $U=\left\{ ~x~|~x는~9~이하의~자연수\right\}$의 두 부분집합 $A=\left\{1,~2,~3,~4,~5 \right\}$, $B=\left\{2,~4,~6,~8 \right\}$에 대하여 $U$의 부분집합 $C$는 $C \cap (A^c \cup B)=\left\{2,~4,~7 \right\}$를 만족한다. 이산확률변수 $X$ 의 확률질량함수가 $\mathrm P (X =x_i )=p_i$ ($i=1,~2,~\cdots,~n$) 일 때, 평균 $\mathrm E (X)$, 분산 $\mathrm V (X)$, 표준편차 $\sigma (X)$는 다음과 같다...

[수학의 기초] 로그함수의 정의(대학에서)-1 [내부링크]

고등학교 수학에서는 로그함수($\ln x$)를 정의하는 절차는 다음과 같다. 1. 중1에서 자연수에서 지수법칙을 정의하기 $\textcolor{red}{m,~n}$ 은 자연수, $a>0,~a \ne 1$인 실수에 대해 $$\begin{align} &a^m \times a^n =a^{m+n} &\frac{a^m }{a^n}= \begin{cases} a^{m-n} &(m>n) \\ 1&(m=n) \\ \frac{1}{a^{n-m}} &(m0,~N>0,~a \neq 1,~a > 0,~n \in \mathbb R$에 대하여 (1) $\log_a MN =\log_a M +\log_a N$ (2) $\log_a \frac{M}{N} =\log_a M -\log_a N$ (3) $\log_a M^n =n \log_..

11단원 [내부링크]

함수 $$ f ( x)= { \begin {cases} -x & & & \left ( x \leq 0 \right )\\x-1 & & & \left ( 02 \right )\end {cases} } $$ 와 상수가 아닌 다항식 $ p ( x) $에 대하여 에서 옳은 것만을 있는 대로 고른 것은? [$ 4 $점] | 보기 | ㄱ. 함수 $ p ( x)f ( x) $가 실수 전체의 집합에서 연속이면 $ p ( 0)=0 $이다. ㄴ. 함수 $ p ( x)f ( x) $가 실수 전체의 집합에서 미분가능하면 $ p ( 2)=0 $이다. ㄷ. 함수 $ p ( x) \left\{ f ( x) \right\} ^ {2} $이 실수 전체의 집합에서 미분가능하면 $ p ( x) $는 $ x ^ {2} ( x-2) ^ {2..

10단원 [내부링크]

함수 $$ f ( x)= { \begin {cases} -x & & & \left ( x \leq 0 \right )\\x-1 & & & \left ( 02 \right )\end {cases} } $$ 와 상수가 아닌 다항식 $ p ( x) $에 대하여 에서 옳은 것만을 있는 대로 고른 것은? [$ 4 $점] | 보기 | ㄱ. 함수 $ p ( x)f ( x) $가 실수 전체의 집합에서 연속이면 $ p ( 0)=0 $이다. ㄴ. 함수 $ p ( x)f ( x) $가 실수 전체의 집합에서 미분가능하면 $ p ( 2)=0 $이다. ㄷ. 함수 $ p ( x) \left\{ f ( x) \right\} ^ {2} $이 실수 전체의 집합에서 미분가능하면 $ p ( x) $는 $ x ^ {2} ( x-2) ^ {2..

9단원 [내부링크]

함수 $$ f ( x)= { \begin {cases} -x & & & \left ( x \leq 0 \right )\\x-1 & & & \left ( 02 \right )\end {cases} } $$ 와 상수가 아닌 다항식 $ p ( x) $에 대하여 에서 옳은 것만을 있는 대로 고른 것은? [$ 4 $점] | 보기 | ㄱ. 함수 $ p ( x)f ( x) $가 실수 전체의 집합에서 연속이면 $ p ( 0)=0 $이다. ㄴ. 함수 $ p ( x)f ( x) $가 실수 전체의 집합에서 미분가능하면 $ p ( 2)=0 $이다. ㄷ. 함수 $ p ( x) \left\{ f ( x) \right\} ^ {2} $이 실수 전체의 집합에서 미분가능하면 $ p ( x) $는 $ x ^ {2} ( x-2) ^ {2..

8단원 [내부링크]

함수 $$ f ( x)= { \begin {cases} -x & & & \left ( x \leq 0 \right )\\x-1 & & & \left ( 02 \right )\end {cases} } $$ 와 상수가 아닌 다항식 $ p ( x) $에 대하여 에서 옳은 것만을 있는 대로 고른 것은? [$ 4 $점] | 보기 | ㄱ. 함수 $ p ( x)f ( x) $가 실수 전체의 집합에서 연속이면 $ p ( 0)=0 $이다. ㄴ. 함수 $ p ( x)f ( x) $가 실수 전체의 집합에서 미분가능하면 $ p ( 2)=0 $이다. ㄷ. 함수 $ p ( x) \left\{ f ( x) \right\} ^ {2} $이 실수 전체의 집합에서 미분가능하면 $ p ( x) $는 $ x ^ {2} ( x-2) ^ {2..

2020학년도 수능 나형 20번 [킬러문항] [내부링크]

함수 $$ f ( x)= { \begin {cases} -x & & & \left ( x \leq 0 \right )\\x-1 & & & \left ( 02 \right )\end {cases} } $$ 와 상수가 아닌 다항식 $ p ( x) $에 대하여 에서 옳은 것만을 있는 대로 고른 것은? [$ 4 $점] | 보기 | ㄱ. 함수 $ p ( x)f ( x) $가 실수 전체의 집합에서 연속이면 $ p ( 0)=0 $이다. ㄴ. 함수 $ p ( x)f ( x) $가 실수 전체의 집합에서 미분가능하면 $ p ( 2)=0 $이다. ㄷ. 함수 $ p ( x) \left\{ f ( x) \right\} ^ {2} $이 실수 전체의 집합에서 미분가능하면 $ p ( x) $는 $ x ^ {2} ( x-2) ^ {2..

[수학의 기초] 자연상수 $e$ 무리수 증명 [내부링크]

1. $ \displaystyle e^x$이 다음과 같이 표현될 수 있다고 하자. Taylor Series $$e ^ {x} =1+ \frac {x} {1!} + \frac {x ^ {2} } {2!} + \cdots + \frac {x ^ {n} } {n!} + \cdots$$ 여기에 $x=1$을 대입하면 $$ e=1+ \frac {1} {1!} + \frac {1} {2!} + \cdots + \frac {1} {n!} + \cdots $$ 이다. (1) 다음을 증명하여라. $$ 00 $이므로 $ \displaystyle F _ {k+1} ( x) $는 증가함수이다. 또 $ \displaystyle F _ {k+1} ( 0)=0 $이다. 따라서 $ \displaystyle x>0 $일 때, $ \di..

[더플러스수학] 2020학년도 수능 나형 30번 [킬러문항] [내부링크]

최고차항의 계수가 양수인 삼차함수 $ f \left ( x \right ) $가 다음 조건을 만족시킨다. (가) 방정식 $ f \left ( x \right ) -x=0 $의 서로 다른 실근의 개수는 $ 2 $이다. (나) 방정식 $ f \left ( x \right ) +x=0 $의 서로 다른 실근의 개수는 $ 2 $이다. $ f \left ( 0 \right ) =0,~f ' \left ( 1 \right ) =1 $일 때, $ f \left ( 3 \right ) $의 값을 구하시오. [$ 4 $점] https://youtu.be/oGCffCCbfW8[구독과 좋아요] 정답 및 풀이을 보려면 아래를 클릭하세요. 더보기 정답 $51$ $ f ( x)=x $의 실근이 $ 2 $개, $ f ( x)=-x ..

[더플러스수학]2020학년도 수능 가형 21번[킬러문항] [내부링크]

실수 $t$에 대하여 $y=e^x$ 위의 점 $(t,~e^t )$에서의 접선의 방정식을 $y=f(x)$라 할 때, 함수 $y=|f(x)+k-\ln x |$가 양의 실수 전체에서 미분가능하도록 하는 실수 $k$의 최솟값을 $g(t)$라 하자. 두 실수 $a,~b$에 대하여 $\int_{a}^{b} g(t)dt=m$이라 할 때, 에서 옳은 것만을 있는 대로 고른 것은? [$4$점] | 보기 | ㄱ. $me ^ {2} $ $(\mathrm{ii})$에서 $ -2e ^ {2 \beta }

수능이 다가왔습니다. 수능준비 [내부링크]

2020학년도 수능시험이 이틀 앞으로 다가왔습니다. 이에 수능시험 하루 전날인 11월 13일에 수험생들이 알아둬야 할 것과 관련해서 작성한 원고를 이곳에 올립니다. 수험생들께서는 참조하여 11월 14일 수능시험일에 최고의 컨디션으로 최상의 성적을 거두웠으면 좋겠습니다. 수험생 여러분, 그 동안 수고 많았습니다. 좋은 결과가 있길 간절한 마음으로 두손 모읍니다. 11월 13일은 2020학년도 수능시험 D-1일, 예비 소집일이다. 이때 수험생들이 해야 할 일들을 정리해 보았다. 꼭 숙지하여 11월 14일 수능시험 당일 이로 인한 불미스러운 일이 생기지 않도록 유의했으면 한다. 특히 반입 금지 물품은 꼭 기억하여 수능시험장에 가져가지 않도록 한다. ∙수능시험장 정보를 확인하라. 일시 : 2019년 11월 1..

[수학의 기초] 기저와 기저변환 행렬 [내부링크]

기저와 기저변환행렬이란? 선형대수학 또는 고등학교 과정의 기하와 벡터 단원에서 기저(basis)란 용어가 등장한다. 중$\cdot$고등과정에서 좌표를 말할 때, 그 속에서는 기저라는 내용이 암묵적으로 들어가 있다. 예를 들어 $\mathrm P(2,~3)$이란 좌표를 말할 때는 다음과정이 진행된다. 먼저 원점(Origin)을 먼저 생각하고 원점을 지나는 서로 수직인 두 개의 축을 생각했을 때, 흔히 우리는 $x$축, $y$축을 말한다. $\mathrm P$의 좌표가 $(2,~3)$이란 말은 점 $\mathrm P$에서 $x$축에 내린 수선의 발의 눈금을 읽으면 $2$이고 $y$축에 내린 수선의 발의 눈금이 $3$이란 말이다. 이 눈금을 순서로 읽어서 $(2,~3)$으로 적는다. 여기서 순서로 쌍을 괄로로..

2010학년도 고려대 수리논술(2009년 11월 21일) [내부링크]

(가) 그림 1과 같이 곡선 $ y=x ^ {2} +1 $위에 두 점 $\mathrm P _ {1} ( a _ {1} ,~a _ {1} ^ {2} +1) $과 $\mathrm P _ {2} ( a _ {2} ,~a _ {2} ^ {2} +1) $가 있다. (단 $ a _ {1}

[수학의 기초]이차함수와 직선으로 둘러싸인 부분의 넓이 [내부링크]

이차함수와 직선으로 둘러싸인 부분의 넓이 1. 이차함수 $f(x)=ax^2 +bx+c$와 $x$축의 교점이 $\alpha,~\beta~(\alpha

[평가원 기출-킬러문항] 2018학년도 나형 9월 30번 [내부링크]

두 함수 $ f ( x) $와 $ g ( x) $가 $$ \begin{align} &f ( x)= { \begin {cases} ~0 &( x \leq 0)\\~x &( x>0)\end {cases} } \\& g ( x)= { \begin {cases} ~x ( 2-x) &( \left | x-1 \right | \leq 1)\\~0 &( \left | x-1 \right | >1)\end {cases} } \end{align}$$ 이다. 양의 실수 $ k,~a,~b $ $ ( a

[수학의 기초] 수열의 극한의 엄밀한 정의 [내부링크]

정의 "임의의 양수 $\epsilon>0$에 대하여 적당한 $N=N(\epsilon)$이 존재하여 $n>N$인 모든 $n$에 대하여 $\left| a_n -L \right|0$에 대하여 적당한 $N=N(\epsilon)$이 존재하여"라는 표현이 우리가 이해하기가 어렵다. 그래서 영어표현 "Let $\epsilon>0$ be given"을 직역하면 $\epsilon>0$이 주어졌다고 할 때, 그것에 대응하는(corresponding) $N$을 찾아야 한다. 따라서 우리는 $\left| a_n -L \right|N$인 모든 $n$에 대하여 $$\left| \frac{2}{n+1}-0 \right|

울산과고 미적분 -급수 수렴판정법 [내부링크]

심화수학1 보충자료 https://tv.kakao.com/channel/3372901/cliplink/403654400 1. 단조수열정리(단조수렴정리) 정의1. $ n \geq 1 $인 모든 $ n $에 대하여 $ a _ {n}

[옥동수학학원][더플러스수학] 2020년도 울산과고수업 개설안내 [내부링크]

안녕하세요. 더(THE)플러스수학학원입니다. 울산 옥동에 있는 울산과고 수업의 전문가 더플러스수학학원 권도형 원장의 예비울산과고 1, 2, 3학년 학생들을 위한 내신완벽대비반이 개설됩니다. 예비울산과고 1학년 수(상,하) 실력정석교재 수업+진도별 기출문제풀이를 통한 내신대비 수업 미적분 등 특강개설 수업 : 매주 (월,수) 오후 9:00 ~ 10:30 (3시간) 수업 TEST : 매주 (토) 오후 4:00 ~ 6:00 특강개설시 : 매주 (토) 오후 6:00~9:00 실력정석 연습문제 풀이 동영상 제공 https://youtu.be/Y4LNhWkFsZo YouTube 언어를 선택하세요. www.youtube.com 예비울산과고 2학년 수1, 미적분 실력정석교재 내신대비 수업 수업..

[더플러스수학]2020학년도 부산대, 경북대(AAT) 수리논술반모집 [내부링크]

권도형원장님의 수리논술 완벽대비특강 - 서울대 공대졸 - 부산대 수학과 석사 - 울산대 교육학 석사 - 포항제철고 수리논술 출강 (2014학년도~2016학년도 3년간) - “6인치 기하와 벡터” 책 감수 - “고려대 방과후 학교” 수리논술 총괄 강사 개강 : (1차) 11월 15일 (금) , (2차) 11월 18일(월) 수업시간 : (의대) 오후 1시~4시(3시간) (일반) 오후 2시~5시(3시간) 수업내용 : 기출문제풀이 및 첨삭지도, www.plusmath.net 동영상강의 제공 ※ 예약접수문의 : 052-258-4409 ,052-260-9981, 블로그를 통해 수업예약하시는 분들께는 5% 할인혜택이 주어집니다. 수업을 희망하시면 댓글로 신청 (비밀댓글로 학교, 이름 ,전화번호 메모) 을 해..

2019학년도 경북대 AAT [내부링크]

https://tv.kakao.com/v/404010631 수학(문제1) [1] 다음 글을 읽고 물음에 답하시오. (가) (1) 서로 다른 $ n $개에서 $ r $ ($ 0c>0 $, $ b ^ {2} =a ^ {2} -c ^ {2} $) 이다. (나) 타원의 방정식 $ \frac {x ^ {2} } {a ^ {2} } + \frac {y ^ {2} } {b ^ {2} } =1 $을 $ x $축의 방향으로 $ m $만큼, $ y $축의 방향으로 $ n $만큼 평행이동한 타원의 방정식은 $$ \frac { ( x-m) ^ {2} } {a ^ {2} } + \frac { ( y-n) ^ {2} } {b ^ {2} } =1 $$ 이다. (다) 타원 $$ \frac {x ^ {2} } {a ^ {2} } + ..

2020학년도 경북대 모의 AAT (의학계열) [내부링크]

https://tv.kakao.com/v/403948173 【1】 다음 글을 읽고 물음에 답하시오. (가) 두 함수 $ f ( x),~g ( x) $가 닫힌 구간에서 연속일 때, (1) $ \int _ {a} ^ {b} {kf ( x)dx} =k \int _ {a} ^ {b} {f ( x)dx} $ (단, $ k $는 상수) (2) $ \int _ {a} ^ {b} {\left\{ f ( x)\pm g ( x) \right\} dx} = \int _ {a} ^ {b} {f ( x)dx} \pm \int _ {a} ^ {b} {g ( x)dx} $ (3) $ \int _ {a} ^ {b} {f ( x)dx} =- \int _ {b} ^ {a} {f ( x)dx} $ (4) 실수 $ c $가 닫힌 구간 $ ..

2020학년도 경북대 AAT 모의논술 [내부링크]

https://tv.kakao.com/v/403946475 수학(문제 1) 【1】 다음 글을 읽고 물음에 답하시오. (가) (1) 서로 다른 $ n $개의 서로 다른 $ n $개에서 $ r $ ($ 0 \leq r \leq n $)개를 택하는 순열의 수는 $$ {} _ {n} \mathrm P _ { r } = n ( n-1) ( n-2) \cdots ( n-r+1) $$ 이다. (2) 서로 다른 $ n $개의 서로 다른 $ n $개에서 $ r $ ($ 0 \leq r \leq n $)개를 택하는 조합의 수는 $$ {} _ {n} \mathrm C _ { r } = \frac { {} _ { n } \mathrm C _ { r } } { r! } = \frac {n!} {r! ( n-r)!} $$ 이다...

[수학의 기초] 정사영 벡터 - orthogonal Projection vector [내부링크]

먼저 정사영 벡터에 대해 알아 보자. 위의 그림에서 $\overrightarrow{v}$의 종점 $\mathrm P$를 $\overrightarrow{u}$에 내린 수선의 발을 $\mathrm H$라 할 때, $\overrightarrow {\mathrm{OH}}$을 벡터 $\overrightarrow v$의 벡터 $\overrightarrow u$ 위로의 정사영벡터라고 하고 $\overrightarrow{\mathrm{OH}}$를 $\overrightarrow {\mathrm{OH}}=\mathrm{Proj}_{u}v$로 나타낸다. 또, $$\mathrm{Proj}_{u}v = \frac{ u \cdot v}{u \cdot u}u$$ (증명) $\overrightarrow {\mathrm{OH}}$는 ..

8단원 [내부링크]

이차함수 $f(x)=a(x-\alpha)(x-\beta)$ ($\alpha

6단원 [내부링크]

이차함수 $f(x)=a(x-\alpha)(x-\beta)$ ($\alpha

5단원 [내부링크]

이차함수 $f(x)=a(x-\alpha)(x-\beta)$ ($\alpha

4단원 [내부링크]

이차함수 $f(x)=a(x-\alpha)(x-\beta)$ ($\alpha

7단원 [내부링크]

조합론적 증명(combinatorial proof) https://en.wikipedia.org/wiki/Combinatorial_proof A proof by double counting. A combinatorial identity is proven by counting the number of elements of some carefully chosen set in two different ways to obtain the different expressions in the identity. Since those expressions count the same objects, they must be equal to each other and thus the identity is established..

조합론적 증명 [내부링크]

조합론적 증명(combinatorial proof) https://en.wikipedia.org/wiki/Combinatorial_proof A proof by double counting. A combinatorial identity is proven by counting the number of elements of some carefully chosen set in two different ways to obtain the different expressions in the identity. Since those expressions count the same objects, they must be equal to each other and thus the identity is established..

6단원 [내부링크]

정의 곡선의 오목 $\bullet$ 볼록과 변곡점어떤 구간에서 곡선 $y=f(x)$ 위의 임의의 두 점 $\mathrm{A,~B}$에 대하여 $\mathrm{A,~B}$ 사이에 있는 곡선 부분이 항상 선분 $\mathrm{\overline{AB}}$ 보다 아래쪽에 있을 때, 곡선 $y=f(x)$는 이 구간에서 아래로 볼록(convex down)하다고 한다. 반대로 임의의 두 점 $\mathrm{A,~B}$ 사이에 있는 곡선 부분이 항상 선분 $\mathrm{\overline{AB}}$ 보다 위쪽에 있을 때, 곡선 $y=f(x)$는 이 구간에서 위로 볼록(convex up)하다고 한다. 또, 곡선 $y=f(x)$ 위의 한 점의 좌우에서 곡선의 오목$\bullet$볼록이 바뀔 때, 이 점을 곡선 $y=f(x..

[수학의 기초] 곡선의 볼록성 정의(위로볼록,아래로볼록), 이계도함수 [내부링크]

정의 곡선의 오목 $\bullet$ 볼록과 변곡점어떤 구간에서 곡선 $y=f(x)$ 위의 임의의 두 점 $\mathrm{A,~B}$에 대하여 $\mathrm{A,~B}$ 사이에 있는 곡선 부분이 항상 선분 $\mathrm{\overline{AB}}$ 보다 아래쪽에 있을 때, 곡선 $y=f(x)$는 이 구간에서 아래로 볼록(convex down)하다고 한다. 반대로 임의의 두 점 $\mathrm{A,~B}$ 사이에 있는 곡선 부분이 항상 선분 $\mathrm{\overline{AB}}$ 보다 위쪽에 있을 때, 곡선 $y=f(x)$는 이 구간에서 위로 볼록(convex up)하다고 한다. 또, 곡선 $y=f(x)$ 위의 한 점의 좌우에서 곡선의 오목$\bullet$볼록이 바뀔 때, 이 점을 곡선 $y=f(x..

[이항분포] 이항분포 평균 분산 증명 [내부링크]

정의 이항분포 어떤 시행에서 사건 $\mathrm E$가 일어날 확률을 $p$라고 하자. 이 시행을 독립적으로 $n$회 반복할 때, 사건 $\mathrm E$가 일어나는 횟수를 확률변수 $X$라고 하면 $X$의 확률질량함수는 $$\mathrm P (X=r)={}_n \mathrm C _r p^r q^{n-r} ~~(단, ~q=1-p,~r=0,~1,~2,~\cdots,~n)$$ 이고, $X$의 확률분포를 표로 나타내면 다음과 같다. $X$ $0$ $1$ $2$ $\cdots$ $r$ $\cdots$ $n$ $\mathrm P (X=r)$ ${}_n \mathrm C _0 p^0 q^n$ ${}_n \mathrm C _1 p^1 q^{n-1}$ ${}_n \mathrm C _2 p^2 q^{n-2}$ $\c..

더플러스수학학원 문제풀이 영상 제공및 안내 [내부링크]

수능모의고사, 서울대 심층면접, 수리논술 동영상 제공 울산 남구 옥동에 위치한 더플러스수학학원의 블로그는 수능, 교육청모의고사, 삼사, 경찰대 풀이 동영상, 서울대, 카이스트 등 명문대 심층면접문제, 수리논술문제 풀이 동영상 제공하여 자기주도적 학습을 할 수 있게 한 블로그입니다. 2019/10/18 - [수능 모의고사] - [교육청 기출] 2019년 가형 10월 21번 [킬러문항] [교육청 기출] 2019년 가형 10월 21번 [킬러문항] https://tv.kakao.com/v/403026164 [교육청 기출] 2019년 가형 10월 21번 [킬러문항] 정수 $ n $에 대하여 점 $ \left ( a,0 \right ) $에서 곡선 $ y= ( x-n)e ^ {x} $에 그은 접선의 개수를 $ f ..

5단원 [내부링크]

모든 실수 $x$에 대하여 $$f(-x)=f(x)$$를 만족하는 함수 $f(x)$를 우함수(even-function), $$f(-x)=-f(x)$$를 만족하는 함수 $f(x)$를 기함수(odd-function)이라 한다. "임의의 함수 $f(x)$는 다음과 같이 우함수와 기함수의 합으로 나타낼 수 있고 그 표현 방식은 유일하다." $$f(x)=\frac{f(x)+f(-x)}{2}+\frac{f(x)-f(-x)}{2}$$ 정답 및 풀이를 보려면 아래를 클릭하세요. 더보기 증명 $E(x)$를 우함수, $O(x)$를 기함수라 하자. 그러면 $$E(-x)=E(x),~~O(-x)=-O(x)$$ 이제 $f(x)$가 우함수와 기함수의 합으로 나타낼 수 있다고 하자. 즉 $$f(x)=E(x)+O(x)~\cdots\cd..

4단원 [내부링크]

모든 실수 $x$에 대하여 $$f(-x)=f(x)$$를 만족하는 함수 $f(x)$를 우함수(even-function), $$f(-x)=-f(x)$$를 만족하는 함수 $f(x)$를 기함수(odd-function)이라 한다. "임의의 함수 $f(x)$는 다음과 같이 우함수와 기함수의 합으로 나타낼 수 있고 그 표현 방식은 유일하다." $$f(x)=\frac{f(x)+f(-x)}{2}+\frac{f(x)-f(-x)}{2}$$ 정답 및 풀이를 보려면 아래를 클릭하세요. 더보기 증명 $E(x)$를 우함수, $O(x)$를 기함수라 하자. 그러면 $$E(-x)=E(x),~~O(-x)=-O(x)$$ 이제 $f(x)$가 우함수와 기함수의 합으로 나타낼 수 있다고 하자. 즉 $$f(x)=E(x)+O(x)~\cdots\cd..

3단원 [내부링크]

모든 실수 $x$에 대하여 $$f(-x)=f(x)$$를 만족하는 함수 $f(x)$를 우함수(even-function), $$f(-x)=-f(x)$$를 만족하는 함수 $f(x)$를 기함수(odd-function)이라 한다. "임의의 함수 $f(x)$는 다음과 같이 우함수와 기함수의 합으로 나타낼 수 있고 그 표현 방식은 유일하다." $$f(x)=\frac{f(x)+f(-x)}{2}+\frac{f(x)-f(-x)}{2}$$ 정답 및 풀이를 보려면 아래를 클릭하세요. 더보기 증명 $E(x)$를 우함수, $O(x)$를 기함수라 하자. 그러면 $$E(-x)=E(x),~~O(-x)=-O(x)$$ 이제 $f(x)$가 우함수와 기함수의 합으로 나타낼 수 있다고 하자. 즉 $$f(x)=E(x)+O(x)~\cdots\cd..

2단원 [내부링크]

모든 실수 $x$에 대하여 $$f(-x)=f(x)$$를 만족하는 함수 $f(x)$를 우함수(even-function), $$f(-x)=-f(x)$$를 만족하는 함수 $f(x)$를 기함수(odd-function)이라 한다. "임의의 함수 $f(x)$는 다음과 같이 우함수와 기함수의 합으로 나타낼 수 있고 그 표현 방식은 유일하다." $$f(x)=\frac{f(x)+f(-x)}{2}+\frac{f(x)-f(-x)}{2}$$ 정답 및 풀이를 보려면 아래를 클릭하세요. 더보기 증명 $E(x)$를 우함수, $O(x)$를 기함수라 하자. 그러면 $$E(-x)=E(x),~~O(-x)=-O(x)$$ 이제 $f(x)$가 우함수와 기함수의 합으로 나타낼 수 있다고 하자. 즉 $$f(x)=E(x)+O(x)~\cdots\cd..

1단원 [내부링크]

모든 실수 $x$에 대하여 $$f(-x)=f(x)$$를 만족하는 함수 $f(x)$를 우함수(even-function), $$f(-x)=-f(x)$$를 만족하는 함수 $f(x)$를 기함수(odd-function)이라 한다. "임의의 함수 $f(x)$는 다음과 같이 우함수와 기함수의 합으로 나타낼 수 있고 그 표현 방식은 유일하다." $$f(x)=\frac{f(x)+f(-x)}{2}+\frac{f(x)-f(-x)}{2}$$ 정답 및 풀이를 보려면 아래를 클릭하세요. 더보기 증명 $E(x)$를 우함수, $O(x)$를 기함수라 하자. 그러면 $$E(-x)=E(x),~~O(-x)=-O(x)$$ 이제 $f(x)$가 우함수와 기함수의 합으로 나타낼 수 있다고 하자. 즉 $$f(x)=E(x)+O(x)~\cdots\cd..

[우함수 기함수] 함수 우함수와 기함수의 합으로 표현할 수 있다. [내부링크]

모든 실수 $x$에 대하여 $$f(-x)=f(x)$$를 만족하는 함수 $f(x)$를 우함수(even-function), $$f(-x)=-f(x)$$를 만족하는 함수 $f(x)$를 기함수(odd-function)이라 한다. "임의의 함수 $f(x)$는 다음과 같이 우함수와 기함수의 합으로 나타낼 수 있고 그 표현 방식은 유일하다." $$f(x)=\frac{f(x)+f(-x)}{2}+\frac{f(x)-f(-x)}{2}$$ 정답 및 풀이를 보려면 아래를 클릭하세요. 더보기 증명 $E(x)$를 우함수, $O(x)$를 기함수라 하자. 그러면 $$E(-x)=E(x),~~O(-x)=-O(x)$$ 이제 $f(x)$가 우함수와 기함수의 합으로 나타낼 수 있다고 하자. 즉 $$f(x)=E(x)+O(x)~\cdots\cd..

[교육청 기출] 2019년 가형 10월 21번 [킬러문항] [내부링크]

https://tv.kakao.com/v/403026164 [교육청 기출] 2019년 가형 10월 21번 [킬러문항] 정수 $ n $에 대하여 점 $ \left ( a,0 \right ) $에서 곡선 $ y= ( x-n)e ^ {x} $에 그은 접선의 개수를 $ f ( n) $이라 하자. 에서 옳은 것만을 있는 대로 고른 것은? [4점] ㄱ. $ a=0 $일 때, $ f ( 4)=1 $이다. ㄴ. $ f ( n)=1 $인 정수 $ n $의 개수가 $ 1 $인 정수 $ a $가 존재한다. ㄷ. $ \sum\limits _ {n=1} ^ {5} f ( n)=5 $를 만족시키는 정수 $ a $의 값은 $ -1 $ 또는 $ 3 $이다. ① ㄱ ② ㄱ, ㄴ ③ ㄱ, ㄷ ④ ㄴ, ㄷ ⑤ ㄱ, ㄴ, ㄷ 정답 및 풀이..

물리학교수가 들려주는 <수학과 물리학을 잘하는 몇 가지 방법> [내부링크]

김현철 교수님의 수학 물리학 잘하는 법 개인적인 경험 속에서 체득하신 방법입니다. 학생들에게 도움이 되기를.... 쓰다보니 많이 길어졌는데, 혹시 학생들에게 도움이 될까 해서 올립니다. 이론물리학을 30년 넘게 했으니 수학과 물리학을 어떻게 공부해야 하는지는 학생들에게 조금은 이야기해줄 수 있게 되었다. 하지만 이렇게 이야기하는 나도 처음부터 수학과 물리학이 쉬웠던 것은 아니었다. 수학과 물리학을 어떻게 하면 잘할 수 있느냐라는 이야기를 하기 전에 내 이야기부터 하면 수학을 못한다고 좌절하는 사람에게 조금은 꿈을 줄 수 있겠다는 생각이 들었다. 중학교 3학년 때까지는 물리학에 관심이 있어 과학반 활동도 열심히 했고, 공부도 잘하는 축에 들었던 학생이었다. 초등학교 때는 공부를 무척 잘해서 학생 대표로 상..

[교육청 기출] 2019년 가형 10월 30번 [킬러문항] [내부링크]

https://tv.kakao.com/v/403026179 2019년 가형 10월 30번 [킬러문항] 실수 전체의 집합에서 미분가능한 두 함수 $ f ( x) $, $ g ( x) $가 모든 실수 $ x $에 대하여 다음 조건을 만족시킨다. (가) $ g \left ( x+1 \right ) -g ( x)= - \pi \left ( e+1 \right ) e ^ {x} \sin \left ( \pi x \right ) $ (나) $ g ( x+1)= \int _ {0} ^ {x} {\left\{ f \left ( t+1 \right ) e ^ {t} -f ( t)e ^ {t} +g ( t) \right\} } dt $ $ \int _ {0} ^ {1} {f ( x)} dx= \frac {10} {9} e..

2019년 가형 10월 29번 [킬러문항] [내부링크]

https://tv.kakao.com/v/402999340 2019년 가형 10월 29번 좌표공간의 세 점 $ \rm A \left ( -1,~0,~6 \right ) $, $ \rm B \left ( 2,~- \sqrt {3} ,~0 \right ) $, $ \rm C \left ( 3,~0,~0 \right ) $에 대하여 두 점 $ \rm P $, $ \rm Q $가 $$ \left | \overrightarrow {\mathrm{ AP}} \right | =2 ,~~\rm \left | \overrightarrow {\mathrm{ CQ}} \right | =2 \sqrt {3} ,~~ \rm \overrightarrow {\mathrm{ BC}} \cdot \overrightarrow {\mat..

8-3. 위상수학자 sin함수와 연속성 [내부링크]

다음 함수의 $ x=0 $에서의 연속성과 미분가능성을 조사하여라. (1) $ f \left ( x \right ) = \root {3} \of {x ^ {2} } $ (2) $$ f \left ( x \right ) = { \begin {cases} x\sin \frac {1} {x} & & \left ( x \neq 0 \right )\\0 & & \left ( x=0 \right )\end {cases} } $$ 정답 및 풀이를 보려면 아래를 클릭하세요. (1) 연속, 미분불가능 (2) 연속, 미분불가능 8-3. $ f \left ( x \right ) = { \begin {cases} x ^ {2} \sin \frac {1} {x} & & \left ( x \neq 0 \right )\\0 & & ..

미적분 기초-열린구간, 닫힌구간 [내부링크]

구간의 정의 두 실수 $a,~b~(a

[2018 수능 기출] 2018학년도 수능 나형 29번 [내부링크]

https://tv.kakao.com/v/402947930 [2018 수능 기출] 2018학년도 수능 나형 29번 두 실수 $ a $와 $ k $에 대하여 두 함수 $ f \left ( x \right ) $와 $ g \left ( x \right ) $는 $$ \begin{align} f \left ( x \right ) &= \begin {cases} 0~ & \left ( x \leq a \right )\\ \left ( x-1 \right ) ^ {2} \left ( 2x+1 \right ) ~~ & \left ( x>a \right )\end {cases} ,\\ g \left ( x \right ) &= \begin {cases} 0 & \left ( x \leq k \right )\\12 \..

[근과 계수와의 관계 응용 1] 이차함수와 직선의 교점 구한다 [내부링크]

이차함수 $y=ax^2 +bx+c$와 직선 $y=mx+n$에서 $y$를 소거한 방정식 $$ax^2 +bx+c=mx+n$$의 근을 구하는 것은 "두 함수의 교점의 $x$좌표를 구하는 것"이다. 그럼 아래의 한양대 모의 논술문제 (가), (나)에서 "기주가 봉착하게 되는 모순이 어떤 오류"때문인가? 이것에 대한 호기심에서 이 글을 쓰게 된 계기이다. 정사영의 자취방정식을 구하는 것 결론은 "이차방정식의 해를 구한다는 말의 의미가 두 함수의 교점을 $x$축에 정사영시킨 점의 자취를 구한다"는 것이다. 공통부분을 $ x $-평면에 정사영한 도형의 방정식이다. 즉, 교점을 $x$축에 수선의 발을 내린 점의 자취방정식이다. 우선 아래의 문제를 풀면서 시작하자. 2012학년도 한양대 수시2 다음 제시문을 읽고 물음에..

[삼사관학교 기출] 2016학년도 A 삼사 21번 [내부링크]

더보기 https://tv.kakao.com/v/402947937 최고차항의 계수가 $ 1 $인 삼차함수 $ f ( x) $에 대하여 곡선 $ y=f ( x) $가 $ y $축과 만나는 점을 $ \rm A $라 하자. 곡선 $ y=f ( x) $ 위의 점 $ \rm A $에서의 접선을 $ l $이라 할 때, 직선 $ l $이 곡선 $ y=f ( x) $와 만나는 점 중에서 $ \rm A $가 아닌 점을 $ \rm B $라 하자. 또, 곡선 $ y=f ( x) $ 위의 점 $ \rm B $에서의 접선을 $ m $이라 할 때, 직선 $ m $이 곡선 $ y=f ( x) $와 만나는 점 중에서 $ \rm B $가 아닌 점을 $ \rm C $라 하자. 두 직선 $ l $, $ m $이 서로 수직이고 직선 $ m..

4-24 [내부링크]

$2$보다 큰 실수 $a$에 대하여 두 곡선 $y=\log_2 ~x,~y=\log_2 ~(a-x)$가 $x$축과 만나는 점을 각각 $\mathrm {A,~B}$라고 하고, 두 곡선이 만나는 점을 $\mathrm C$라고 하자. 직선 $\mathrm{AC}$의 기울기를 $f(a)$, 직선 $\mathrm{BC}$의 기울기를 $g(a)$라고 할 때, $$\lim\limits _{a \rightarrow 2+} \left\{ f(a)-g(a)\right\}$$의 값을 구하여라. https://tv.naver.com/v/10425688 ㅅㄹㅁㅈㅂ-4-11 더플러스수학 tv.naver.com 정답 및 풀이를 보려면 아래를 클릭하세요.

4-20 [내부링크]

$2$보다 큰 실수 $a$에 대하여 두 곡선 $y=\log_2 ~x,~y=\log_2 ~(a-x)$가 $x$축과 만나는 점을 각각 $\mathrm {A,~B}$라고 하고, 두 곡선이 만나는 점을 $\mathrm C$라고 하자. 직선 $\mathrm{AC}$의 기울기를 $f(a)$, 직선 $\mathrm{BC}$의 기울기를 $g(a)$라고 할 때, $$\lim\limits _{a \rightarrow 2+} \left\{ f(a)-g(a)\right\}$$의 값을 구하여라. https://tv.naver.com/v/10425688 ㅅㄹㅁㅈㅂ-4-11 더플러스수학 tv.naver.com 정답 및 풀이를 보려면 아래를 클릭하세요.

4-18 [내부링크]

$2$보다 큰 실수 $a$에 대하여 두 곡선 $y=\log_2 ~x,~y=\log_2 ~(a-x)$가 $x$축과 만나는 점을 각각 $\mathrm {A,~B}$라고 하고, 두 곡선이 만나는 점을 $\mathrm C$라고 하자. 직선 $\mathrm{AC}$의 기울기를 $f(a)$, 직선 $\mathrm{BC}$의 기울기를 $g(a)$라고 할 때, $$\lim\limits _{a \rightarrow 2+} \left\{ f(a)-g(a)\right\}$$의 값을 구하여라. https://tv.naver.com/v/10425688 ㅅㄹㅁㅈㅂ-4-11 더플러스수학 tv.naver.com 정답 및 풀이를 보려면 아래를 클릭하세요.

4-16 [내부링크]

$2$보다 큰 실수 $a$에 대하여 두 곡선 $y=\log_2 ~x,~y=\log_2 ~(a-x)$가 $x$축과 만나는 점을 각각 $\mathrm {A,~B}$라고 하고, 두 곡선이 만나는 점을 $\mathrm C$라고 하자. 직선 $\mathrm{AC}$의 기울기를 $f(a)$, 직선 $\mathrm{BC}$의 기울기를 $g(a)$라고 할 때, $$\lim\limits _{a \rightarrow 2+} \left\{ f(a)-g(a)\right\}$$의 값을 구하여라. https://tv.naver.com/v/10425688 ㅅㄹㅁㅈㅂ-4-11 더플러스수학 tv.naver.com 정답 및 풀이를 보려면 아래를 클릭하세요.

4-12 [내부링크]

$2$보다 큰 실수 $a$에 대하여 두 곡선 $y=\log_2 ~x,~y=\log_2 ~(a-x)$가 $x$축과 만나는 점을 각각 $\mathrm {A,~B}$라고 하고, 두 곡선이 만나는 점을 $\mathrm C$라고 하자. 직선 $\mathrm{AC}$의 기울기를 $f(a)$, 직선 $\mathrm{BC}$의 기울기를 $g(a)$라고 할 때, $$\lim\limits _{a \rightarrow 2+} \left\{ f(a)-g(a)\right\}$$의 값을 구하여라. https://tv.naver.com/v/10425688 ㅅㄹㅁㅈㅂ-4-11 더플러스수학 tv.naver.com 정답 및 풀이를 보려면 아래를 클릭하세요.

[더플러스수학]2019학년도 1학년 1학기 중간고사 [내부링크]

$2$보다 큰 실수 $a$에 대하여 두 곡선 $y=\log_2 ~x,~y=\log_2 ~(a-x)$가 $x$축과 만나는 점을 각각 $\mathrm {A,~B}$라고 하고, 두 곡선이 만나는 점을 $\mathrm C$라고 하자. 직선 $\mathrm{AC}$의 기울기를 $f(a)$, 직선 $\mathrm{BC}$의 기울기를 $g(a)$라고 할 때, $$\lim\limits _{a \rightarrow 2+} \left\{ f(a)-g(a)\right\}$$의 값을 구하여라. https://tv.naver.com/v/10425688 ㅅㄹㅁㅈㅂ-4-11 더플러스수학 tv.naver.com 정답 및 풀이를 보려면 아래를 클릭하세요.

4-11 [내부링크]

$2$보다 큰 실수 $a$에 대하여 두 곡선 $y=\log_2 ~x,~y=\log_2 ~(a-x)$가 $x$축과 만나는 점을 각각 $\mathrm {A,~B}$라고 하고, 두 곡선이 만나는 점을 $\mathrm C$라고 하자. 직선 $\mathrm{AC}$의 기울기를 $f(a)$, 직선 $\mathrm{BC}$의 기울기를 $g(a)$라고 할 때, $$\lim\limits _{a \rightarrow 2+} \left\{ f(a)-g(a)\right\}$$의 값을 구하여라. https://tv.naver.com/v/10425688 ㅅㄹㅁㅈㅂ-4-11 더플러스수학 tv.naver.com 정답 및 풀이를 보려면 아래를 클릭하세요.

4-9 [내부링크]

https://tv.kakao.com/v/402897248 [경찰대 기출] 2018학년도 경찰대 20번 미분가능한 함수 $ f ( x) $, $ g ( x) $가 \begin{align} f ( x+y)=f ( x)g ( y)+f ( y)g ( x),~~f ( 1)=1 \end{align} \begin{align} g ( x+y)=g ( x)g ( y)+f ( x)f ( y),~~\lim\limits _ {x \rightarrow 0} {} \frac {g ( x)-1} {x} =0 \end{align} 을 만족시킬 때, 옳은 것만을 에서 있는 대로 고른것은? [5점] ㄱ. $ f ' ( x)=f ' ( 0)g ( x) $ ㄴ. $ g ( x) $는 $ x=0 $에서 극솟값 1을 갖는다. ㄷ. $ \le..

4-8 [내부링크]

https://tv.kakao.com/v/402897248 [경찰대 기출] 2018학년도 경찰대 20번 미분가능한 함수 $ f ( x) $, $ g ( x) $가 \begin{align} f ( x+y)=f ( x)g ( y)+f ( y)g ( x),~~f ( 1)=1 \end{align} \begin{align} g ( x+y)=g ( x)g ( y)+f ( x)f ( y),~~\lim\limits _ {x \rightarrow 0} {} \frac {g ( x)-1} {x} =0 \end{align} 을 만족시킬 때, 옳은 것만을 에서 있는 대로 고른것은? [5점] ㄱ. $ f ' ( x)=f ' ( 0)g ( x) $ ㄴ. $ g ( x) $는 $ x=0 $에서 극솟값 1을 갖는다. ㄷ. $ \le..

4-7 [내부링크]

https://tv.kakao.com/v/402897248 [경찰대 기출] 2018학년도 경찰대 20번 미분가능한 함수 $ f ( x) $, $ g ( x) $가 \begin{align} f ( x+y)=f ( x)g ( y)+f ( y)g ( x),~~f ( 1)=1 \end{align} \begin{align} g ( x+y)=g ( x)g ( y)+f ( x)f ( y),~~\lim\limits _ {x \rightarrow 0} {} \frac {g ( x)-1} {x} =0 \end{align} 을 만족시킬 때, 옳은 것만을 에서 있는 대로 고른것은? [5점] ㄱ. $ f ' ( x)=f ' ( 0)g ( x) $ ㄴ. $ g ( x) $는 $ x=0 $에서 극솟값 1을 갖는다. ㄷ. $ \le..

4-6 [내부링크]

https://tv.kakao.com/v/402897248 [경찰대 기출] 2018학년도 경찰대 20번 미분가능한 함수 $ f ( x) $, $ g ( x) $가 \begin{align} f ( x+y)=f ( x)g ( y)+f ( y)g ( x),~~f ( 1)=1 \end{align} \begin{align} g ( x+y)=g ( x)g ( y)+f ( x)f ( y),~~\lim\limits _ {x \rightarrow 0} {} \frac {g ( x)-1} {x} =0 \end{align} 을 만족시킬 때, 옳은 것만을 에서 있는 대로 고른것은? [5점] ㄱ. $ f ' ( x)=f ' ( 0)g ( x) $ ㄴ. $ g ( x) $는 $ x=0 $에서 극솟값 1을 갖는다. ㄷ. $ \le..

4-4 [내부링크]

https://tv.kakao.com/v/402897248 [경찰대 기출] 2018학년도 경찰대 20번 미분가능한 함수 $ f ( x) $, $ g ( x) $가 \begin{align} f ( x+y)=f ( x)g ( y)+f ( y)g ( x),~~f ( 1)=1 \end{align} \begin{align} g ( x+y)=g ( x)g ( y)+f ( x)f ( y),~~\lim\limits _ {x \rightarrow 0} {} \frac {g ( x)-1} {x} =0 \end{align} 을 만족시킬 때, 옳은 것만을 에서 있는 대로 고른것은? [5점] ㄱ. $ f ' ( x)=f ' ( 0)g ( x) $ ㄴ. $ g ( x) $는 $ x=0 $에서 극솟값 1을 갖는다. ㄷ. $ \le..

4-2 [내부링크]

https://tv.kakao.com/v/402897248 [경찰대 기출] 2018학년도 경찰대 20번 미분가능한 함수 $ f ( x) $, $ g ( x) $가 \begin{align} f ( x+y)=f ( x)g ( y)+f ( y)g ( x),~~f ( 1)=1 \end{align} \begin{align} g ( x+y)=g ( x)g ( y)+f ( x)f ( y),~~\lim\limits _ {x \rightarrow 0} {} \frac {g ( x)-1} {x} =0 \end{align} 을 만족시킬 때, 옳은 것만을 에서 있는 대로 고른것은? [5점] ㄱ. $ f ' ( x)=f ' ( 0)g ( x) $ ㄴ. $ g ( x) $는 $ x=0 $에서 극솟값 1을 갖는다. ㄷ. $ \le..

4-1 [내부링크]

https://tv.kakao.com/v/402897248 [경찰대 기출] 2018학년도 경찰대 20번 미분가능한 함수 $ f ( x) $, $ g ( x) $가 \begin{align} f ( x+y)=f ( x)g ( y)+f ( y)g ( x),~~f ( 1)=1 \end{align} \begin{align} g ( x+y)=g ( x)g ( y)+f ( x)f ( y),~~\lim\limits _ {x \rightarrow 0} {} \frac {g ( x)-1} {x} =0 \end{align} 을 만족시킬 때, 옳은 것만을 에서 있는 대로 고른것은? [5점] ㄱ. $ f ' ( x)=f ' ( 0)g ( x) $ ㄴ. $ g ( x) $는 $ x=0 $에서 극솟값 1을 갖는다. ㄷ. $ \le..

3-40 [내부링크]

https://tv.kakao.com/v/402897248 [경찰대 기출] 2018학년도 경찰대 20번 미분가능한 함수 $ f ( x) $, $ g ( x) $가 \begin{align} f ( x+y)=f ( x)g ( y)+f ( y)g ( x),~~f ( 1)=1 \end{align} \begin{align} g ( x+y)=g ( x)g ( y)+f ( x)f ( y),~~\lim\limits _ {x \rightarrow 0} {} \frac {g ( x)-1} {x} =0 \end{align} 을 만족시킬 때, 옳은 것만을 에서 있는 대로 고른것은? [5점] ㄱ. $ f ' ( x)=f ' ( 0)g ( x) $ ㄴ. $ g ( x) $는 $ x=0 $에서 극솟값 1을 갖는다. ㄷ. $ \le..

3-35 [내부링크]

https://tv.kakao.com/v/402897248 [경찰대 기출] 2018학년도 경찰대 20번 미분가능한 함수 $ f ( x) $, $ g ( x) $가 \begin{align} f ( x+y)=f ( x)g ( y)+f ( y)g ( x),~~f ( 1)=1 \end{align} \begin{align} g ( x+y)=g ( x)g ( y)+f ( x)f ( y),~~\lim\limits _ {x \rightarrow 0} {} \frac {g ( x)-1} {x} =0 \end{align} 을 만족시킬 때, 옳은 것만을 에서 있는 대로 고른것은? [5점] ㄱ. $ f ' ( x)=f ' ( 0)g ( x) $ ㄴ. $ g ( x) $는 $ x=0 $에서 극솟값 1을 갖는다. ㄷ. $ \le..

3-33 [내부링크]

https://tv.kakao.com/v/402897248 [경찰대 기출] 2018학년도 경찰대 20번 미분가능한 함수 $ f ( x) $, $ g ( x) $가 \begin{align} f ( x+y)=f ( x)g ( y)+f ( y)g ( x),~~f ( 1)=1 \end{align} \begin{align} g ( x+y)=g ( x)g ( y)+f ( x)f ( y),~~\lim\limits _ {x \rightarrow 0} {} \frac {g ( x)-1} {x} =0 \end{align} 을 만족시킬 때, 옳은 것만을 에서 있는 대로 고른것은? [5점] ㄱ. $ f ' ( x)=f ' ( 0)g ( x) $ ㄴ. $ g ( x) $는 $ x=0 $에서 극솟값 1을 갖는다. ㄷ. $ \le..

3-24 [내부링크]

https://tv.kakao.com/v/402897248 [경찰대 기출] 2018학년도 경찰대 20번 미분가능한 함수 $ f ( x) $, $ g ( x) $가 \begin{align} f ( x+y)=f ( x)g ( y)+f ( y)g ( x),~~f ( 1)=1 \end{align} \begin{align} g ( x+y)=g ( x)g ( y)+f ( x)f ( y),~~\lim\limits _ {x \rightarrow 0} {} \frac {g ( x)-1} {x} =0 \end{align} 을 만족시킬 때, 옳은 것만을 에서 있는 대로 고른것은? [5점] ㄱ. $ f ' ( x)=f ' ( 0)g ( x) $ ㄴ. $ g ( x) $는 $ x=0 $에서 극솟값 1을 갖는다. ㄷ. $ \le..

3-23 [내부링크]

https://tv.kakao.com/v/402897248 [경찰대 기출] 2018학년도 경찰대 20번 미분가능한 함수 $ f ( x) $, $ g ( x) $가 \begin{align} f ( x+y)=f ( x)g ( y)+f ( y)g ( x),~~f ( 1)=1 \end{align} \begin{align} g ( x+y)=g ( x)g ( y)+f ( x)f ( y),~~\lim\limits _ {x \rightarrow 0} {} \frac {g ( x)-1} {x} =0 \end{align} 을 만족시킬 때, 옳은 것만을 에서 있는 대로 고른것은? [5점] ㄱ. $ f ' ( x)=f ' ( 0)g ( x) $ ㄴ. $ g ( x) $는 $ x=0 $에서 극솟값 1을 갖는다. ㄷ. $ \le..

3-22 [내부링크]

https://tv.kakao.com/v/402897248 [경찰대 기출] 2018학년도 경찰대 20번 미분가능한 함수 $ f ( x) $, $ g ( x) $가 \begin{align} f ( x+y)=f ( x)g ( y)+f ( y)g ( x),~~f ( 1)=1 \end{align} \begin{align} g ( x+y)=g ( x)g ( y)+f ( x)f ( y),~~\lim\limits _ {x \rightarrow 0} {} \frac {g ( x)-1} {x} =0 \end{align} 을 만족시킬 때, 옳은 것만을 에서 있는 대로 고른것은? [5점] ㄱ. $ f ' ( x)=f ' ( 0)g ( x) $ ㄴ. $ g ( x) $는 $ x=0 $에서 극솟값 1을 갖는다. ㄷ. $ \le..

[경찰대 기출] 2018학년도 경찰대 20번 [내부링크]

https://tv.kakao.com/v/402897248 [경찰대 기출] 2018학년도 경찰대 20번 미분가능한 함수 $ f ( x) $, $ g ( x) $가 \begin{align} f ( x+y)=f ( x)g ( y)+f ( y)g ( x),~~f ( 1)=1 \end{align} \begin{align} g ( x+y)=g ( x)g ( y)+f ( x)f ( y),~~\lim\limits _ {x \rightarrow 0} {} \frac {g ( x)-1} {x} =0 \end{align} 을 만족시킬 때, 옳은 것만을 에서 있는 대로 고른것은? [5점] ㄱ. $ f ' ( x)=f ' ( 0)g ( x) $ ㄴ. $ g ( x) $는 $ x=0 $에서 극솟값 1을 갖는다. ㄷ. $ \le..

[평가원 기출] 2015학년도 나형 6월 30번 [내부링크]

https://tv.kakao.com/v/402843596 [평가원 기출] 2015학년도 나형 6월 30번 실수 전체의 집합에서 미분가능한 함수 $ f ( x) $가 다음 조건을 만족시킨다. (가) 모든 실수 $ x $에 대하여 $ 1 \leq f ' ( x) \leq 3 $이다. (나) 모든 정수 $ n $에 대하여 함수 $ y=f ( x) $의 그래프는점 $ ( 4n,~8n) $, 점 $ ( 4n+1,~8n+2) $, 점 $ ( 4n+2,~8n+5), $ 점 $ ( 4n+3,~8n+7) $을 모두 지난다. (다) 모든 정수 $ k $에 대하여 닫힌구간 $ [2k,~2k+1] $에서 함수 $ f ( x) $의 그래프는 각각 이차함수의 그래프의 일부이다. $ \int _ {3} ^ {6} {f ( x)d..

[연세대 입시] 2020학년도 연세대학교 학생부종합전형 면접 안내 영상 [내부링크]

https://admission.yonsei.ac.kr/seoul/admission/html/counsel/noticeView.asp?BBS_NO=2442&s_type= 연세대학교 입학처 | 입학도우미 | 공지사항 이 브라우저는 재생할 수 없습니다. ※ 학생부종합전형 이외의 면접은 2020학년도 연세대학교 모집요강 86, 87페이지(홈페이지에 업로드된 PDF파일 82, 83페이지에 해당) 참고 admission.yonsei.ac.kr 연세대 학생부 종합전형 면접 안내 영상이 홈페이지에 있어서 올립니다.

[연세대 특기자전형] 2017학년도 연세대 특기자전형 (과학공학 IT명품인재계열) [내부링크]

[문제1] 다음에 설명하는 규칙에 따라 주어진 박스 모양의 빈 칸에 숫자를 채우려고 한다. 규칙1 : 각 층에서 왼쪽에 있는 숫자는 오른쪽에 있는 숫자보다 작다. 규칙2 : 아래층에 있는 모든 숫자는 위층에 있는 숫자보다 작다. 규칙3 : 어떤 박스의 인접한 북동 방향($ \nearrow $)에 박스가 있는 경우 두 박스 안의 숫자의 차이는 $ s $이다. 규칙4 : 어떤 박스의 인접한 북동 방향($ \searrow $)에 박스가 있는 경우 두 박스 안의 숫자의 차이는 $ t $이다. [1-1] $ s=5,~t=4 $일 때, $ 1 $에서 $ 20 $까지의 자연수 중 $ 10 $개를 골라 다음 $ 4 $층 모양의 빈칸에 채우는 경우의 수는 몇 가지인지 설명하시오. [1-2] $ 1 $부터 $ 25 $까지..

[연세대 특기자전형] 2016학년도 연세대 특기자전형 (과학공학인재계열) [내부링크]

[문제1] 다음에 설명하는 규칙에 따라 주어진 박스 모양의 빈 칸에 숫자를 채우려고 한다. 규칙1 : 각 층에서 왼쪽에 있는 숫자는 오른쪽에 있는 숫자보다 작다. 규칙2 : 각 열에서 위에 있는 숫자는 아래에 있는 숫자보다 작다. [1-1] 다음의 $ 3 \times 3 $ 모양의 박스에 $ 1 $부터 $ 9 $까지의 숫자를 채우려고 할 때, 숫자 $ 5 $가 들어갈 수 잇는 칸을 모두 찾으시오. [1-2] 다음과 같은 $ 9 \times 9 $ 모양의 박스에 $ 1 $부터 $ 81 $까지의 숫자를 채우려고 한다. 아래에 $ \times $로 표시된 $ 7 $행 $ 6 $열의 칸에 들어갈 수 있는 숫자의 최솟값과 최댓값을 구하시오. [1-3] 자연수 $ n $에 대하여 $ \left ( 2n+1 \rig..

2018학년도 1학년 1학기 중간고사 [내부링크]

적분하는 방법은 크게 두 가지로 나눌 수 있다. 첫째, 기본함수(다항함수를 포함하는 $ x^r $($ r $실수)꼴의 함수, 지수함수, 삼각함수, 로그함수)를 적분할 수 있다. 둘째, 기본 함수에 대한 적분법을 알고 있을 때, 합성함수의 미분법의 역과정인 치환적분법과 곱미분에서 유도된 부분적분법이 있다. 부분적분의 원리 여기서는 부분적분에 집중하겠다. 부분적분의 원리를 보이면서 이것의 확장된 형태인 “표에 의한 부분적분”-Tabular Integration by Parts)을 고찰하면서 다항한 함수에 적용해 보자. 먼저 부분적분법은 곱미분에서 출발한다. $$ ( uv)' =u'v+uv' $$ $$ u'v= ( uv)'-uv' $$ 양변을 적분하면 적분은 (+), (-)연산에서는 분리할 수 있으므로 $$ ..

[더플러스수학] 부분적분 1 - LIATE 'tabular integration by parts' [내부링크]

적분하는 방법은 크게 두 가지로 나눌 수 있다. 첫째, 기본함수(다항함수를 포함하는 $ x^r $($ r $실수)꼴의 함수, 지수함수, 삼각함수, 로그함수)를 적분할 수 있다. 둘째, 기본 함수에 대한 적분법을 알고 있을 때, 합성함수의 미분법의 역과정인 치환적분법과 곱미분에서 유도된 부분적분법이 있다. 부분적분의 원리 여기서는 부분적분에 집중하겠다. 부분적분의 원리를 보이면서 이것의 확장된 형태인 “표에 의한 부분적분”-Tabular Integration by Parts)을 고찰하면서 다항한 함수에 적용해 보자. 먼저 부분적분법은 곱미분에서 출발한다. $$ ( uv)' =u'v+uv' $$ $$ u'v= ( uv)'-uv' $$ 양변을 적분하면 적분은 (+), (-)연산에서는 분리할 수 있으므로 $$ ..

[2022학년도 수능 개편안] 문항비율 공통 75%.... [내부링크]

http://news.naver.com/main/read.nhn?mode=LSD&mid=sec&sid1=001&oid=082&aid=0000931888 [2022학년도 수능 개편안] 문항비율 공통 75%·선택 25% 확정… 제2외국어·한문 절대평가 사진은 2019년 5월 부산진로진학지원센터에서 열린 부산지역 대학 입시설명회 모습. 부산일보DB 교육부가 12일 발표한 ‘2022학년도 대학수학능력시험 기본 계획’의 핵심은 국어와 수학영역의 문항 구성을 확정한 것이 news.naver.com

[연세대 수리논술]2018학년도 연세대 수리논술 [내부링크]

https://tv.kakao.com/v/402751657 [연세대 수리논술]2018학년도 연세대 수리논술 ※다음 제시문을 읽고 아래 질문에 답하시오. [제시문 1] 좌표평면 위의 세 점 $ ( 1,~0),~ ( 0,~1),~ ( -1,~0) $을 꼭짓점으로 하는 삼각형이 주어져 있다. 타원 $$ \frac{x^2}{a^2}+\frac{(y-b)^2}{b^2} =1 $$ 은 주어진 삼각형에 내접해 있다. 이 타원의 넓이는 $ \pi ab $이다. [1-1] 제시문의 조건을 만족하는 $ a $와 $ b $의 관계식과 범위를 구하시오.[5점] [1-2] 타원의 넓이가 최대가 되도록 하는 $ b $의 값을 구하시오.[5점] [1-3] 타원의 넓이가 $ \frac {3} {16} \pi $가 되도록 하는 $ a..

[연세대 특기자 전형] 2019학년도 연세대 특기자 전형(과학인재,IT명품인재) [내부링크]

https://tv.kakao.com/v/401199719 [연세대 특기자 전형] 2019학년도 연세대 특기자 전형(과학인재,IT명품인재) [문제1] 다음을 만족하는 집합에 대하여 물음에 답하시오. (1) $ A,~B,~C $는 각각 집합 $ \left\{ 1,~2,~3,~4,~5,~6 \right\} $의 부분집합이고, $ A \cup B \cup C= \left\{ 1,~2,~3,~4,~5,~6 \right\} $이다. (2) $ D $와 $ E $는 각각 집합 $ \left\{ 7,~8,~9 \right\} $의 부분집합이고, $ D \cup E= \left\{ 7,~8,~9 \right\} $이다. (3) $ n ( A \cap B)=2 $, $ n ( A \cap C)=1 $ (4) $ n ( ..

[평가원기출] 2007학년도 가형 9월 12번 [내부링크]

# 정적분의 정의, 구분구적법에 대한 정확한 이해, 좌종점 합, 우종점 합, 리만합 등등에 대해 반드시 알아야 할 핵심문제임. https://tv.kakao.com/v/402658579 [평가원기출] 2007학년도 가형 9월 12번 함수 $ f ( x)=x ^ {2} $에 대하여 그림과 같이 구간 $ \left [ 0,~1 \right ] $을 $ 2n $등분한 후, 구간 $ \left [ \frac {k-1 _ {} } {2n ^ {} } ,~ \frac {k _ {} } {2n ^ {} } \right ] $를 밑변으로 하고 높이가 $ f \left ( \frac {k _ {} } {2n ^ {} } \right ) $인 직사각형의 넓이를 $ S _ {k} $라 하자. (단, $ n $은 자연수이고 ..

[평가원기출] 2011학년도 가형 9월 11번 [내부링크]

실수 전체의 집합에서 연속인 함수 $ f \left ( x \right ) $가 있다. $ 2 $ 이상인 자연수 $ n $에 대하여 폐구간 $ [0,1] $을 $ n $등분한 각 분점 (양 끝점도 포함)을 차례대로 $ 0=x _ {0} ,~x _ {1} ,~x _ {2} , ~\cdots ,~x _ {n-1} ,~x _ {n} =1 $이라 할 때, 옳은 것만을 에서 있는 대로 고른 것은? [4점][2010년 9월] ㄱ. $ n=2m $ ($ m $은 자연수)이면 $ \sum\limits _ {k=0} ^ {m-1} \frac {f \left ( x _ {2k} \right )} {m} \leq \sum\limits _ {k=0} ^ {n-1} \frac {f \left ( x _ {k} \right )}..

과학고 대입 심층면접 대비 5 [내부링크]

함수 $y=f(x)$와 그 역함수 $y=f^{-1} (x)$가 만난다면 그 교점은 $y=x$ 위에 있다. 이 문제에 대하여 잘 정리된 블로그가 있어서 링크합니다. 참조하세요 http://godingmath.com/invfunc2 역함수의 함정 Ⅱ, 함수와 역함수의 교점 함수 \(f(x)\)와 \(f(x)\)의 역함수 \(g(x)\)의 그래프가 모두 \((a,b)\)를 지날 때, 다음 문장은 참일까요? 거짓일까요? 이 글에서는 함수와 역함수의 교점에 대해 흔히 빠질 수 있는 논리 함정에 대해 이야기 하고, 함수와 역함수의 교점에 대한 중요한 몇가지 성질들에 대해 이야기 합니다.… godingmath.com 여기서는 다음 명제를 증명해 보자. 여러분도 한 번 증명해 보세요. 명제. 함수 $y=f(x)$가 증..

[더플러스수학] 함수와 그 역함수의 교점은 직선 $y=x$ 위에 있다(?) [오개념] [내부링크]

함수 $y=f(x)$와 그 역함수 $y=f^{-1} (x)$가 만난다면 그 교점은 $y=x$ 위에 있다. 이 문제에 대하여 잘 정리된 블로그가 있어서 링크합니다. 참조하세요 http://godingmath.com/invfunc2 역함수의 함정 Ⅱ, 함수와 역함수의 교점 함수 \(f(x)\)와 \(f(x)\)의 역함수 \(g(x)\)의 그래프가 모두 \((a,b)\)를 지날 때, 다음 문장은 참일까요? 거짓일까요? 이 글에서는 함수와 역함수의 교점에 대해 흔히 빠질 수 있는 논리 함정에 대해 이야기 하고, 함수와 역함수의 교점에 대한 중요한 몇가지 성질들에 대해 이야기 합니다.… godingmath.com 여기서는 다음 명제를 증명해 보자. 여러분도 한 번 증명해 보세요. 명제. 함수 $y=f(x)$가 증..

과학고 대입 심층면접 대비 4 [내부링크]

김현철 물리학교수님의 '덧댐'의 중요성 인용합니다. 물리학 뿐 아니라 수학공부에도... 1. 학부 학생들에게 주는 조언 중 중요한 것 하나. 그것은 "덧댐"의 중요성이다. 다른 말로는 땜빵이라고 불러도 되겠다. 학생들은 물리 공부를 하다가 이해가 잘 안 되거나 수학에서 막히면, 자신의 기초실력이 부족함을 절감((切感)하며 다시 저학년이나 고등학교 때 배운 과정으로 되돌아간다. 그러나 그건 그다지 현명한 방법이 아니다. 2학년 때 역학을 배우다가 막히면, 1학년 때 일반물리학이나 미적분학으로 되돌아가 처음부터 다시 그 책을 보기 시작한다. 그러나 역학 진도는 정신 없이 나가니 결국 뒤로 돌아간 학생은 진도를 따라가지 못하고 더 좌절할 수 밖에 없다. 여기서 악순환이 시작된다. 2. 그래서 본격적인 기초 공..

덧댐의 법칙 "땜빵"의 중요성 [내부링크]

김현철 물리학교수님의 '덧댐'의 중요성 인용합니다. 물리학 뿐 아니라 수학공부에도... 1. 학부 학생들에게 주는 조언 중 중요한 것 하나. 그것은 "덧댐"의 중요성이다. 다른 말로는 땜빵이라고 불러도 되겠다. 학생들은 물리 공부를 하다가 이해가 잘 안 되거나 수학에서 막히면, 자신의 기초실력이 부족함을 절감((切感)하며 다시 저학년이나 고등학교 때 배운 과정으로 되돌아간다. 그러나 그건 그다지 현명한 방법이 아니다. 2학년 때 역학을 배우다가 막히면, 1학년 때 일반물리학이나 미적분학으로 되돌아가 처음부터 다시 그 책을 보기 시작한다. 그러나 역학 진도는 정신 없이 나가니 결국 뒤로 돌아간 학생은 진도를 따라가지 못하고 더 좌절할 수 밖에 없다. 여기서 악순환이 시작된다. 2. 그래서 본격적인 기초 공..

과학고 대입 심층면접 대비 3 [내부링크]

https://tv.kakao.com/v/402552621 [연세대수리논술] 2008학년도 연세대 수리논술 다음 제시문은 적분의 개념에 관한 것이다. (아래 문제에서 $ x _ {k} =a+k \Delta x $이다.) 적분의 기본 개념 및 원리는 17세기 뉴턴과 라이프니쯔에 의해 독립적으로 체계화되었고, 적분에 관한 엄밀한 수학적 정의는 코오시와 리이만이 극한의 개념을 도입함으로써 완성되었다. 적분의 기본 원리인 구분구적법은 어떤 도형의 넓이나 부피를 구할 때, 그 도형을 여러 개의 간단한 도형으로 세분하여 이들 도형의 넓이나 부피의 합을 구한 후, 이 합의 극한값으로 원래 도형의 넓이나 부피를 구하는 방법이다. 닫힌구간 $ [a,b] $에서 연속인 함수 $ f ( x) $에 대해서 정적분을 다음과 같..

과학고 대입 심층면접 대비 2 [내부링크]

https://tv.kakao.com/v/402552621 [연세대수리논술] 2008학년도 연세대 수리논술 다음 제시문은 적분의 개념에 관한 것이다. (아래 문제에서 $ x _ {k} =a+k \Delta x $이다.) 적분의 기본 개념 및 원리는 17세기 뉴턴과 라이프니쯔에 의해 독립적으로 체계화되었고, 적분에 관한 엄밀한 수학적 정의는 코오시와 리이만이 극한의 개념을 도입함으로써 완성되었다. 적분의 기본 원리인 구분구적법은 어떤 도형의 넓이나 부피를 구할 때, 그 도형을 여러 개의 간단한 도형으로 세분하여 이들 도형의 넓이나 부피의 합을 구한 후, 이 합의 극한값으로 원래 도형의 넓이나 부피를 구하는 방법이다. 닫힌구간 $ [a,b] $에서 연속인 함수 $ f ( x) $에 대해서 정적분을 다음과 같..

[연세대수리논술] 2008학년도 연세대 수리논술 [내부링크]

https://tv.kakao.com/v/402552621 [연세대수리논술] 2008학년도 연세대 수리논술 다음 제시문은 적분의 개념에 관한 것이다. (아래 문제에서 $ x _ {k} =a+k \Delta x $이다.) 적분의 기본 개념 및 원리는 17세기 뉴턴과 라이프니쯔에 의해 독립적으로 체계화되었고, 적분에 관한 엄밀한 수학적 정의는 코오시와 리이만이 극한의 개념을 도입함으로써 완성되었다. 적분의 기본 원리인 구분구적법은 어떤 도형의 넓이나 부피를 구할 때, 그 도형을 여러 개의 간단한 도형으로 세분하여 이들 도형의 넓이나 부피의 합을 구한 후, 이 합의 극한값으로 원래 도형의 넓이나 부피를 구하는 방법이다. 닫힌구간 $ [a,b] $에서 연속인 함수 $ f ( x) $에 대해서 정적분을 다음과 같..

<더플러스수학학원> 찾아오시는 길 [내부링크]

더플러스수학학원 오시는길을 안내해드립니다. 052-260-9981 052-258-4409 https://place.map.kakao.com/1438017220 더플러스수학학원 울산 남구 대공원입구로21번길 45-1 2층 (옥동 258-5) place.map.kakao.com

과학고 심층면접 대비 1 [내부링크]

https://tv.kakao.com/v/402486135 I. 다음 제시문을 읽고 논제에 답하시오. (60점) [가] (1) 좌표평면 위의 한 점 $\mathrm {A} (x_1 ,~y_1 ) $을 지나고 기울기가 $m$인 직선의 방정식은 $$y-y_1 =m(x-x_1 )$$이다. (2) 중심이 $(a,~b)$이고 반지름의 길이가 $r$인 원의 방정식은 $$(x-a)^2 +(y-b)^2 =r^2$$이다. [나] $x$의 값이 $a$보다 크면서 $a$에 한없이 가까워질 때, 함수 $f(x)$의 값이 일정한 값 $a$에 한없이 가까워지는 것을 기호로 $$\lim\limits_{n \rightarrow a+} f(x)=L$$ 과 같이 나타내고, $L$을 $x=a$에서의 함수 $f(x)$의 우극한이라고 한다...

[경희대 수리논술] 2019학년도 경희대 수리논술(일) [내부링크]

https://tv.kakao.com/v/402486135 I. 다음 제시문을 읽고 논제에 답하시오. (60점) [가] (1) 좌표평면 위의 한 점 $\mathrm {A} (x_1 ,~y_1 ) $을 지나고 기울기가 $m$인 직선의 방정식은 $$y-y_1 =m(x-x_1 )$$이다. (2) 중심이 $(a,~b)$이고 반지름의 길이가 $r$인 원의 방정식은 $$(x-a)^2 +(y-b)^2 =r^2$$이다. [나] $x$의 값이 $a$보다 크면서 $a$에 한없이 가까워질 때, 함수 $f(x)$의 값이 일정한 값 $a$에 한없이 가까워지는 것을 기호로 $$\lim\limits_{n \rightarrow a+} f(x)=L$$ 과 같이 나타내고, $L$을 $x=a$에서의 함수 $f(x)$의 우극한이라고 한다...

[고급수학 중간고사] 증명문제 정리 [내부링크]

#더플러스수학, #울산과고 중간고사 대비 고급수학 증명문제 모음 정의. 비특이행렬(non-singular matrix), 가역행렬(invertible matrix) 정규행렬(regular matrix) $ n $차 정사각행렬 $ A $, $ B $에 대하여 $ AB=I _ {n} =BA $를 만족하는 행렬 $ B $가 존재할 때, 행렬 $ A $를 비특이행렬(non-singular matrix) 또는 가역행렬(invertible matrix) 또는 정규행렬(regular matrix)이라 부른다. 또, 행렬 $ B $를 행렬 $ A $의 역행렬이라 부르고 행렬 $ A $가 역행렬을 가지지 않을 때, 행렬 $ A $를 특이행렬(singular matrix)라고 부른다. 1. 단위행렬의 유일성 $ n $차 ..

[고려대면접] 2020학년도 고려대 면접영상 학교장추천 I,II, 일반전형 [내부링크]

#고려대 면접영상,# 학생부종합, https://youtu.be/DuILQKQRvP4

[서울대 심층면접] 2008학년도 서울대 심층면접 [내부링크]

[문제1] (1). 양수 $ a,~b,~c $에 대하여 $$ \lim\limits _ {n \rightarrow \infty } \left ( a ^ {n} +b ^ {n} +c ^ {n} \right ) ^ { \frac {1} {n} } = A $$인 극한값 $ A $를 구하라. (2). $$ \lim\limits _ {n \rightarrow \infty } { \frac {\left ( 1+ \left ( \frac {1} {2} \right ) ^ {n} \right ) ^ { \frac {1} {n} } -1} {\left ( \frac {1} {2} \right ) ^ {n} } =0} $$임을 보이고 $$ \lim\limits _ {n \rightarrow \infty } {~n ^ {k}..

[서울대 심층면접] 2007학년도 심층면접 [내부링크]

[문제1] $$ f _ {\delta } ( x)= { \begin {cases} 1~~ & ( 0 \leq x \leq \delta )\\-1 & ( \delta \leq x \leq 1)\end {cases} } $$ (단, $ 0< \delta 0 $인 경우 $ 0 \leq t \leq 1 $에 원점과 $ \overrightarrow {X} \left ( t \right ) $ 를 잇는 직선 $ \overline {OX \left ( t \right )} $가 휩쓸고 간 영역의 면적$ A $와 곡선 $ \overrightarrow {X} \left ( t \right ) $의 길이 $ L $ 간의 관계식이 $ A= \frac {1} {2} L $임을 보여라. 3-3 $ \frac {d \overri..

[서울대 심층면접] 2006학년도 서울대 심층면접 수시 [내부링크]

[서울대 2006학년도 특기자 수시] [문제1] 함수 $ f: $ R → R 이 $ \lim\limits _ {h \rightarrow 0} {\left\{ f \left ( a+h \right ) -f \left ( a-h \right ) \right\} =0} $을 만족할 때 “$ x=a $에서 대칭연속”이라고 정의하자. 함수 $ f $가 모든 점에서 대칭연속일 때 $ f $를 “대칭연속함수”라고 하자. 한편 다음 극한 $ \lim\limits _ {h \rightarrow 0} { \frac {f \left ( a+h \right ) -f \left ( a-h \right )} {2h} } $가 존재할 때 “$ x=a $에서 대칭미분가능” 하다고 정의하고, 또한 모든 점에서 대칭미분가능하면 함수 $..

[서울대 심층면접] 2005학년도 서울대 심층면접 수시 [내부링크]

[서울대 2005학년도 수시] [문제1] $ x,~a $가 1보다 큰 실수일 때, $ \log _ {a} x $를 다음과 같이 정의할 수도 있다. $$ \log _ {a} x= \frac { \int _ {1} ^ {x} \frac {1} {t} dt} { \int _ {1} ^ {a} \frac {1} {t} dt} $$ (1) 부등식 $$ 0

[서울대 심층면접] 2005학년도 서울대 심층면접 정시 [내부링크]

[서울대 2005학년도 정시] [문제1] 다음 물음에 답하라. (1) 주어진 삼각형 $ \rm ABC $ 내부의 점 $ \rm X $에 대하여 $ \rm \left | AX \right | ^ {2} + \left | BX \right | ^ {2} + \left | CX \right | ^ {2} $이 최소가 되는 $ \rm X $가 삼각형 $ \rm ABC $의 무게중심임을 증명하시오. (2) 세 점 $ \rm A $$ \left ( 0,~a \right ) $, $ \rm B $$ \left ( -1,~0 \right ) $, $ \rm C $$ \left ( 1,~0 \right ) $을 꼭짓점으로 하는 $ \rm \triangle ABC $의 내부의 점 $ \rm X $$ ( m,~t) $에 대..

[서울대 심층면접] 2004학년도 서울대 심층면접 수시 [내부링크]

[서울대 2004학년도 수시] [문제1] 반지름이 1$ cm $이고 중심의 좌표가 $ \left ( 0,1 \right ) $인 원이 있다. 여기서 좌표의 단위는 $ cm $이다. 이 원의 중심이 일정한 속도 $ 2cm/\sec $로 수직방향으로 위로 올라가고 그와 동시에 반지름은 일정한 속도 $ 1cm/\sec $로 커지기 시작했다. 1초 후부터는 중심이 올라가는 속도가 $ \frac {1} {2} cm/\sec $로, 반지름의 변화하는 속도는 0으로 바뀐다. 원이 움직이기 시작한 $ t $초가 지났을 때 중심의 좌표를$ \left ( 0,y \left ( t \right ) \right ) $, 반지름을 $ r \left ( t \right ) cm $, 원점에서 이 원에 그은 두 접선이 이루는 예각..

[서울대 심층면접] 2004학년도 서울대 심층면접 정시 [내부링크]

[문제1] 아래 도형과 같이 가로의 길이, 세로의 길이가 $ 2 \rm cm $인 정사각형이 있다. 동전의 앞면이 나오면 정사각형의 둘레를 화살표 방향으로 $ 2 \rm cm $ 전진하게 되고, 동전의 뒷면이 나온다면 정사각형의 둘레를 화살표의 반대방향으로 $ 1 \rm cm $ 움직이게 된다. 점 $ A $를 출발점으로 하여 동전을 일곱 번 던지고 난 후 점 $ A $에 도착할 확률을 수학적으로 설명하시오. [문제2] 주사위의 각 면에 $ 1,2,3,4 $가 표시되고 각 면이 나올 확률이 동일한 정사면체 형태의 주사위 두 개를 사용하여 던지는 경우를 생각해보자. 이 두 개의 주사위를 던져, 나오는 수의 합을 확률변수 $ X $라고 한다. (1) 확률변수 $ X $의 평균을 구하시오. (2) 이 두 주사..

[서울대 심층면접] 2003학년도 서울대 심층면접 수시 [내부링크]

[서울대 2003학년도 수시] 좌표평면에서 점 $ \rm P $의 좌표 $ ( x,~y) $가 시각 $ t $의 함수로써 다음과 같이 주어진다. $$ x=2\cos \left ( \frac {\pi } {3} t \right ) -\sin \left ( \frac {\pi } {3} t \right ),~ y=2\cos \left ( \frac {\pi } {3} t \right ) +\sin \left ( \frac {\pi } {3} t \right ) $$ (1) $ t=0 $일 때의 점에서 출발하여 다시 출발점으로 돌아오기 위해서는 얼마만큼의 시간이 경과해야 하는지 설명하여라. (2) $ \rm P $가 어느 위치일 때 가장 속력이 큰 지를 설명하여라. (3) 시각 $ t $가 변함에 따라서 $ ..

[서울대 심층면접] 2003학년도 서울대 심층면접 정시 [내부링크]

(1) 세 점 $ \rm P ( 3~,0~,0) $, $ \rm Q ( 0,~6,~0) $, $ \rm R ( 0,~0,~9) $을 지나는 평면의 방정식을 구하시오. (2) 원점 $ \rm O $와 점 $ \rm P,~Q,~R $을 서로 연결해 사면체를 만들었다. 이 때, 사면체 $ \rm OPQR $에 내접하는 구의 반지름을 구하시오. (3) (2)에서 구한 구를 $ S _ {1} $이라고 하자. $ xy $평면에 평행하고 구 $ S _ {1} $에 접하는 평면을 $ \alpha _ {1} $이라 하자. 평면 $ \alpha _ {1} $, $ yz $평면, $ xz $평면, 평면 $ \rm PQR $에 접하는 구를 $ S _ {2} $라 하자. 또, $ xy $평면에 평행하고 구 $ S _ {2} $..

[서울대 심층면접] 2003학년도 서울대 심층면접 정시 [내부링크]

[문제1] 좌표평면 위에 정점 $ \rm A \left ( 0~,1 \right ) ,B \left ( 1~,1 \right ) ,C \left ( 1~,0 \right ) $이 주어져 있다. 동점 $ \rm P $$ \left ( x~,y \right ) $가 평면 위를 움직이는데, 벡터 $ \rm \overrightarrow {OP} $가 $ x $축의 양의 방향과 이루는 각 $ \theta $는 $ 0 \leq \theta \leq \frac {\pi} {2} $를 만족한다. (1) 벡터 $ \rm \overrightarrow {OP} $가 다음 조건을 만족할 때, 점 $ \rm P $$ \left ( x,~y \right ) $의 자취를 구하시오. $ 0 \leq \theta \leq \frac ..

[서울대 심층면접] 2003학년도 서울대 심층면접 [내부링크]

(1) 타원 $ \frac {x ^ {2} } {a ^ {2} } + \frac {y ^ {2} } {b ^ {2} } =1 $ 위의 점 $ \left ( \alpha ,~ \beta \right ) $에서 이 타원에 접하는 직선의 방정식을 말하라.[단답형] (2) 마름모꼴 $ |x|+ \frac {|y|} {2} =1 $에 내접하는 타원 중, 점 $ \left ( \alpha ,~2-2 \alpha \right ) $에서 접하는 타원의 방정식을 구하라. (단, $ 0< \alpha

[서울대 심층면접] 2002학년도 서울대 심층면접 [내부링크]

[문제1] 다음 물음에 답하시오. (1) 위와 같이 검은 공이 떨어지고 있다. 검은 공이 흰 공과 충돌할 때는 같은 확률로 좌우로 갈라지게 된다. 검은 공과 가장 밑의 $5$개의 흰공들이 충돌할 확률은 각각 얼마나 될 지 구하시오. (2) 흰 공이 위 그림에 표시되어 있는 만큼만 있는 것이 아니라, 그 밑에도 흰 공이 무한히 늘어서 있다고 가정하자. 단, 한 줄에 있는 흰공의 수는 한 줄 밑으로 내려갈 때마다 하나씩 늘어난다고 하자. (그림의 제일 밑에 있는 줄에 있는 흰 공의 수는 $5$개이므로, 그 밑의 줄에 있는 흰 공의 개수는 $6$개, 그 다음 줄은 $7$개, $\cdots$ ) 이 때 검은 공이 $n$번 충돌한 뒤 $n+1$번째 줄에서 왼쪽으로부터 번째에 위치한 흰 공과 충돌할 확률은 얼마인가..

[서울대 심층면접] 2001학년도 서울대 심층면접 [내부링크]

$f(t)$는 $[-1,~1]$에서 연속함수이다. $P_n (t)$는 다음과 같이 정의된 함수이다. $$ P_n (t)= \begin{cases} n& \left (- \frac{1}{n} \leq t \leq \frac{1}{n} \right) \\ 0& \left( t < -\frac{1}{n} ,~t > \frac{1}{n} \right)\end{cases}$$ 이 때, $\int_{-1}^{1} f(t)P(t)dt$ 라고 할 때, $\lim\limits_{n \rightarrow \infty} a_n $을 구하여라. [서울대 2001학년도 수시]

[서울대 심층면접] 2001학년도 서울대 심층면접(공대) [내부링크]

다음을 구하시오. (1) 평면에서 3개의 직선으로 구분할 수 있는 평면의 최대 개수는 7개이다. 4개의 직선으로 구분 할 수 있는 평면의 최대 개수는 몇 개인지 설명하여라. (2) $ n $개의 직선으로 구분할 수 있는 최대 개수는 몇인가? (3) 공간에서 3개의 평면으로 구분할 수 있는 공간의 최대 개수는 8개이다. 4개의 평면으로 구분 할 수 있는 최대 개수는 몇인가? (4) 공간에서 $ n $의 평면으로 구분할 수 있는 공간의 최대 개수를 구하여라.

[서울대 본고사] 1995학년도 서울대 본고사(인문계) [내부링크]

실수 $ a $에 대하여 $ $는 $ a $에 가장 가까운 정수와 $ a $ 사이의 거리를 나타낸다. 보기를 들면 $ =0.3 $, $ =0.4 $이다. 음이 아닌 정수 $ n $에 대하여 $$ f _ {n} ( x)= \frac {1} {2 ^ {n} } $$ 이라고 하자. 함수 $ f _ {n} ( x) $는 주기함수임을 보여라. (5점) $ 0 \leq x \leq 1 $인 범위에서 $ y=f _ {n} ( x) $의 그래프와 $ x $축으로 둘러싸인 부분의 넓이를 $ S _ {n} $이라고 할 때, $ \sum\limits _ {n=0} ^ {\infty } S _ {n} $의 값을 구하라.(10점)

[서울대 본고사] 1977학년도 서울대 본고사 [내부링크]

[문제1] ⑴ 평균값의 정리를 쓰라. (10점) ⑵ 실수 $ C _ {0} ,~C _ {1} , \cdots ,~C _ {n} $이 다음 관계식을 만족한다. $$ C _ {0} + \frac {C _ {1} } {2} + \cdots + \frac {C _ {n-1} } {n} + \frac {C _ {n} } {n+1} =0 $$ 이 때, 방정식 $$ C _ {0} +C _ {1~} x+ \cdots +C _ {n-1~} x ^ {n-1} +C _ {n} ~x ^ {n} =0 $$ 이 $ 0 $과 $ 1 $ 사이에 실근을 가짐을 증명하라. (10점)

[서울대 본고사] 1994학년도 서울대 본고사 문제 [내부링크]

[문제1] 1. 자연수 $ l,~m,~n $에 대하여 $ n \leq l,~n \leq m $일 때, $$ \sum\limits _ {k=0} ^ {n} {}_ {l} \mathrm C _ {k} \cdot {} _ {m} \mathrm C _ {n-k} ={} _ {l+m} C _ {n} $$ 임을 증명하여라. 2019/10/24 - [수학과 공부이야기] - 조합론적 증명 2. 어떤 제품이 $ 3M $개 들어있는 상자 속에 불량품이 $ M $개 들어있다. 이 상자에서 동시에 $ n $개의 제품을 임의추출할 때, 추출된 불량품의 개수를 $ X $라고 하자. 단, $ 1 \leq n \leq M $이다. 이 때, $\mathrm P ( X=k) $를 $ P _ {k} $라 하면 $$ \mathrm E ( ..

[서울대심층면접] 서울대심층면접 연도미상 [내부링크]

함수 $ f \left ( x \right ) = \frac {1} {1+e ^ {-x} } $ 일 때, 다음에 답하여라. (1) 도함수 $ f ' \left ( x \right ) $의 최대값을 구하여라. (2) 방정식 $ f \left ( x \right ) =x $은 1개의 실수해만 가짐을 보이시오. (3) 점화식 $$ a _ {n+1} =f \left ( a _ {n} \right )~ \left ( n=1,2,3, \cdots \right ) $$이라고 주어져 있을 때, 수열 $ \left\{ a _ {n} \right\} $은 초항 $ a _ {1} ~\left ( 0

[서울대심층면접] 서울대심층면접 연도미상 [내부링크]

$ xyz- $공간에 주어진 선분 $$ x=t, ~ y=1-t ,~ z=1 $$ 을 $ x $축 주위로 회전시켜 생기는 곡면을 생각해 보자. (1) 이 곡면을 $ xz $평면으로 자른 곡선의 방정식을 구하여라. (2) 이 곡면과 두 평면 $ x=0 $, $ x=2 $로 둘러싸인 입체의 부피를 구하여라.

[서울대심층면접] 서울대심층면접 연도미상 [내부링크]

좌표공간에서 $$ x^2 +y^2 +z^2 =2 , ~z=1 $$ 로 나타내는 원을 생각하자. (1) 이 원의 반지름을 말하여라. (2) 이 원을 $ x $축 둘레로 회전시켜 얻은 도형의 생김새를 설명하여라. (3) 원판 $$ x^2 +y^2 +z^2 \leq 2,~ z=1 $$ 을 $ x $축 둘레로 회전시켜 얻은 입체의 부피를 구하여라.

[서울대심층면접] 서울대심층면접 연도미상 [내부링크]

다음 물음에 답하시오. (1) $ n $을 임의의 양의 정수라 할 때, $ x>0 $이면 부등식 $$ e ^ {x} >1+ \frac {x} {1!} + \frac {x ^ {2} } {2!} + \cdots + \frac {x ^ {n} } {n!} $$ 가 성립한다. 이 사실을 수학적 귀납법을 이용하여 증명하여라. (2) 위의 부등식을 이용하여 주어진 임의의 정수 $ k $에 대하여 $$ \lim\limits _ {x \rightarrow \infty } {} x ^ {k} e ^ {-x} =0 $$ 임을 증명하여라.(단, 로피탈 정리를 쓰면 안된다.)

[서강대수리논술] 2009학년도 서강대 수리논술 [내부링크]

https://tv.kakao.com/v/402304689 다음 제시문을 읽고 물음에 답하라. [가] 우리는 대부분의 경우 사람들과 사회적 관계 속에서 일어나는 여러 가지 사건(현상)을 경험하며 살고 있다. 이러한 관계는 구성원 간의 약속을 통하여 만들어지곤 한다. 약속의 연속이 매일을 이룬다고 하여도 무방할 것이다. 다음에 제시되는 이야기는 흔히 우리가 경험하는 것인데, 이를 과학적으로 접근하여, 호기심을 가질 만한 수치를 얻고 그 의미를 찾아보기로 하자. 진우와 서희는 친구 사이로서 서울의 서로 다른 지역에 거주한다. 일요일인 오늘 진우는 서희에게 전화하여 지하철 신촌역 근처에 있는 서점에 들러 미적분학 교재를 사기로 약속한다. 그들은 만남의 편리함 때문에 자주 이용하던 신촌역에서 만나기로 정한다..

[연세대수리논술] 2017학년도 연세대 수리논술 [내부링크]

https://tv.kakao.com/v/402303861 [연세대수리논술] 2017학년도 연세대 수리논술 ※다음 제시문을 읽고 아래 질문에 답하시오. [제시문 1] [가] 다항함수 $ h ( x) $의 그래프 위의 점 $ ( a,~h ( a)) $에서의 접선의 방정식은 다음과 같다. $$ y=h ' ( a) ( x-a)+h ( a) $$ [나] 다항함수 $ h ( x) $가 $ h ( x)= ( x-a) ^ {n} g ( x) $ (단, $ n $은 자연수이고, $ g ( x) $는 다항함수이다.) 로 나타내어질 때, 방정식 $ h ( x)=0 $은 $ x=a $를 근으로 갖는다고 한다. 특히, $ n \geq 2 $이면 방정식 $ h ( x)=0 $은$ x=a $에서 중근을 갖는다고 한다. [1-1]..

[연세대수리논술] 2011학년도 연세대 수리논술 [내부링크]

https://tv.kakao.com/v/402302472 [연세대수리논술] 2011학년도 연세대 수리논술 다음 제시문을 읽고 아래 질문에 답하시오. [가] 단위원 $ x ^ {2} +y ^ {2} =1 $위를 점 $ \rm A ( 1,~0) $에서에서 출발하여 시계 반대 방향으로 움직이는 점 $ \rm P $의 시각 $ t $에서의 좌표를 $ ( x ( t),~y ( t)) $라고 하자. 타원 $ x ^ {2} +k ^ {2} y ^ {2} =1 $ (단, $ k>1 $인 실수)은 두 점 $ ( 1,~0),~ ( -1,~0) $에서 단위원에 접한다. 점 $ \rm P $에서 $ x $축으로 내린 수선이 타원과 처음 만나는 점을 $ Q $라고 하자. [나] 점 $ \rm P $와 원점 $ \rm O $를..

[시립대수리논술] 2018학년도 시립대 모의논술 [내부링크]

https://tv.kakao.com/v/402301572 [문제 1] (100점) 좌표평면에서 점 $ \rm A ( -2,0) $을 지나는 직선이 원 $ ( x-1) ^ {2} +y ^ {2} =1 $과 제1사분면의 서로 다른 두 점에서 만날 때, 두 점 중에서 점 $ \rm A $에 가까운 점을 $ \rm P $라 하자. $ \angle \rm PAO= \theta $라 할 때, $ \lim\limits _ {\theta \rightarrow 0 ^ {+} } { \frac {\overline {\rm AP \it } -2} {\theta ^ {2} } } $의 값을 구하여라. (단, $ \rm O $는 원점이다.) [문제 2] (100점) 한 변의 길이가 $ 6 $인 정사각형 $ \rm ABCD $..

[킬러문항] 2017학년도 가형 6월 평가원 21번 [더플러스수학] [내부링크]

실수 전체의 집합에서 미분가능한 함수$ f ( x) $가 모든 실수 $ x $에 대하여 다음 조건을 만족시킨다. (가) $ f ( x) \neq 1 $ (나) $ f ( x)+f ( -x)=0 $ (다) $ f ' ( x)= \left\{ 1+f ( x) \right\} \left\{ 1+f ( -x) \right\} $ 에서 옳은 것만을 있는 대로 고른 것은? [4점] [2016년 6월] ㄱ. 모든 실수 $ x $에 대하여 $ f ( x) \neq -1 $이다. ㄴ. 함수 $ f ( x) $는 어떤 열린 구간에서 감소한다. ① ㄱ ② ㄴ ③ ㄱ, ㄷ ④ ㄴ, ㄷ ⑤ ㄱ, ㄴ, ㄷ 정답 및 풀이를 보려면 아래를 클릭하세요. ...더보기 정답 및 풀이 ① ㄱ. (가), (나)에 의하여 $ f \left (..

[킬러문항 21번] 2012년 가형 4월 교육청 21번 [더플러스수학] [내부링크]

https://tv.kakao.com/v/401140813 함수 $ f \left ( x \right ) =\ln \left ( 2x ^ {2} +1 \right ) $에 대하여 옳은 것만을 에서 있는 대로 고른 것은? [4점] [2012년 4월] ㄱ. 모든 실수 $ x $에 대하여 $ f ' ( -x)=-f ' ( x) $이다. ㄴ. $ f \left ( x \right ) $의 도함수 $ f ' \left ( x \right ) $의 최댓값은 $ \sqrt {2} $이다. ㄷ. 임의의 두 실수 $ x _ {1} $, $ x _ {2} $에 대하여 $ \left | f \left ( x _ {1} \right ) -f \left ( x _ {2} \right ) \right | \leq \sqrt {2..

[시립대 수리논술] 2019학년도 시립대 수리논술 [내부링크]

https://tv.kakao.com/v/401918706 [문제 1] (100점) 함수 $$ f ( x)=x ^ {4} + \left ( 6a+2 \right ) x ^ {3} + \left ( 11a ^ {2} +10a+1 \right ) x ^ {2} + \left ( 6a ^ {3} +14a ^ {2} +4a \right ) x+3a ^ {3} +5a ^ {2} +a $$ 에 대하여 다음 물음에 답하여라. (단, $ a $는 상수이다.) (a) $ f ( x)= ( x ^ {2} +Ax+3a ^ {2} +5a+1) \left ( x ^ {2} +Bx+a \right ) $를 만족시키는 $ A,~B $를 $ a $를 사용하여 나타내어라. (20점) (b) 함수 $ f ( x) $의 최솟값을 $ m ..

[인하대 수리논술] 2019학년도 인하대 수리논술 오전 [내부링크]

https://youtu.be/oooGUjpX19s(구독과 좋아요를....) [문제 1] (30점) 다음 제시문을 읽고 물음에 답하시오. (가) 좌표평면 위의 두 점 $ \mathrm {P} ( x _ {1} ,~y _ {1} ),~ \mathrm {Q} ( x _ {2} ,~y _ {2} ) $사이의 거리는 다음과 같다. $$\displaystyle \overline {\mathrm{PQ}}= \sqrt {\left ( x _ {2} -x _ {1} \right ) ^ {2} + \left ( y _ {2} -y _ {1} \right ) ^ {2} } $$ (나) 서로 다른 두 점 $ ( x _ {1} ,~y _ {1} ),~ ( x _ {2} ,~y _ {2} ) $를 지나는 직선의 방정식은 다음과..

[서울대 심층면접] 2003학년도 서울대 심층면접 정시 [내부링크]

[서울대 2003학년도 정시] 이계도함수를 가지는 함수 $ y=f \left ( x \right ) $가 다음 성질을 만족한다고 한다. $$ f \left ( 0 \right ) =0 ,~ f '' \left ( x \right )

[서울대 의대] 2003학년도 서울대의대 심층면접 [내부링크]

[서울대 2003학년도 의대 수시] 다음 물음에 답하여라. (1) 미분가능한 함수 $ y=f ( x) $의 역함수를 $ g $라 하자. 좌표평면에서 점 $ \rm P \it ( a,~b) $는 함수 $ y=f ( x) $를 만족한다. 이 때, $$ g ' ( b)= \frac {1} {f ' ( a)} $$임을 설명하여라. (2) 함수 $ f ( x)=\sin ^ {2} \left ( \frac {\pi } {2} x \right ) $ (단, $ 0

[서울대 심층면접] 2006학년도 서울대 심층면접 [내부링크]

[2006 서울대 심층면접] 원통은 회전하고 $ \overline {AB} $는 페인트 붓이며 직선으로 내려간다. $ A $가 윗면에 있을 때를 기준으로 하여 $ t $초 후의 붓 $ \overline {AB} $가 내려가는 속도를 $ f ( t) \rm mm/\sec $라 하면 $ t $와 $ f ( t) $의 관계 그래프가 아래와 같다. 원통의 회전 각속도는 $\rm rad/sec$를 단위로 하여 $$ \omega(t)= \frac{\pi}{6} \left( f(t) +1 \right)$$ 이다. (1) 붓 $ \overline {AB} $는 같은 곳을 칠하지 않음을 보여라. (2) $ B $가 원통의 가장 아래에 도달했을 때 붓이 칠한 도형의 넓이를 구하여라. https://tv.naver.com/..

[고려대 논술] 2012학년도 고려대 수시 논술(자연계 A) [내부링크]

https://tv.naver.com/v/9942010 [고려대수리논술] 2012학년도 고려대 수리논술 [더플러스수학] 더플러스수학 | [고려대수리논술] 2012학년도 고려대 수리논술 [더플러스수학] tv.naver.com (가) 두 함수 $ f $와 $ g $는 정의역과 공역이 모두 양의 실수 전체의 집합인 연속함수이다. 함수 $ f $는 정의역의 모든 점에서 양의 미분계수를 갖는다. 그림 1과 같이 임의의 양수 $ t $에 대하여 곡선 $ y=f ( x) $ 위의 점 $ \rm F$ $ ( t,~f ( t)) $에서의 접선과 $ x $축이 이루는 예각의 크기는, 원점과 점 $ \rm G$$ ( t,~g ( t)) $를 잇는 선분과 $ y $축이 이루는 예각의 크기와 같다. (나) 세 양수 $ a,~b..

['94 포스텍] 1994학년도 포스텍 본고사 문제 [내부링크]

실수 위에서 정의된 $ G ( x) $는 최댓값과 최솟값을 갖는 연속함수이고 $ m ( x) $는 양의 값을 갖고 $ m ( x)+m ( x+1)=1 $을 만족하는 함수이다. 모든 실수 $ x $에 대하여 $$ G ( 2x)=m ( x)G ( x)+m ( x+1)G ( x+1) $$ 이 성립한다. (1) 함수 $ G ( x) $가 $ x=x _ {0} $에서 최솟값을 가질 때, $$ G ( x _ {0} )=G \left ( \frac {x _ {0} } {2} \right ) $$임을 보여라. (2) 함수 $ G ( x) $는 상수함수임을 보여라. [‘94 포항공대] https://tv.naver.com/v/9942131 [포스텍 본고사] 1994학년도 포스텍 본고사문제 [더플러스수학] 더플러스수학 |..

[시립대 논술] 2010학년도 서울시립대 논술 [내부링크]

[2010학년도 서울시립대 논술] 반지름의 길이가 1인 원에 내접하는 정$ n $각형이 있다. 이 정$ n $각형을 둘레의 길이와 같은 길이의 실로 한 바퀴 감았다. 이 때, 실의 한 쪽 끝은 한 꼭짓점에 고정이 되어 있다. 정$ n $각형과 같은 평면 위에서 실을 팽팽한 상태로 유지하면서 실 전체가 최초로 직선이 되는 순간까지 풀었을 때, 실의 다른 끝점이 움직인 거리를 $ L _ {n} $이라고 하자. (a) 아래 그림은 정사각형의 경우를 보여준다. $ L _ {4} $를 구하여라. (b) $ L _ {n} $을 구하여라. (c) 정$ n $각형을 반지름의 길이가 1인 원으로 바꾸었을 때, 실의 끝점이 만드는 곡선의 길이를 구하여라. 연결문제 신개선(involute) 늘어나거나 줄지 않는 길이 $ 2..

[카톨릭대 의대] 2019학년도 카톨릭대 의대수리논술 [내부링크]

https://youtu.be/FxrJjUTBzdM(구독과 좋아요) [문항 1] 제시문 (ㄱ)~(ㄷ)을 읽고 논제에 답하시오. (100점) (ㄱ) 아래 그림과 같이 $ \rm \overline {AB} $와 $ \rm \overline {CD} $가 서로 평행인 사다리꼴 $ \rm ABCD $에서 $ \rm \overline {BC} $를 삼등분한 점 중 $ \rm B $에 가까운 점을 $ \rm M $, $ \rm \overline {CD} $를 $ 3:2 $로 내분한 점을 $ \rm N $, $ \rm \overline {BN} $과 $ \rm \overline {DM} $의 교점을 $ \rm E $, $ \rm \overline {CE} $를 포함하는 직선과 $ \rm \overline {AB} $..

서울권 6개대학 공통평가항목 학생부 사례(펌)-더플러스수학 [내부링크]

https://m.blog.naver.com/auraedu/221654032679 #학생부#공통평가항목#학생부사례 해설#서울권대학#더플러스수학

[선형대수학-일차변환] Linear Mappings [내부링크]

Linear Mappings 1. Suppose the mapping $ F~:~R ^ {2} \rightarrow R ^ {2} $ is defined by $ F ( x,~y)= ( x+y,~x) $. Show that F is linear. pf) We need to show that $ F ( v+w)=F ( v)+F ( w) $ and $ F ( kv)=kF ( v) $, where $ v $ and $ w $ are any elements of $ R ^ {2} $ and $ k $ is any scalar. Let $ v= ( a,~b) $ and $ w= ( a ' ,~b ' ) $. Then $ v+w= ( a+a ' ,~b+b ' ) $ and $ kv= ( ka,~kb) $ We ha..

[행렬식] 행렬식의 성질과 문제풀이 [내부링크]

Properties of Determinants THEOREM 8.1: The determinant of a matrix $ A $ and its transpose $ A ^ {T} $ are equal; that is, $ |A|=|A ^ {T} | $. Note that expanding A by column k is equivalent to expanding AT by row k. THEOREM 8.2: Let $ A $ be a square matrix. (i) If $ A $ has a row (column) of zeros, then $ |A|=0 $. (ii) If $ A $ has two identical rows (columns), then $ |A|=0 $. (iii) If $ A $ ..

[선형대수학] 연립방정식과 Echelon Form 문제풀이 [내부링크]

Systems in Triangular and Echelon Form 1. Determine the pivot and free variables in each of the following systems: (a) $$ \begin{split} 2x _ {1} -3x _ {2} -6 x _ {3} -5 x _ {4} +2 x _ {5} = 7 \\ x _ {3} +3 x _ {4} -7 x _ {5} = 6\\ x _ {4} -2 x _ {5} = 1 \end{split}$$ (b) $$ \begin{split} 2x-6y +7z=1\\4y+3z=8\\ -2z=4 \end{split}$$ (c) $$ \begin{split} &x+2y-3&z&=2\\2&x+3y+&z&=4\\3&x+4y+5&z&=8\end..

[선형대수학] 행렬 연산 문제풀이 [내부링크]

1. Prove Theorem 2.2(i) : $ \left ( AB \right ) C=A \left ( BC \right ) $. pf) Let $ A=[a _ {ij} ] $, $ B=[b _ {jk} ] $, $ C=[c _ {kl} ] $, and let $ AB=S=[s _ {ik} ] $, $ BC=T=[t _ {jl} ] $. Then $ s _ {ik} = \sum\limits _ {j=1} ^ {m} a _ {ij} b _ {jk} $ and $ t _ {jl} = \sum\limits _ {k=1} ^ {n} b _ {jk} c _ {kl} $ Multiplying $ S=AB $ by $ C $, the $ il- $entry of $ \left ( AB \right ) C $ is..

[옥동수학학원][대칭식과 교대식] 대칭식과 교대식의 성질과 인수분해[더플러스수학] [내부링크]

*대칭식, 교대식 정의① 대칭식대칭식은 식을 구성하고 있는 변수 중에서 어떠한 $ 2 $개의 변수를 바꾸어 계산하여도 그 결과가 원래의 식과 같아지는 식이다. 예를 들면 $ x+y $, $ xy $, $ x ^ {2} +y ^ {2} $, $ xyz $ 등등이 있다. $ 2 $개 변수 $ x,~y $를 사용하는 식을 $ f ( x,y) $라 하면 $$ f ( x,y)=f ( y,x) $$ $ 3 $개의 변수 $ x,~y,~z $를 사용하는 식을 $ f ( x,y,z) $라 하면 $$ f(x,y,z)=f(y,x,z)=f(x,z,y)=f(z,y,x)$$ 모든 대칭식은 기본대칭식에 대한 다항식으로 표현이 가능하다. 보조정리1 대칭식 $ S _ {n} =x ^ {n} +y ^ {n} $을 기본대칭식들과 간단한 ..

[성균관대 수리논술] 2009학년도 성균관대 과고전형 [더플러스수학] [내부링크]

http://tv.naver.com/v/9863952?openType=nmp [성균관대 수리논술] 2009학년도 성균관대 과고전형 [더플러스수학] 더플러스수학 | [성균관대 수리논술] 2009학년도 성균관대 과고전형 [더플러스수학] tv.naver.com [2009학년도 성균관대 면접고사] 함수 $ g~:~[a,~b] \rightarrow [a,~b] $을 닫힌 구간 $ [a,~b] $에서 연속인 함수라고 하자. 그러면 $ g ( p)=p $을 만족하는 실수 $ p \in [a,~b] $가 존재한다는 것은 잘 알려져 있다. 이 $ p $를 함수 $ g $의 부동점이라 부른다. 이 정리를 부동점 정리라고 한다. 만일 함수 $ g $가 (i) $ ( a,~b) $에서 미분가능이고, (ii) 어떤 양의 상수 ..

[2019학년도 나형 6월30번] 평가원 나형 6월30번 [내부링크]

https://tv.naver.com/v/10321434 [평가원기출] 2019학년도 나형 6월 30번 더플러스수학 | [평가원기출] 2019학년도 나형 6월 30번 tv.naver.com 사차함수 $ f ( x) $가 다음 조건을 만족한다. (가) $ 5 $이하의 모든 자연수 $ n $에 대하여 $ \sum\limits _ {k=1} ^ {n} f ( k)=f ( n)f ( n+1) $ 이다. (나) $ n=3,~4, $일 때, $ f ( x) $에서 $ x $의 값이 $ n $에서 $ n+2 $까지 변할 때의 평균변화율은 양수가 아니다. $ 128 \times f \left ( \frac {5} {2} \right ) $의 값을 구하시오. [4점][2018년 6월 30] 정답 및 풀이를 보려면 아래를..

[2020 나형 모평 9월30번] 2020학년도 나형 평가원 9월 30번 [내부링크]

https://tv.kakao.com/v/401998043 [2020 나형 모평 9월30번] 2020학년도 나형 평가원 9월 30번 최고차항의 계수가 $ 1 $인 사차함수 $ f \left ( x \right ) $에 대하여 네 개의 수 $ f \left ( -1 \right ) $, $ f \left ( 0 \right ) $, $ f \left ( 1 \right ) $, $ f \left ( 2 \right ) $가 이 순서대로 등차수열을 이루고, 곡선 $ y=f \left ( x \right ) $위의 점 $ \left ( -1,~f \left ( -1 \right ) \right ) $에서의 접선과 점 $ \left ( 2,~f \left ( 2 \right ) \right ) $에서의 접선이 ..

[2019학년도 모평 나형] 2019학년도 나형 6월 21번 [내부링크]

상수 $ a,~b $에 대하여 삼차함수 $ f ( x)=x ^ {3} +ax ^ {2} +bx $가 다음 조건을 만족시킨다. (가) $ f ( -1)>-1 $ (나) $ f ( 1)-f ( -1)>8 $ 에서 옳은 것만을 있는 대로 고른 것은? [4점][2018년 6월 21] ㄱ. 방정식 $ f ' ( x)=0 $은 서로 다른 두 실근을 갖는다. ㄴ. $ -1b $ $\cdots\cdots$ (나) 조건에 의해 $\begin{align} f ( 1)-f ( -1) &=1+a+b- ( -1+a-b)\\&=2+2b>8 \end{align} $ $ \therefore ~b>3 $ $\cdots\cdots$ ㄱ. $ f ' ( x)=3x ^ {2} +2ax+b $ , 에 의해 $ a>b~ \Right..

[사관학교 기출] 2018학년도 나형 21번 [내부링크]

자연수 $ n $에 대하여 함수 $ f ( x) $를 $ f ( x)=x ^ {2} + \frac {1} {n} $이라 하고 함수 $ g ( x) $를 $ g ( x)= \begin {cases} ( x-1)f ( x) & & ( x \geq 1) \\ ( x-1) ^ {2} f ( x) & & ( x

[연세대수리논술] 2019학년도 연세대 수리논술 [내부링크]

https://tv.kakao.com/v/402389393 [연세대수리논술] 2019학년도 연세대 수리논술 ※다음 제시문을 읽고 아래 질문에 답하시오. [제시문 1] 좌표평면 위의 두 초점 $\rm F$ $(4,~0)$, $ \rm F' (-4,~0)$ 로부터 거리의 합이 $10$인 타원 $C$ 가 있다. 타원 $C$ 위의 점 $\rm P$ $(x,~y)$와 초점 $ \rm F' (-4,~0)$를 지나는 직선이 $x$ 축의 양의 방향과 이루는 각을 $\alpha$ 라 하고, 점 $\rm P$ $(x,~y)$와 초점 $\rm F$ $(4,~0)$를 지나는 직선이 $x$ 축의 양의 방향과 이루는 각을 $\beta$ 라 하자. [1-1] 타원 $C$ 의 방정식을 구하시오. [$5$점] [1-2] $\ \cos..

[2018년나형10월30번] 2018년10월 나형 30번 킬러문항 [내부링크]

https://youtu.be/_jb9KvBbMSc(구독과 좋아요를..) 최고차항의 계수가 $ 1 $인 삼차함수 $ f \left ( x \right ) $와 실수 $ t $가 다음 조건을 만족시킨다. 등식 $ f \left ( a \right ) $$ +1=f ' \left ( a \right ) \left ( a-t \right ) $를 만족시키는 실수 $ a $의 값이 $ 6 $ 하나뿐이기 위한 필요충분조건은 $ -2

[연세대수리논술] 2017학년도 연세대 수리논술 [내부링크]

※ 다음 제시문을 읽고 아래 질문에 답하시오. [제시문 1] [가] 다항함수 $ h ( x) $ 위의 점 $ ( a,~h ( a)) $에서의 접선의 방정식은 다음과 같다. $$ y=h ' ( a) ( x-a)+h ( a) $$ [나] 다항함수 $ h ( x) $가 $ h ( x)= ( x-a) ^ {n} g ( x) $ (단, $ n $은 자연수이고, $ g ( x) $는 다항함수이다.)로 나타내어질 때, 방정식 $ h ( x)=0 $는 $ x=a $를 근으로 갖는다고 한다. 특히, $ n \geq 2 $이면 방정식 $ h ( x)=0 $은 $ x=a $에서 중근을 갖는다고 한다. [1-1] 곡선 $ y=x ^ {3} +1 $ 위의 점 $ ( 1,~2) $에서 접선의 방정식을 구하시오. [4점] [1-2..

[연세대수리논술] 2016학년도 연세대 수리논술 [내부링크]

다음 제시문을 읽고 물음에 답하시오. 【가】 자연수 $ n $에 대하여 집합 $ A _ {n} $을 $$A_n =\left \{ \frac{0}{2^n},~\frac{1}{2^n},~\cdots,~\frac{2^{n}-1}{2^n} \right \}$$ 이라 하자. 【나】 $ A _ {n} $의 임의의 원소 $ x $가 주어졌을 때, $ 0 $ 또는 $ 1 $의 값을 가지는 수열 $ a _ {1} ,~a _ {2} ,~a _ {3} , ~\cdots ,~a _ {n} $이 존재하여 항상 $$ x= \frac {a _ {1} } {2 ^ {1} } + \frac {a _ {2} } {2 ^ {2} } + \frac {a _ {3} } {2 ^ {3} } + \cdots + \frac {a _ {n} } {..

[연세대수리논술]2015학년도 연세대 수리논술 [내부링크]

연세대 2015학년도 수리논술문제 풀이입니다. https://tv.kakao.com/v/401942763 [연세대수리논술] 2015학년도 연세대 수리논술 다음 제시문을 읽고 아래 질문에 답하시오. [가] $ C _ {0} $는 좌표평면 위의 원 $ x ^ {2} +y ^ {2} =1 $이다. [나] $ n=1,2,3, \cdots $에 대하여 $ C _ {n} $은 다음 조건을 만족하는 원이라고 귀납적으로 정의한다. ① $ C _ {n} $은 좌표평면위의 $ x>0 $인 영역에서 $ C _ {n-1} $과 접한다. ② $ C _ {n} $은 쌍곡선 $ y ^ {2} -x ^ {2} =1 $의 $ y>0 $인 부분과 $ y

[한양대수리논술] 2017 한양대 모의논술 풀이 [더플러스수학] [내부링크]

[문제 1번] 다음 제시된 두 함수 $ f ( x) $와 $ g ( x) $에 대한 물음에 답하시오. (50점) $$ f ( x)= \frac {1} {2-\sin x} - \frac {1} {2-\cos x} ,~ g ( x)= \frac {2} {4-\sin ^ {2} x} - \frac {2} {4-\cos ^ {2} x} $$ 1. 모든 실수 $ x $에 대해서 $ f ( x)=f ( x+ \theta ) $를 만족시키는 최솟값 $ \theta $를 구하시오. 2. 방정식 $ f ( x)=g ( x) $을 만족시키는 $ x $값을 모두 구하시오. 3. 함수 $ g ( x) $의 최댓값과 최솟값을 구하시오. 4. 부등식 $$ \int _ {0} ^ { \frac {\pi } {4} } {f ( x)..

[한양대수리논술] 2018학년도 한양대 모의논술 [내부링크]

[문제 1번] 다음 물음에 답하시오. (50점) 1. 양의 실수 $ x $에 대하여 $ f ( x)= \frac {8+x} {3} - \root {3} \of {15x} $의 최솟값을 구하시오. 2. 모든 양의 실수 $ x $에 대하여$ g ( x)= \frac {10+x} {5} - \root {5} \of {24x} >0 $임을 보이시오. 3. 임의의 양의 실수 $ a _ {1} ,~a _ {2} ,~ \cdots ,~a _ {2017} $에 대하여 다음 부등식이 성립함을 보이시오. $$ \frac {a _ {1} + \cdots +a _ {2017} } {2017} \geq \root {2017} \of {a _ {1} \cdots a _ {2017} } $$ [문제 2번] 양의 실수 $ a,~b ..

[성균관대 수리논술] 2017학년도 성균관대 수리논술 자연1 [내부링크]

[수학 1] 다음 ~ 을 읽고 [수학 1 -ⅰ] ~ [수학 1 -ⅱ]를 문항별로 풀이와 함께 답하시오. 함수 $ f ( x) $가 $ x=a $에서 미분가능할 때, 곡선 $ y=f ( x) $ 위의 점 $ \rm P$ $(a,~f ( a)) $에서의 접선의 방정식은 다음과 같다. $$ y-f ( a)=f ' ( a) ( x-a) $$ 구간 $ [a,~b] $에서 연속인 두 곡선 $ y=f ( x),~y=g ( x) $와 두 직선 $ x=a,~x=b $로 둘러싸인 도형의 넓이는 $ \int _ {a} ^ {b} {} \left | f ( x)-g ( x) \right | dx $이다. 실수 전체의 집합 $ R $에 대하여 함수 $ f:R \rightarrow R $를 $$ f(x)= \begin{cases..

[2020 모평 나형 9월21번] 2020학년도 평가원 나형 9월 21번 킬러문항 [내부링크]

함수 $ f ( x)=x ^ {3} +x ^ {2} +ax+b $에 대하여 함수 $ g ( x) $를 $ g ( x)=f ( x)+ ( x-1)f ' ( x) $라 하자. 에서 옳은 것만을 있는 대로 고른 것은? (단, $ a,~b $는 상수이다.) [$ 4 $점] ㄱ. 함수 $ h ( x) $가 $ h ( x)= ( x-1)f ( x) $이면 $ h ' ( x)=g ( x) $이다. ㄴ. 함수 $ f ( x) $가 $ x=-1 $에서 극값 $ 0 $을 가지면 $ \int _ {0} ^ {1} {} g ( x)dx=-1 $이다. ㄷ. $ f ( 0)=0 $이면 방정식 $ g ( x)=0 $은 열린 구간 $ ( 0,~1) $에서적어도 하나의 실근을 갖는다. ① ㄱ ② ㄴ ③ ㄱ, ㄴ ④ ㄱ, ㄷ ⑤ ㄱ, ..

[2020학년도 나형 모평 9월 16번] 2020학년도 평가원 나형 9월 16번 [내부링크]

https://tv.kakao.com/v/401885511 2020학년도 평가원 나형 9월 16번 다항함수 $ f \left ( x \right ) $가 $ \lim\limits _ {x \rightarrow \infty } { \frac {f \left ( x \right )} {x ^ {3} } =1,~~ \lim\limits _ {x \rightarrow -1} { \frac {f \left ( x \right )} {x+1} =2} } $를 만족시킨다. $ f \left ( 1 \right ) \leq 12 $일 때, $ f \left ( 2 \right ) $의 최댓값은? [$ 4 $점] ① $ 27 $ ② $ 30 $ ③ $ 33 $ ④ $ 36 $ ⑤ $ 39 $ 정답 및 풀이를 보려면 아래..

[2019학년도 이화여대 의대논술] 이화여대 의대논술 [내부링크]

https://tv.kakao.com/v/401721198 1. 도형 $ S $는 좌표평면에서 중심이 원점이고 반지름의 길이가 $ 1 $인 원을 직선 $ x=h $ ($ -1 \leq h \leq 1 $)으로 왼쪽 부분을 자른 도형이다. 도형 $ S $의 내부에 있고 가로는 $ x $축에 평행하고 세로는 $ y $축에 평행한 직사각형에 대하여 다음 물음에 답하시오. [40점] (1) $ h=-1 $일 때, 직사각형의 넓이의 최댓값을 구하시오. (2) $ h=- \frac {3} {4} $일 때, 직사각형의 넓이의 최댓값을 구하시오. (3) $ h=- \frac {1} {\sqrt {6} } $일 때, 직사각형의 넓이의 최댓값을 구하시오. 2. 양의 실수 $ a $에 대하여 실수 전체의 집합에서 미분가능한..

[2019학년도 이화여대수리논술] 이대수리논술 [내부링크]

https://tv.naver.com/v/9787661 [이화여대수리논술] 2019학년도 이화여대 수리논술 더플러스수학 | [이화여대수리논술] 2019학년도 이화여대 수리논술 tv.naver.com 1. 도형 $ S $는 좌표평면에서 중심이 원점이고 반지름의 길이가 $ 1 $인 원을 직선 $ x=h $ ($ -1 \leq h \leq 1 $)으로 왼쪽 부분을 자른 도형이다. 도형 $ S $의 내부에 있고 가로는 $ x $축에 평행하고 세로는 $ y $축에 평행한 직사각형에 대하여 다음 물음에 답하시오. [40점] (1) $ h=-1 $일 때, 직사각형의 넓이의 최댓값을 구하시오. (2) $ h=- \frac {3} {4} $일 때, 직사각형의 넓이의 최댓값을 구하시오. (3) $ h= \frac {1} ..

[2020학년도 9월 평가원] 2020학년도 9월 가형 19번풀이 [내부링크]

https://tv.kakao.com/v/401826482 2020학년도 9월 가형 19번 좌표평면 위에 두 점 $ \rm A ( 1,~0),~B ( 0,~1) $이 있다. 중심각의 크기가 $ \frac {\pi } {2} $인 부채꼴 $ \rm OAB $의 호 $ \rm AB $ 위를 움직이는 점 $ \rm X $와 함수 $ y= ( x-2) ^ {2} +1~ ( 2 \leq x \leq 3) $의 그래프 위를 움직이는 점 $ \rm Y $에 대하여 $ \overrightarrow {\rm OP \it } = \overrightarrow {\rm OY \it } - \overrightarrow {\rm OX \it } $를 만족시키는 점 $ \rm P $가 나타내는 영역을 $ R $라 하자. 점 $ \..

[2020학년도 가형 9월 15번] 2020학년도 평가원 9월 15번 [내부링크]

https://tv.kakao.com/v/401826481 2020학년도 평가원 9월 15번 함수 $ y=e ^ {x} $의 그래프 위의 $ x $좌표가 양수인 점 $ \rm A $ 와 함수 $ y= -\ln x $의 그래프 위의 점 $ \rm B $가 다음 조건을 만족시킨다. (가) $ \overline {\rm OA}$ $=2 \overline {\rm OB } $ (나) $ \angle \rm AOB $$ =90 $ 직선 $ \rm OA $의 기울기는? (단, $ \rm O $는 원점이다.) [$ 4 $점] ① $ e $ ② $ \frac {3} {\ln 3} $ ③ $ \frac {2} {\ln 2} $ ④ $ \frac {5} {\ln 5} $ ⑤ $ \frac {e ^ {2} } {2} $ ..

[2020학년도 9월 17번] 9월 평가원 가형 17번 풀이 [내부링크]

https://tv.kakao.com/v/401865773 2020학년도 9월 평가원 가형 17번 두 함수 $ f \left ( x \right ) $, $ g \left ( x \right ) $는 실수 전체의 집합에서 도함수가 연속이고 다음 조건을 만족시킨다. (가) 모든 실수 $ x $에 대하여 $ f \left ( x \right ) g \left ( x \right ) =x ^ {4} -1 $이다. (나) $ \int _ {-1} ^ {1} {\left\{ f \left ( x \right ) \right\} ^ {2} g ' \left ( x \right ) dx} =120 $ $ \int _ {-1} ^ {1} {x ^ {3} f \left ( x \right ) dx} $의 값은? [$ 4..

[2020학년도 9월 가형 30번] 9월 평가원 30번 킬러문항 풀이 [내부링크]

#울산남구 수학학원#더플러스수학 https://tv.kakao.com/v/401832306 [2020학년도 9월 가형 30번] 9월 평가원 30번 킬러문항 풀이 실수 전체의 집합에서 미분가능한 함수 $ f ( x) $가 모든 실수 $ x $에 대하여 $ f ' ( x ^ {2} +x+1)= \pi f ( 1) \sin \pi x+f ( 3)x+5x ^ {2} $ 을 만족시킬 때, $ f ( 7) $의 값을 구하시오. [$ 4 $점] 정답 및 풀이를 보려면 아래를 클릭하세요. ...더보기 정답 $ 93 $ 준식의 양변에 $ ( 2x+1) $을 곱하면 $ \left ( 2x+1 \right ) \cdot f ' ( x ^ {2} +x+1) $$ = \pi f ( 1) \cdot \left ( 2x+1 \ri..

[2020학년도 9월 모의고사] 9월 평가원 29번(킬러문항) [내부링크]

https://tv.kakao.com/v/401826488 2020학년도 가형 9월 평가원 29번 좌표공간에서 원점 $ \rm O $와 점 $ \rm A ( 4,~0,~0) $에 대하여 평면 $ x+y+ \sqrt {2} z=0 $ 위의 점 $ \rm P $가 다음 조건을 만족시킨다. (가) $ \left | \rm \overrightarrow {OP} \right | $는 $ 9 $이하의 자연수이다. (나) $ \rm \overrightarrow {OA} \cdot \overrightarrow {AP} =6 $ $ \rm \overrightarrow {AP} \cdot \overrightarrow {OP} $의 최댓값을 $ M $, 최솟값을 $ m $이라 할 때, $ M+m $의 값을 구하시오. [$ ..

[2020학년도 9월 모평] 2020학년도 9월 가형 21번 풀이 (킬러문항) [내부링크]

https://tv.kakao.com/v/401826500 2020학년도 9월 가형 21번 좌표평면에서 두 점 $ \rm A ( -2,~0) $, $ \rm B ( 2,~0) $에 대하여 다음 조건을 만족시키는 직사각형의 넓이의 최댓값은? [$ 4 $점] 직사각형 위를 움직이는 점 $ \rm P $에 대하여 $ \overline {\rm PA \it } + \overline {\rm PB \it } $의 값은 점 $ \rm P $의 좌표가 $ ( 0,~6) $일 때 최대이고 $ \left ( \frac {5} {2} ,~ \frac {3} {2} \right ) $일 때 최소이다. ① $ \frac {200} {19} $ ② $ \frac {210} {19} $ ③ $ \frac {220} {19} $ ..

[한국외대 수시]한국외대, 수시 교과전형 수능 최저학력 기준 적용 폐지(펌) [내부링크]

#교과전형 #한국외대 연세대가, 외대가 왜 수능을 폐지하는가? 9평 1등급이 실제 수능에서 1등급 받는 비율이 반도 안되는.. 수능최저 없으면 경쟁률 어마무시 작용 반작용의 법칙 아시죠?~ ㅡㅡㅡㅡㅡㅡㅡㅡㅡㅡ 한국외대, 수시 교과전형 수능 최저학력 기준 적용 폐지 교과 100%만으로 선발, 수능 성적 상관없이 지원가능 올해 한국외대 학생부교과전형에 지원하고자 하는 수험생은 수능최저학력기준에 대한 부담을 덜게 됐다. 한국외대는 2020학년도 수시모집에서 총 2153명을 선발한다고 28일 밝혔다. 전형별로는 학생부교과전형 562명 학생부종합전형(고른기회전형 포함) 1011명 논술전형 493명 특기자전형(외국어, 소프트웨어) 87명 등이다. 올해 수시모집에서 한국외대 학생부교과전형에 지원하는 수험생..

2. 행렬식 [내부링크]

2. 행렬식 정의 2.1.1. 행렬식 $ n $차의 정사각행렬 $ A $의 행렬식을 $ |A| $ 또는 $ detA $로 나타내며, 다음과 같이 귀납적으로 정의한다. (i) $ n=1 $일 때, $ |a _ {11} |=a _ {11} $ (ii) $ n>1 $일 때, $$ \left| \matrix {a _ {11} & a _ {12} & \cdots & a _ {1n} \\a _ {21} & a _ {22} & \cdots & a _ {2n} \\\vdots & \vdots & \vdots & \vdots \\a _ {n1} & a _ {n2} & \cdots & a _ {nn} } \right| = \sum\limits _ {k=1} ^ {n} \left ( -1) ^ {i+n} a _ {i n }..

[더플러스수학] 2019학년도 부산대학교 수리논술 [내부링크]

https://tv.kakao.com/channel/3372901/cliplink/401670285 【문항 1】다음 제시문을 이용하여 아래 논제의 풀이 과정과 답을 논리적으로 서술하시오. (가) 함수 $ f ( x) $의 도함수 $ f ' ( x) $가 미분가능하면 $ f ' ( x) $의 도함수 $$ \lim\limits _ {\Delta x \rightarrow 0} {} \frac {f ' ( x+ \Delta x)-f ' ( x)} {\Delta x} $$ 를 함수 $ f ( x) $의 이계도함수라고 하며, 이것을 $ f '' ( x) $로 나타낸다. (나) 미분가능한 함수 $ g ( t) $에 대하여 $ x=g ( t) $로 놓으면 $$ \int {f ( x)dx} = \int f ( g ( t..

[더플러스수학] 2022학년도 대입전형 계획(펌) [내부링크]

올해 고등학교 1학년에 제학중인 학생들이 치르게 2022학년도 대입전형 기본계획이 어제 발표되었습니다. 발표 내용 가운데 가장 중요하다고 볼 수 있는 것 중 하나인 전형 일정을 정리해 보았습니다. 고1 학생들이 참조했으면 해서 이곳에 올립니다. 혹시 아래 원고를 활용하거나 옮기고자 하실 때는꼭 출처를 밝혀주시면 고맙겠습니다. 올해 고등학교 1학년에 재학중인 학생들이 치르게 될 2022학년도 대입전형의 주요 일정은 고2 학생들이 치르게 될 2021학년도 대입전형보다 사나흘 늦추어 진행된다. 이는 수시 모집의 입학원서 접수 기간이 2021학년도 대입전형에서는 9월 7일부터 11일까지인데 2022학년도 대입전형에서는 9월 10일부터 14일까지로 사흘 늦어짐에 따라서이다. 이에 정시 모집의 입학원서 접수 기간도..

[더플러스수학] 2019학년도 경북대 논술(AAT) [내부링크]

https://tv.kakao.com/channel/3372901/cliplink/401464617 수학 (문제1) [1] 다음 글을 읽고 물음에 답하시오. (가) (1) 서로 다른 $ n $개에서 $ r $ ($ 0 c > 0 $, $ b ^ {2} =a ^ {2} -c ^ {2} $)이다. (나) 타원의 방정식 $\displaystyle \frac {x ^ {2} } {a ^ {2} } + \frac {y ^ {2} } {b ^ {2} } =1 $을 $ x $축의 방향으로 $ m $만큼, $ y $축의 방향으로 $ n $만큼 평행이동한 타원의 방정식은 $\displaystyle \frac { ( x-m) ^ {2} } {a ^ {2} } + \frac { ( y-n) ^ {2} } {b ^ {2} }..

[더플러스수학] 2011학년도 서강대 수리논술 [내부링크]

다음 글을 읽고, 물음에 답하라. 미분 가능한 두 함수 $ f ( x) $와 $ g ( x) $가 있을 때, 곱의 미분법에 의하면 $ f ( x)g ( x) $의 도함수는 다음과 같이 구해진다. $$ \frac {d} {dx} \left [ f ( x)g ( x) \right ] =f ( x)g ' ( x)+f ' ( x)g ( x) $$ 부정적분의 기호를 사용해서 위 식을 나타내면 다음과 같다. $$ \int _ {} ^ {} {\left [ f ( x)g ' ( x)+f ' ( x)g ( x) \right ] dx=f ( x)g ( x)} $$ 또는 $$ \int _ {} ^ {} {} f ( x)g ' ( x)dx+ \int _ {} ^ {} {} f ' ( x)g ( x)dx=f ( x)g ( x..

[더플러스수학] 2005학년도 서울대 구술면접문제 [내부링크]

[2005 서울대 구술] $ x,~a $가 1보다 큰 실수일 때, $ \log _ {a} x $를 다음과 같이 정의할 수도 있다. $$ \log _ {a} x= \frac { \int _ {1} ^ {x} {} \frac {1} {t} dt} { \int _ {1} ^ {a} {} \frac {1} {t} dt} $$ (1) 부등식 $ 0

[더플러스수학] 2009학년도 서울대 심층면접 [내부링크]

[서울대 2009학년도 특기자 자연대] 실수에서 실수로 가는 함수 $ f $는 두 번 미분가능하고, 함수 $ f,~f ' ,~f '' $는 모두 실수에서 연속이다. 이 때 다음 물음에 답하여라. (1) $$ g ( c)= \frac {2} {h ^ {2} } \int _ {0} ^ {h} {} \int _ {0} ^ {s} {} g ( t)dtds $$ 를 만족하는 $ c $가 $ ( 0,h) $에 존재함을 보여라. 단, $ g ( x) $도 연속함수이다. (2) $$ \frac {f ( x+h)+f ( x-h)-2f ( x)} {h ^ {2} } = \frac {f '' ( x+c)+f '' ( x-c)} {2} $$ 인 $ c $가 $ ( 0,h) $에 존재함을 보여라. (3) $$ \frac {f ..

[더플러스수학] 2010 UNIST 심층면접문제 [내부링크]

[2010 UNIST 입학사정관제] 울산에서 열리는 국제 도자기 축제에 어떤 외국인이 도공에게 도기를 주문제작 요청을 했는데 이 때 도공이 도자기를 빚는데 사용할 진흙의 양을 계산한다고 하자. (1) 다음 도자기에 사용된 진흙의 부피를 회전체 적분법을 이용하여 적분식으로 표현하시오.(단, $ x=g ^ {-1} ( y) $와 $ x=f ^ {-1} ( y) $가 존재한다.) (2) 그림3에서 빗금 친 부분의 한 쪽을 잘라서 펼친 모양이 그림4와 같다고 할 때 길이 $ L $을 정의하시오. (3) (2)의 결과를 이용하여 아래의 그림 5, 6과 같은 방법으로 도기 제작에 사용할 진흙의 부피를 구하고자 한다. 이때 부피를 적분식으로 표현하시오 (4) (1)의 방법으로 구한 부피와 (3)의 방법으로 구한 진흙..

2012학년도 아주대 수리논술 예시문제 [내부링크]

[2012학년도 아주대 논술 예시문제] 바퀴에 야광패널이 붙은 자전거가 어둠 속에서 지나가면 이 야광패널이 매우 독특한 곡선을 그리게 된다. 이 곡선을 수학적으로 정의하면 싸이클로이드(cycloid)곡선이 된다. 이 싸클로이드곡선은 직선 위를 미끄러지지 않고 굴러가는 원 위의 한 점이 그리는 곡선이다. 싸이클로이드곡선을 방정식으로 나타낼 때는 매개변수를 이용한 방정식으로 나타내는 것이 편리하다. 위 그림에서, 원점에서 $ x $-축에 접하고 있는 반지름 $ r $인 원 $ C $가 $ x $-축을 따라 오른쪽으로 굴러 이동하여 점 $ P $에서 접하는 원 $ C ' $이 되었다고 하자, 그리고 원점과 접한 원 위의 점 $ A $는 이 이동으로 인해 접점 $ P $로부터 시계방향으로 $ \theta $만큼..

[더플러스수학] 2003학년도 중앙대 수리논술 [내부링크]

[2003학년도 중앙대] 무한히 늘어날 수 있는 고무로 된 띠 위를 따라 개미가 분당 $ \frac {1} {3} \rm m $의 속력으로 직선 위를 기어가고 있다. 처음의 띠 길이는 $ \rm 1m $였고 $ 1 $분 지날 때마다 띠의 길이가 $ k $배씩 늘어난다고 하자. 띠의 한쪽 끝에서 출발을 한 개미가 결국 띠의 다른 쪽 끝에 도달하려면 $ k $는 어떤 조건을 만족해야 하는지 설명하여라.(여기서 $ k $는 $ 1 $보다 큰 유리수다.) https://tv.kakao.com/channel/3372901/cliplink/401423148 정답 $ 1

[연세대모의논술] 2009학년도 연세대 모의수리논술 [내부링크]

[연세대 2009년 모의논술] 제시문을 읽고 물음에 답하시오. (40점) 함수 $ f ( x) $의 도함수 $ f ' ( x) $가 닫힌구간 $ \left [ a,~b \right ] $에서 연속이고, $ y=f ( x) $의 그래프가 [그림 1]과 같을 때, 다음 물음에 답하시오. [문제 1-1] 곡선 $ y=f ( x) $ 위의 점 $ ( a,~f ( a)) $부터 점 $ ( b,~f ( b)) $까지의 곡선의 길이를 정적분의 정의를 이용하여 구하시오. (10점) [문제 1-2] [그림 2]는 [그림 1]의 닫힌구간 $ \left [ a,~b \right ] $를 $ 2n $개의 균등한 소구간으로 나눈 그래프이다. 이때, 점 $ ( x _ {2k-1} ,~f ( x _ {2k-1} ) ) $에서의 접..

[더플러스수학] 2005학년도 서울대 심층면접 [내부링크]

[서울대 2005학년도 수시] 다음 물음에 답하시오. (1) $ y= \sqrt {x ( 1-x)} $의 그래프의 개형을 설명하고 $ \int _ {0} ^ {1} {} \sqrt {x ( 1-x)} dx $의 값을 설명하시오. (2) (1)을 활용하여 $ \int _ {0} ^ {1} {x} \sqrt {x ( 1-x)} dx $의 값을 구하시오. (1) 생략 (2) $ \frac {\pi } {8} $

[더플러스수학] 2004학년도 서울대 심층면접 [내부링크]

[서울대 2004학년도 수시] 반지름이 1$ cm $이고 중심의 좌표가 $ \left ( 0,1 \right ) $인 원이 있다. 여기서 좌표의 단위는 $ cm $이다. 이 원의 중심이 일정한 속도 $ 2cm/\sec $로 수직방향으로 위로 올라가고 그와 동시에 반지름은 일정한 속도 $ 1cm/\sec $로 커지기 시작했다. 1초 후부터는 중심이 올라가는 속도가 $ \frac {1} {2} cm/\sec $로, 반지름의 변화하는 속도는 0으로 바뀐다. 원이 움직이기 시작한 $ t $초가 지났을 때 중심의 좌표를$ \left ( 0,y \left ( t \right ) \right ) $, 반지름을 $ r \left ( t \right ) cm $, 원점에서 이 원에 그은 두 접선이 이루는 예각을 $ \t..

[더플러스수학] 2009학년도 고려대 수리논술(모의) [내부링크]

2009학년도 고려대 모의논술 (나) [평균값의 정리] 함수 $ y=f ( x) $가 닫힌구간$ [a,~b] $에서 연속이고, 열린구간 $ ( a,~b) $에서 미분가능하면 $ \frac {f ( b)-f ( a)} {b-a} =f ' ( c) $인 $ c $가 열린구간 $ ( a,~b) $ 안에 적어도 하나 존재한다. (다) 형과 동생이 일직선 도로에서 자전거 시합을 한다. 동생은 출발선으로부터 50지점에서 출발하기로 하였다. 둘이 동시에 출발하여 $ T $초 후 형은 200미터 지점을, 동생은 150미터 지점을 통과하였다. 출발 $ t $초 후 형의 위치를 $ x _ {t} ( t) $미터라 하고 동생의 위치를 $ x _ {2} ( t) $미터라 하면 운동의 물리적 특성으로 인해 $ x _ {1} (..

[더플러스수학] 2002학년도 서울대 의대 심층면접(정시) [내부링크]

[서울대 2002학년도 의대 정시] 자연로그의 밑을 $ e $라 둔다. (1) 극한값 $ \lim\limits _ {t \rightarrow + \infty } { \frac {t} {e ^ {t} } } $을 말하라. [단답형] (2) 다음 함수가 $ x=0 $에서 미분가능함을 보이고, $ y=f \left ( x \right ) $의 도함수를 구하라. $$ f \left ( x \right ) = { \begin {cases} e ^ {- \frac {1} {x} }, & x>0 \\ 0, & x \le 0\end {cases} } $$ (3) 위의 함수 $ y=f \left ( x \right ) $에 대하여 정적분 $ \int _ {0} ^ {1} {f \left ( t \right ) f \le..

[더플러스수학] 서강대 심층면접문제(연도미상) [내부링크]

[서강대] 함수 $ f ( x) $가 닫힌구간 $ [-1,1] $위의 모든 $ x $에 대하여 $ |f ( x)| \leq |x| ^ {2} $을 만족할 때, (1) $ \lim\limits _ {x \rightarrow 0} {} \frac {f ( x)} {x} $의 값을 구하여라. (2) $ f ( x) $가 $ x=0 $에서 미분가능함을 설명하고, $ f ' ( 0) $를 구하여라. (3) 함수 $ f ( x) $가 $ f ( x)= { \begin {cases} x ^ {2} \sin \frac {1} {x ^ {2} } & ( x \neq 0) \\ 0 & ( x=0)\end {cases} } $으로 주어질 때, $ f ' ( x) $를 구하여라. 힌트 및 정답 (1) $ 0 $ (2) $ 0..

[더플러스수학] 2006학년도 서울대 심층면접(특기자전형) [내부링크]

[서울대 2006학년도 특기자 수시] 함수 $ f: $ R → R 이 $ \lim\limits _ {h \rightarrow 0} {\left\{ f \left ( a+h \right ) -f \left ( a-h \right ) \right\} =0} $을 만족할 때 “$ x=a $에서 대칭연속”이라고 정의하자. 함수 $ f $가 모든 점에서 대칭연속일 때 $ f $를 “대칭연속함수”라고 하자. 한편 다음 극한 $ \lim\limits _ {h \rightarrow 0} { \frac {f \left ( a+h \right ) -f \left ( a-h \right )} {2h} } $가 존재할 때 “$ x=a $에서 대칭미분가능” 하다고 정의하고, 또한 모든 점에서 대칭미분가능하면 함수 $ f $가 ..

[더플러스수학] 2008학년도 서울대 수리논술 (정시) [내부링크]

https://youtu.be/scf2gzZ2dA4[서울대 2008학년도 정시] 다음 제시문을 읽고 논제에 답하시오.(가) 닫힌구간 $ \left [ a,b \right ] $에서 연속인 함수 $ f $에 대하여 $$ \frac {1} {b-a} \int _ {a} ^ {b} {f \left ( x \right ) dx=f \left ( c \right )} $$ 를 만족하는 $ c $가 $ a $와 $ b $ 사이에 적어도 하나는 존재한다는 사실이 잘 알려져 있다. 이를 ‘적분에 관한 평균값의 정리’라고 한다. 이것은 닫힌구간 $ \left [ a,~b \right ] $에서 $ f \left ( x \right ) \geq 0 $일 때, 곡선 $ y=f \left ( x \right ) $와 $ x ..

[더플러스수학] 포스텍 심층면접문제(연도미상) [내부링크]

[포스텍] (1) 닫힌 구간 $ [0,~1] $에서 방정식 $$ 2\ln ( \cos x)+x ^ {2} =0 $$ 의 해는 $ x=0 $ 하나뿐임을 보여라. (2) 무한급수 $$ \sum\limits _ {n=1} ^ {\infty } n \left | \ln \left ( \cos \frac {1} {n} \right ) \right | $$ 이 발산함을 보여라.

[더플러스수학] 2005학년도 서울대 심층면접 문제 (정시) [내부링크]

[서울대 2005학년도 정시] 다음 물음에 답하여라. (1) $ P $가 $ n $차 다항식일 때, 방정식 $ P \left ( x \right ) =0 $의 근의 개수는 $ n $보다 클 수 없음을 증명하시오. (2) 다항식 $ f _ {1} ,f _ {2} ,f _ {3} , \cdots $가 다음을 만족한다. (가) $ f _ {1} \left ( x \right ) =x $ (나) $ \frac {d} {dx} f _ {n} \left ( x \right ) =nf _ {n-1} \left ( x \right ) \left ( n=2,3, \cdots \right ) $ (다) $ \int _ {-1} ^ {1} {f _ {n} \left ( x \right ) dx=0 \left ( n=1,2,..

[더플러스수학] 2011학년도 연세대(원주) 의대 수리논술 [내부링크]

[2011학년도 연세대(원주) 의예과 논술] [문제1] 다음 제시문을 읽고 아래 질문에 답하시오.(50점) 함수 $ y=f ( x) $가 어떤 구간 $ I $에 속하는 임의의 두수 $ a,~b $에 대하여 $ a

[더플러스수학] 서울대 심층면접문제(연도미상) [내부링크]

다음 물음에 답하시오.[서울대] (1) $ n $이 자연수이고 $ x>0 $이면 $ e ^ {x} >1+ \frac {1} {1!} x+ \frac {1} {2!} x ^ {2} + \cdots + \frac {1} {n!} x ^ {n} $임을 보여라. (2) 위의 부등식을 이용하여 주어진 임의의 정수 $ n $에 대하여 $ \lim\limits _ {x \rightarrow \infty } {x ^ {n} } e ^ {-x} =0 $임을 보여라.(단, 로피탈 정리를 쓰면 안된다.)

[더플러스수학] 2010학년도 성균관대 과고전형 [내부링크]

[2010학년도 성균관대 과학인재전형] 부등식 $ a>b>0 $을 만족하는 모든 실수 $ a,~b $ 에 대하여 다음 부등식이 성립하는 최소의 양의 실수 $ m $을 구하여라. (답안 작성시 풀이과정을 정확하게 쓰시오.) $$ ab ( a ^ {2} -b ^ {2} ) \leq m ( a ^ {2} +b ^ {2} ) ^ {2} $$ [2008학년도 성균관대 과학인재전형] 미분가능한 함수 $ f~:~[0,~ \infty ) \rightarrow R $는 모든 $ x $에 대하여 $ f ( x)>0 $이고 다음 등식을 만족한다고 가정한다. $$ 27+ \int _ {0} ^ {x} { \frac {3 ( 1-t)} { ( t+1) ( t ^ {2} +1)} f ^ {2} ( t)dt=f ^ {3} ( x)..

[더플러스수학] 2011학년도 서강대 수리논술 [내부링크]

(2011학년도 서강대 수시1차 논술) 두 개의 원이 직각으로 만난다는 것은, 두 개의 점에서 만나며 각 교점에서의 각 원에 대한 접선이 수직인 것을 말한다. 중심이 점 $ O $이고 반지름이 $ 1 $인 원 $ C $에 대하여, [그림 3]과 같이 $ C $와 직각으로 만나며 반지름이 $ r _ {n} $이고 중심이 $ O _ {n} $인 원 $ C _ {n} $들을 배치하려고 한다($ n=1,2, \cdots $). 이때 원 $ C _ {1} ,C _ {2} ,C _ {3} , \cdots $의 내부들은 서로 만나지 않도록 한다. $ C $와 $ C _ {n} $의 두 교점을 각각 $ P _ {n} $, $ Q _ {n} $이라 하면 직선 $ OP _ {n} $과 $ OQ _ {n} $은 각각 점 $..

[더플러스수학] 2007학년도 포스텍 구술면접 [내부링크]

[2007 포항공대] 임의의 자연수 $ n $에 대하여 $ \cos nx $를 $ \cos x $에 관한 다항식으로 표현할 수 있음을 보여라. (힌트 $ -\sin x= \left ( \cos x \right ) ' $, $ -n\sin nx= \left ( \cos nx \right )'$)

[더플러스수학] 2008학년도 성균관대 과학인재전형 [내부링크]

[2008학년도 성균관대 수시2 과학인재전형] 폐구간 $ [a,~b] $에 정의된 함수 $ f~:~[a,~b] \rightarrow R $이 연속이고 $ ( a,~b) $에서 미분가능하면 다음 평균값 정리를 만족하는 것이 알려져 있다. 평균값 정리 : $ a $와 $ b $사이에 어떤 $ c $가 존재하여 $ f ( b)-f ( a)= \frac {df} {dx} ( c) ( b-a) $를 만족한다. (a) 위의 정리의 의미를 (그래프를 생각하여 기하학적인 관점에서) 설명하시오. (b) 폐구간 $ [-1,~1] $에서 연속이면서 미분가능한 함수 $ f $가 다음과 같이 정의되어 있다. $ f ( x)= \frac {x ^ {4} +x ^ {3} +x ^ {2} +x+4} {2x ^ {8} +3x ^ {6..

[더플러스수학] 2009학년도 서울시립대 심층면접 [내부링크]

[2009학년도 서울시립대 심층면접문제] 지수함수 $ e ^ {x} $를 이용하여 두 함수 $ f ( x) $와 $ g ( x) $를 다음과 같이 정의하자. $ f ( x)= \frac {1} {2} \left ( e ^ {x} -e ^ {-x} \right )$ , $g ( x)= \frac {1} {2} \left ( e ^ {x} +e ^ {-x} \right ) $ 그러면 두 함수는 서로 도함수가 된다. 즉, $ \frac {d ( f ( x))} {dx} =g ( x), ~\frac {d ( g ( x))} {dx} =f ( x) $이 성립한다. 또한 $ [g ( x)] ^ {2} -[f ( x)] ^ {2} =1 $의 방정식도 성립한다. 주어진 두 함수를 이용하여 다음의 물음에 답하여라. (a..

[더플러스수학] 2011학년도 시립대 수리논술 [내부링크]

[2011년도 시립대 수리논술] 개구간 $ \left ( - \frac {\pi } {2} , \frac {\pi } {2} \right ) $에서 정의된 함수 $ F ( x)=\sin x $의 역함수를 $ G ( x) $라고 하고, 개구간 $ ( -2,2) $에서 정의된 함수 $ f ( x)=2\sin \left ( \frac {\pi x} {4} \right ) $의 역함수를 $ g ( x) $라고 하자. (a) $ g ( 1) $과 $ h ( g ( \sqrt {2} )) $의 값을 구하여라. 여기서 $ h ( x) $는 개구간 $ \left ( - \frac {\pi } {2} , \frac {\pi } {2} \right ) $에서 정의된 $ \tan x $의 역함수이다. (b) 함수 $ G $..

[더플러스수학] 2011학년도 서강대 구술면접(심층면접) [내부링크]

[2011학년도 서강대 구술면접문제] 다음 질문에 답하시오. 1. 함수 $ f ( x) $가 $ \lim\limits _ {h \rightarrow 0} { \frac {f ( 2+h)-f ( 2)} {\root {3} \of {h} } =1} $ 을 만족할 때 $ f ( x) $는 $ x=2 $ 에서 연속인가? 2. 함수 $ f ( x) $가 $ \lim\limits _ {x \rightarrow 2} { \frac {f ( x)-f ( 2)} {x-2} =0} $ 을 만족할 때, $ \lim\limits _ {h \rightarrow 0} { \frac {f ( 2+h)-f ( 2-h)} {2h} =0} $ 이 성립하는가? 3. $ f ( x)=|x-2| $ 일 때 $ \lim\limits _ {h ..

[더플러스수학] 2009학년도 성균관대 과고전형 [내부링크]

[2009학년도 성균관대 과고전형] 다음과 같이 주어진 함수 $ f ( x) $는 $ x=0 $에서 미분가능함을 보여라. $$ f ( x)= { \begin {cases} 0 & ( x \leq 0) \\ \frac {1} {3 ^ {2n} } & \left ( \frac {1} {3 ^ {n+1} }

[더플러스수학] 2009학년도 한양대 모의논술 [내부링크]

[2009학년도 한양대 모의논술] 1. 다음의 제시문을 읽고 물음에 답하시오.(25점) 흔히 우리는 $ \lim\limits _ {x \rightarrow \infty } {f ( x)= \infty } $를 “$ x $값이 한없이 증가하면 함수 $ f ( x) $의 값도 한없이 증가한다”는 뜻으로 이해한다. 이를 좀 더 엄밀히 정의하면 다음과 같다. 임의로 주어진 실수 $ b $에 대해 “$ x>a $이면 $ f ( x)>b $”를 만족하는 적당한 실수 $ a $가 항상 존재하면 이를 $ \lim\limits _ {x \rightarrow \infty } {f ( x)} = \infty $로 나타낸다. 임의의 다항함수 $ f ( x)=a _ {n} x ^ {n} +a _ {n-1} x ^ {n-1} +..

[더플러스수학] 2012학년도 성균관대 수리논술 [내부링크]

2012학년도 성균관대 수리논술 다음 을 읽고 [문제 2-i]과 [문제 2-ii]에 대해 문항별로 풀이와 함께 답하시오. 음이 아닌 정수 $ n $에 대해 수열 $ \left \{\theta _ {n} \right \} $과 $ \left \{ r _ {n} \right \} $을 아래와 같이 정의하자. $ \theta _ {0} =0 $, $ \theta _ {n} =1+ \frac {1} {2} + \frac {1} {3} + \cdots + \frac {1} {n} $ ($ n \geq 1 $) $ r _ {0} =1 $, $ r _ {n} =r _ {n-1} \cos \left ( \frac {1} {n} \right ) $ ($ n \geq 1 $) 이 두 개의 수열을 이용하였을 때, $ xy-..

[더플러스수학] 2012학년도 고려대 수리논술 [내부링크]

2012학년도 고려대 (나) 직원뿔의 꼭짓점 $ O $에서 밑면의 둘레 위의 한 점 $ A $를 잇는 모선의 중점을 $ B $라 하자. 그림 1과 같이 $ B $를 지나고 밑면과 평행한 평면으로 원뿔을 잘라 윗면의 반지름이 $ 1 $, 밑면의 반지름이 $ 2 $, 선분 $ AB $의 길이가 $ x $인 직원뿔대를 얻었다. 한 점 $ P $가 점 $ A $에서 출발하여 직원뿔대의 옆면을 한 바퀴 돌아 점 $ B $에 도달할 때, 그 경로가 최단거리를 가지게 되는 경우를 생각해 보자. (단, 경로는 윗면의 경계 또는 밑면의 경계의 일부를 포함할 수 있다.) 논제 2. (필수) 위의 제시문 (나)를 읽고 다음 질문에 답하시오. (a) $ x=2 $일 때 경로의 최단거리를 구하시오. (b) 경로의 최단거리를 $..

[연세대] 1995학년도 연세대 본고사문제 [더플러스수학] [내부링크]

https://tv.kakao.com/v/402653242 [연세대 본고사] 다음 물음에 답하여라. (1) 정의역이 $ ( 0,~ \infty ) $인 함수 $ f ( x) $가 $ f ( x ^ {2} )= \frac {f ( x)} {x} $를 만족시킨다. 모든 자연수 $ n $에 대하여 $$ f ( x)= \frac {f ( x ^ {2 ^ {-n} } )} {x ^ {1-2 ^ {-n} } } $$이 성립함을 수학적 귀납법으로 증명하여라. (2) 위의 (가)을 이용하여 정의역 $ ( 0,~ \infty ) $에서 연속인 함수 $ f ( x) $로서 $ f ( 1)=1 $와 $ \int _ {1} ^ {x ^ {2} } {f ( t)dt} = \int _ {x ^ {2} } ^ {x ^ {4} } ..

[더플러스수학] 2006학년도 포스텍 면접구술고사 [내부링크]

[포스텍 2006학년도 면접구술고사]-조기졸업자 수시 [문제1] 3차방정식은 $ \sqrt {2} x ^ {3} +x ^ {2} + \sqrt {2} x+2=0 $가 하나의 실수근만을 가지면 그 실수근은 무리수임을 보여라. [문제2] 삼차함수 $ f ( x)=ax ^ {3} +bx ^ {2} +cx+d $이 조건 $ f ( 0)>0 $, $ f ( 1)2 $, $ f ( 3)

[더플러스수학] 1980학년도 고려대 본고사문제 [내부링크]

['80 고려대] 단위원 $ x ^ {2} +y ^ {2} =1 $과 $ x $축과의 교점을 $ \rm A ( 1,~0) $이라 하고, 점 $ \rm P,~Q $를 반시계방향으로 측정한 각의 크기가 $ \rm \angle AOP= \alpha $, $ \rm \angle POQ= \frac {\pi } {2} $ 되게 잡을 때, (1) $ \rm \overrightarrow {OP} ,~ \overrightarrow {OQ} $의 성분을 $ \alpha $를 써서 나타내어라. (2) 단위원 위의 임의의 점을 $ \rm R $이라 하고, $ \rm \angle POR= \beta $라 할 때, $ \rm \overrightarrow {OR} $을 $ \rm \overrightarrow {OP} $, $ \..

[더플러스수학] 2010학년도 홍익대 심층면접 기출 [내부링크]

[2010학년도 홍익대 수시1 심층면접] 집합 $ X $의 부분집합 $ S _ {1} ,~S _ {2} ,~S _ {3} , ~\cdots $에 대해 "$ \lim\limits _ {} {} S _ {n} $" 을 다음과 같이 정의하자. "$ \lim\limits _ {} {} S _ {n} $" = $ \left\{ x \in X~| ~\right . $ 적당한 $ N $이 존재하여 $ n>N $ 인 모든 $ n $에 대해 $ \left . x \in S _ {n} \right\} $ 다음의 각 경우 "$ \lim\limits _ {} {} S _ {n} $"을 구하여라. 아래에서 $ X= R $ 이다. 여기서 $ R $은 실수의 집합이다. ① $ S _ {n} = \left\{ x ~ |~ \frac..

[더플러스수학] 2011학년도 서울시립대 수리논술 [내부링크]

https://tv.kakao.com/channel/3372901/cliplink/401542336 [2011학년도 시립대학교 논술] 수열 $ \left\{ f _ {n} \right\} $이 $ f _ {1} =1,~f _ {2} =1,~f _ {n+2} =f _ {n+1} +f _ {n} $을 만족할 때, 이 수열을 피보나치 수열이라 하고, 수열 $ \left\{ l _ {n} \right\} $이 $ l _ {1} =1,~l _ {2} =3,~l _ {n+2} =l _ {n+1} +l _ {n} $을 만족할 때, 이 수열을 루카스 수열이라고 한다. 다음 물음에 답하시오. (a) $ a _ {n} = \frac {1} {\sqrt {5} } \left [ \left ( \frac {1+ \sqrt {..

[더플러스수학] 2006학년도 고려대 수리논술 [내부링크]

https://tv.kakao.com/channel/3372901/cliplink/401541863 [2006학년도 고려대 수시2] 그림과 같이 정사각형 모양을 이루며 나열되어 있는 $ ( n+2) ^ {2} $개의 점들 중에서 네 점을 꼭짓점으로 하는 임의의 정사각형을 생각하자. 그림에서 정사각형 $ P _ {1} $과 같이 각 변이 수평 또는 수직인 것을 “똑바른 정사각형”이라 하고, 정사각형 $ P _ {2} $와 같이 그렇지 못한 것을 “비스듬한 정사각형”이라 하자.(단, $ n \geq 1 $) [논제1] 한 변위에 $ ( k+2) $개의 점이 놓여 있는 똑바른 정사각형의 개수를 구하시오.(단, $ 1 \leq k \leq n $) [논제2] 한 변위에 $ ( k+2) $개의 점이 놓여 있는 똑..

[더플러스수학] 2008학년도 서울대 특기자전형 심층면접(일반전형) [내부링크]

[2008 서울대 특기자 구술] $ x _ {1} =a,~x _ {n+1} = \frac {1} {2} x _ {n} +x _ {n} ^ {b} $ ($ a,b>0 $)인 수열 $ \left\{ x _ {n} \right\} $이 있다. (1) $ a $에 따라 수열 $ \left\{ x _ {n} \right\} $의 극한값이 2개 이상 존재하는 $ b $의 값의 범위를 구하여라. (2) $ 0

[더플러스수학] 2008학년도 서울대 특기자 심층면접 (일반전형) [내부링크]

[2008 서울대 특기자] (1). 양수 $ a,~b,~c $에 대하여 $ \lim\limits _ {n \rightarrow \infty } {\left ( a ^ {n} +b ^ {n} +c ^ {n} \right ) ^ { \frac {1} {n} } =} A $ 인 극한값 $ A $를 구하라. (2). $$ \lim\limits _ {n \rightarrow \infty } { \frac {\left ( 1+ \left ( \frac {1} {2} \right ) ^ {n} \right ) ^ { \frac {1} {n} } -1} {\left ( \frac {1} {2} \right ) ^ {n} } =0} $$임을 보이고 $ \lim\limits _ {n \rightarrow \infty } ..

[더플러스수학] 2008학년도 성균관대 수시 과고전형 기출 [내부링크]

[2008학년도 성균관대 수시2 문제] $ p $는 1보다 큰 양의 실수이다. 임의의 자연수 $ n \geq 2 $에 대하여 $ I _ {p} ( n) $을 다음과 같이 정의한다. $$ I _ {p} ( n)= \frac {1} {2 ^ {p} } + \frac {1} {3 ^ {p} } + \cdots + \frac {1} {n ^ {p} } $$ (a) 다음 부등식이 성립함을 보여라. $$ I _ {p} ( n) \leq \frac {1} {p-1} $$ (b) 다음 부등식이 성립함을 보여라. $$ \sum\limits _ {k=2} ^ {\infty } I _ {k} ( n)I _ {k+1} ( n) \leq 1 $$ 힌트 a) $ f ( x)= \frac {1} {x ^ {p} } $의 그래프를 ..

[더플러수학] 2011학년도 서강대 수리논술 기출문제 [내부링크]

다음 글을 읽고, 물음에 답하라. 실수와 유리수를 구분 짓는 핵심은 적합한 극한 이론 구성의 가능성과 관련된 공리이다. 유리수는 수학을 위해 필요한 산술적 성질과 그 밖의 많은 중요한 성질들을 갖고 있지만, 유리수만으로는 수렴하는 수열의 극한값을 표현할 수 없기 때문에 체계적인 극한 이론을 위해서는 적당하지 않다. 예를 들면 $ \sqrt {2} $를 소수 $ n $째 자리까지 나타낸 항들로 이루어진 유리수의 수열 $ 1.4,~1.41,~1.414,~1.4142,~ \cdots $는 명백히$ \sqrt {2} $로 수렴하는데, 우리가 이미 알고 있듯이 해당 수열의 극한값 $ \sqrt {2} $는 유리수가 아니다. 이와 같은 이유로, 극한 이론에서 출발한 미적분학이 수학적 체계를 갖추기 위해서는 완비성을..

[더플러스수학] 2014학년도 부산대 수리논술 [내부링크]

[2014학년도 부산대 수리논술] ※ 다음 제시문을 읽고 물음에 답하시오. 피보나치 수열(Fibonacci sequence) $ \left\{ a _ {n} \right\} _ {n=1} ^ {\infty } $은 귀납적으로 $ a _ {1} =1,~a _ {2} =1, ~ a _ {n+2} =a _ {n+1} +a _ {n} $ (모든 $ n \geq 1 $) 으로 정의된 수열이다. 이 수열의 항을 순서대로 적어보면 $1,~ 1, ~2, ~3, ~5,~ 8, ~13, ~21, ~34, ~55, ~89$, $ bold {\cdots } $ 이다. 이 때 극한 $ \lim\limits _ {n \rightarrow \infty } { \frac {a _ {n+1} } {a _ {n} } } $이 존재하고,..

[더플러스수학] 2011학년도 UNIST 예시문제 [내부링크]

다음 점화식을 만족하는 수열 $ \left\{ x _ {n} \right\} $, $ \left\{ y _ {n} \right\} $의 극한값은 무엇인가? $ x _ {0} =2 $, $ y _ {0} =1 $, $ x _ {n+1} = \frac {x _ {n} +y _ {n} } {2} $, $ y _ {n+1} = \frac {2x _ {n} y _ {n} } {x _ {n} +y _ {n} } $, $ n=0,1,2, \cdots $ 일반적으로, 무한수열 $ \left\{ a _ {n} \right\} $에서 $ n $이 한없이 커질 때, $ a _ {n} $이 어떤 일정한 실수값 $ c $에 한없이 가까워지면 수열 $ \left\{ a _ {n} \right\} $은 $ c $에 수렴한다고 하고..

[더플러스수학] 2008학년도 고려대 수리논술 기출 [내부링크]

(2008년 고려대 수시 2-2) 두 개의 저항을 그림 $ 1 $과 같이 연결하는 방법을 직렬연결이라 하고, 그림 $ 2 $와 같이 연결하는 방법을 병렬연결이라 한다. 크기가 $ R _ {1} $과 $ R _ {2} $인 두 개의 저항을 직렬 연결할 때, 합성 저항 $ \ R $은 각 저항의 합과 같다$ ( R=R _ {1} +R _ {2} ) $. 그리고 크기가 $ R _ {1} $과 $ R _ {2} $인 두 개의 저항을 병렬 연결할 때, 합성 저항의 역수는 각 저항의 역수의 합과 같다$ \rm \left ( \frac {1} {R} = \frac {1} {R _ {1} } + \frac {1} {R _ {2} } \right ) $. 그림 3과 같이 $ 1 \omega $의 저항을 각 단계마다 계속..

[더플러스수학] 유클리드 기하학 공리에 대해 [내부링크]

http://m.blog.naver.com/plusthemath/221616058455 읽어볼 내용이 있어서 공유합니다 에우클리데스의 기하학 (유클리드 기하학)에는 다섯 개의 공리가 있다. 그 중, 가장 잘 알려진 공리가 있다면, 그것은 다음과 같은 '평행선 공준'이다. '선 밖의 한 점을 지나 그 직선에 평행한 직선은 단 하나만 존재한다.' 그런데, 에우클리드데스가 지배하던 기하학의 수천 년의 아성을 깨고, 이 평행선 공준에 정면으로 반기를 든 기하학이 출현하였으니, 당연히 그 이름은 '비유클리드 기하학' 이었다. 비유클리드 기하학이 정면으로 반박하여 새로 만든 공준은 이렇다. '선 밖의 한 점을 지나 그 직선에 평행한 직선은 둘 이상 존재한다.' 이 공준이 성립되는 기하학 중에, 상당히..

[더플러스수학] 2016학년도 평가원 6월 30번 [내부링크]

정의역이 $ \left \{ x ~|~0 \leq x \leq 8~ \right \} $이고 다음 조건을 만족시키는 모든 연속함수 $ f ( x) $에 대하여 $ \int _ {0} ^ {8} f ( x)dx $의 최댓값은 $ p+$$ \frac {q} {\ln 2} $이다. $ p+q $의 값을 구하시오. (단, $ p,~q $는 자연수이고, $ \ln 2 $는 무리수이다.) [4점] [2015년 6월 30번] (가) $ f ( 0)=1 $이고 $ f ( 8) \leq 100 $이다. (나) $ 0 \leq k \leq 7 $인 각각의 정수 $ k $에 대하여 $ f ( k+t)=f ( k)~ ( 0

[더플러스수학] [습관] 멋있게 말하는 다섯가지 방법 (인용) [내부링크]

http://moneyman.kr/archives/3513 [습관] 멋있게 말하는 5가지 방법 말은 자기 캐릭터를 가장 잘 드러내는 요소 중 하나다. moneyman.kr

[더플러스수학] 2015년 교육청 4월 21번 [내부링크]

함수 $ f \left ( x \right ) = { \begin {cases} \left ( x-2 \right ) ^ {2} e ^ {x} +k _ { {} _ { {} _ {} } } & ~ \left ( x \geq 0 \right ) _ { {} _ { {} _ {} } } \\ -x ^ {2 ^ { {} ^ { {} ^ {} } } } & ~ \left ( x

[더플러스수학] 2015년 교육청 4월 30번 [내부링크]

그림과 같이 원점 $ \rm O $를 중심으로 하고 반지름의 길이가 $ 10 $인 원이 있다. 직선 $ y= \sqrt {3} x $와 원이 제$ 1 $사분면에서 만나는 점을 $ \rm A $라 하자. 점 $ \rm P $는 원점 $ \rm O $를 출발하여 $ x $축을 따라 양의 방향으로 매초 $ 2 $의 일정한 속력으로 움직인다. 점 $ \rm P $가 원점 $ \rm O $를 출발하여 $ t $초가 되는 순간, 점 $ \rm P $를 지나고 직선 $ y= \sqrt {3} x $에 평행한 직선이 제$ 1 $사분면에서 원과 만나는 점을 $ \rm Q $라 하자. 세 선분 $ \rm AO $, $ \rm OP $, $ \rm PQ $와 호 $ \rm QA $로 둘러싸인 부분의 넓이를 $ S $라 할 ..

[더플러스수학] 2015년 교육청 모의고사 3월 21번 [내부링크]

실수 전체의 집합에서 정의된 두 함수 $ f ( x)=\sin ^ {2} x+a\cos x $, $ g ( x)= { \begin {cases} 0 & \left ( x

[더플러스수학] 2015학년도 평가원 9월 30번 (2014년 시행) [내부링크]

양의 실수 전체의 집합에서 감소하고 연속인 함수 $ f ( x) $가 다음 조건을 만족시킨다. (가) 모든 양의 실수 $ x $에 대하여 $ f ( x)>0 $이다 (나) 임의의 양의 실수 $ t $에 대하여 세점 $ ( 0,~0), ~( t,~f ( t)), ( t+1,~f ( t+1)) $을 꼭짓점으로 하는 삼각형의 넓이가 $ \frac {t+1} {t} $ 이다 (다) $ \int _ {1} ^ {2} { \frac {f ( x)} {x} dx=2} $ $ \int _ { \frac {7} {2} } ^ { \frac {11} {2} } { \frac {f ( x)} {x} dx= \frac {q} {p} } $라 할 때, $ p+q $의 값을 구하시오 (단, $ p $와 $ q $는 서로소인 자연..

[더플러스수학] 2014년 교육청 7월 30번 [내부링크]

한 변의 길이가 $ 4 $인 정육면체 $ \rm ABCD-EFGH $와 밑면의 반지름의 길이가 $ \sqrt {2} $이고 높이가 $ 2 $인 원기둥이 있다. 그림과 같이 이 원기둥의 밑면이 평면 $ \rm ABCD $에 포함되고 사각형 $ \rm ABCD $의 두 대각선의 교점과 원기둥의 밑면의 중심이 일치하도록 하였다. 평면 $ \rm ABCD $에 포함되어 있는 원기둥의 밑면을 $ \alpha $, 다른 밑면을 $ \beta $라 하자. 평면 $ \rm AEGC $가 밑면 $ \alpha $와 만나서 생기는 선분을 $ \rm \overline {MN} $, 평면 $ \rm BFHD $가 밑면 $ \beta $와 만나서 생기는 선분을 $ \rm \overline {PQ} $라 할 때, 삼각형 $ \r..

[더플러스수학] 2014년 교육청 4월 21번 [내부링크]

그림과 같이 좌표평면 위에 네 점 $ \rm A \left ( 1,~0 \right ) $, $ \rm B \left ( 3,~0 \right ) $, $ \rm C \left ( 3,~2 \right ) $, $ \rm D \left ( 1,~2 \right ) $를 꼭짓점으로 하는 정사각형 $ \rm ABCD $가 있다. 한 변의 길이가 $ 2 $인 정사각형 $ \rm EFGH $의 두 대각선의 교점이 원 $ x ^ {2} +y ^ {2} =1 $ 위에 있을 때, 두 정사각형의 내부의 공통부분의 넓이의 최댓값은? (단, 정사각형의 모든 변은 $ x $축 또는 $ y $축에 수직이다.) [4점][2014년 4월 21번] ① $ \frac {2+ \sqrt {3} } {4} $ ② $ \frac {1+ ..

[더플러스수학] 2014년 교육청 4월 30번 [내부링크]

함수 $ f \left ( x \right ) = \frac {\ln x ^ {2} } {x} $의 극댓값을 $ \alpha $라 하자. 함수 $ f \left ( x \right ) $와 자연수 $ n $에 대하여 $ x $에 대한 방정식 $ f \left ( x \right ) - \frac {\alpha } {n} x=0 $의 서로 다른 실근의 개수를 $ a _ {n} $이라 할 때, $ \sum\limits _ {n=1} ^ {10} a _ {n} $의 값을 구하시오. [4점] [2014년 4월 30번] 정답 및 풀이를 보려면 아래를 클릭하세요. ...더보기 정답 34 [출제의도] 함수의 그래프의 개형으로 문제해결하기 $ f ( x)= \frac {\ln x ^ {2} } {x} $에 대하여 ⅰ)..

[더플러스수학] 2014년 교육청 3월 30번 [내부링크]

실수 $ t $에 대하여 좌표평면에서 원점을 지나고 기울기가 $ \tan ( \sin t) $인 직선과 원 $ x ^ {2} +y ^ {2} =e ^ {2t} $이 만나는 점 중에서 $ x $좌표가 양수인 점을 $ \rm P $라 하고, 점 $ \rm P $가 나타내는 곡선을 $ C $라 하자. $ t= \pi $일 때, 곡선 $ C $ 위의 점 $ \rm P $에서의 접선과 $ x $축 및 $ y $축으로 둘러싸인 부분의 넓이는 $ a \times e ^ {b \pi } $이다. $ 10 ( a+b) $의 값을 구하시오. (단, $ a $와 $ b $는 유리수이다.) [4점][2014년 3월 30번] 정답 25 [출제의도] 삼각함수의 성질과 매개변수의 미분법을 이용하여 문제를 해결한다. 원점을 지나고 ..

[더플러스수학] 2016학년도 부산대 의대논술 문제 (의학계열) [내부링크]

【문항 1】 [제시문] (삼수선의 정리) 평면 $\displaystyle \alpha $위에 있지 않은 한 점 $\displaystyle \mathrm P $와 평면 $\displaystyle \alpha $위 에 있는 직선 $\displaystyle l $, 직선 $\displaystyle l $위에 있는 한 점 $\displaystyle \mathrm H $, 평면 $\displaystyle \alpha $ 위에 있으면서 직선 $\displaystyle l $ 위에 있지 않은 점 $\displaystyle \mathrm O $에 대하여 아래의 사실이 성립한다. (1) $\displaystyle \overline {\mathrm{PO}} \bot \alpha $, $\displaystyle \overl..

[더플러스수학] 2003학년도 포스텍 심층면접고사 [내부링크]

모든 실수에 대하여 정의된 미분 가능한 함수 $ f $가 주어져 있다. (a) $ f ' ( 1)=1 $이고 (b) 모든 실수 $ x,~y~ $에 대하여 $ f ( xy)=f ( x)f ( y) $이 성립한다. 함수 $ f ( x) $를 $ x $의 식으로 표현하여라. https://tv.kakao.com/channel/3372901/cliplink/401207485

[더플러스수학] 2015학년도 평가원 9월 20번 (2014년 시행) [내부링크]

$3$ 이상의 자연수 $ n $에 대하여 함수 $ f ( x) $가 $ f ( x)=x ^ {n} e ^ {-x} $일 때, 에서 옳은 것만을 있는 대로 고른 것은? [$4$점] [2014년 9월] ㄱ. $ f \left ( \frac {n} {2} \right ) =f ' \left ( \frac {n} {2} \right ) $ ㄴ. 함수 $ f ( x) $는 $ x=n $에서 극댓값을 갖는다. ㄷ. 점 $ ( 0,0) $은 곡선 $ y=f ( x) $의 변곡점이다. ① ㄴ ② ㄷ ③ ㄱ, ㄴ ④ ㄱ, ㄷ ⑤ ㄱ, ㄴ, ㄷ https://tv.kakao.com/channel/3372901/cliplink/401141106 정답 및 풀이를 보려면 아래를 클릭하세요. ...더보기 정답 3번

latex 으로 티스토리에 수식입력하기 [내부링크]

https://pkjung.tistory.com/152 MathJax 그간 웹에서 수식을 어떻게 써야 하는지를 물어보시는 분이 많아서 수식을 쓰는 방법 중 꽤 많이 보이는 방법인 MathJax에 대해 정리해 보았습니다. 제가 전문 프로그래머가 아니라 오류가 있을 수 있고, 어떤 내.. pkjung.tistory.com

[더플러스수학] 2019학년도 부산대학교 수시모집 논술전형 논 술 고 사(의학계) [내부링크]

【문항 1】다음 제시문을 이용하여 아래 논제의 풀이 과정과 답을 논리적으로 서술하시오. (가) 영벡터가 아닌 두 공간벡터 $ \overrightarrow {a} $, $ \overrightarrow {b} $가 이루는 각의 크기를 $ \theta ( 0 \leq \theta \leq \pi ) $라고 할 때, $$ \overrightarrow {a} \cdot \overrightarrow {b} = \left | \overrightarrow {a} \right | \left | \overrightarrow {b} \right | \cos \theta $$ (나) 점 $ \rm A ( \it x _ {1} ,y _ {1} ,z _ {1} ) $을 지나고 벡터 $ \overrightarrow {u} = ( ..

2013학년도 (가) 평가원 9월 21번 [더플러스수학] [내부링크]

최고차항의 계수가 $ 1 $인 삼차함수 $ f ( x) $의 역함수를 $ g ( x) $라 할 때, $ g ( x) $가 다음 조건을 만족시킨다. (가) $ g ( x) $는 실수 전체의 집합에서 미분가능하고 $ g ' ( x) \leq \frac {1} {3} $이다. (나) $ \lim\limits _ {x \rightarrow 3} \frac {f ( x)-g ( x)} { ( x-3)g ( x)} = \frac {8} {9} $ $ f ( 1) $의 값은? [4점][2012년 9월] ① $ -11 $ ② $ -9 $ ③ $ -7 $ ④ $ -5 $ ⑤ $ -3 $ https://tv.kakao.com/channel/3372901/cliplink/401140852 정답 및 풀이를 보려면 아래를 클릭..

[더플러스수학] 2018학년도 연세대 특기자전형 과고전형 [내부링크]

https://tv.kakao.com/channel/3372901/cliplink/401290276 [더플러스수학] 2018학년도 연세대 특기자전형 과고전형 [문제1] $ 01 $) I. $ f ( 0)=0 $, $ f ( a)=0 $이다. II. 닫힌 구간 $ [0,~a] $에서 $ \left | f ( x)| \right . $의 최댓값이 $ \frac {1} {2018} $이다. II. 닫힌 구간 $ [0,~a] $에서 $ \left | f ' ( x)| \right . $의 최댓값이 $ 2018 $이다. [문제3] 제시문 2의 조건을 만족하는 함수 $ f ( x) $를 하나 찾으시오.

[더플러스수학] 2018학년도 인하대 수리논술 오후1 [내부링크]

[문제 1] (30점) 다음 제시문을 읽고 물음에 답하시오. (가) 두 함수 $ f ( x),~g ( x) $가 구간 $ [a,~b] $에서 연속일 때, 두 곡선 $ y=f ( x),~y=g ( x) $와 두 직선 $ x=a,~x=b $로 둘러싸인 도형의 넓이는 다음과 같다. $$ S= \int _ {a} ^ {b} {\left | f ( x)-g ( x) \right |} dx $$ 이다. (나) 두 함수 $ f ( x) $가 닫힌 구간 $ [a,~b] $에서 연속이면 다음이 성립한다. $$ \lim\limits _ {n \rightarrow \infty } {\sum\limits _ {k=1} ^ {n} f \left ( x _ {k} \right ) \Delta x} = \int _ {a} ^ {b..

[더플러스수학] [서울대 2003학년도 수시 심층면접] [내부링크]

공간의 $ xy $평면 위에 원 $ S= \left\{ \left ( x,y,z \right ) |x ^ {2} +y ^ {2} =1,~z=0 \right\} $가 주어져 있다. (1) 공간의 한 점 $ P ( a,b,c) $에서 $ S $까지의 최단거리를 구하는 방법을 설명하여라. (2) $ xz $평면 위의 원 $ T= \left\{ \left ( x,y,z \right ) |~ x ^ {2} + \left ( z-1 \right ) ^ {2} =1,~y=0 \right\} $ 에서 $ S $ 까지의 최단거리를 구하는 방법을 설명하고, 그 최단거리를 말하여라. (3) $ xz $ 평면 위의 타원 $ E= \left\{ \left ( x,y,z \right ) | \frac {x ^ {2} } {4} ..

2003학년도 서울대 의대 수시 심층면접 [내부링크]

2003학년도 서울대 의대 수시 심층면접 다음 물음에 답하여라. (1) 미분가능한 함수 $ y=f ( x) $의 역함수를 $ g $라 하자. 좌표평면에서 점 $ \rm P \it ( a,~b) $는 함수 $ y=f ( x) $를 만족한다. 이 때, $ g ' ( b)= \frac {1} {f ' ( a)} $임을 설명하여라. (2) 함수 $ f ( x)=\sin ^ {2} \left ( \frac {\pi } {2} x \right ) $ 단, $ 0

[더플러스수학] 2019년 교육청 7월 20번 [내부링크]

실수 전체의 집합에서 미분가능한 함수 $ f \left ( x \right ) $가 모든 실수 $ x $에 대하여 $$ f \left ( 1+x \right ) =f \left ( 1-x \right ) $$ $$ f \left ( 2+x \right ) =f \left ( 2-x \right ) $$ 를 만족시킨다. 실수 전체의 집합에서 $ f ' \left ( x \right ) $가 연속이고, $ \int _ {2} ^ {5} {f ' \left ( x \right ) dx=4} $일 때, 에서 옳은 것만을 있는 대로 고른 것은? [4점] ㄱ. 모든 실수 $ x $에 대하여 $ f \left ( x+2 \right ) =f \left ( x \right ) $이다. ㄴ. $ f \left ( 1 \..

2019학년도 경희대 수리논술(토) [더플러스수학] [내부링크]

I. 다음 제시문을 읽고 논제에 답하시오. (60점) [가] 두 초점 $ \rm F ( \it c,~0) $, $ \rm F ' ( - \it c,~0) $으로부터의 거리의 합이 $ 2a $ ($ a>c>0 $)인 타원의 방정식은 $ \frac {x ^ {2} } {a ^ {2} } + \frac {y ^ {2} } {b ^ {2} } =1 $이다. (단, $ b ^ {2} =a ^ {2} -c ^ {2} $) [나] 두 변수 $ x,~y $의 함수 관계가 변수 $ t $를 매개로 하여 $$ x=f ( t),~y=g ( t) $$ 와 같이 나타날 때 변수 $ t $를 매개변수라 하고, 위 함수를 매개변수로 나타낸 함수라고 한다. [다] 점 $ \left ( x _ {1} ,~y _ {1} \right )..

[더플러스수학] 2015학년도 가형 수능 30번 [내부링크]

https://tv.kakao.com/channel/3372901/cliplink/401187304 2015학년도 가형 수능 30번 함수 $ f ( x)=e ^ {x+1} -1 $과 자연수 $ n $에 대하여 함수 $ g ( x) $를 $ g ( x)=100 \left | f ( x) \right | - \sum\limits _ {k=1} ^ {n} \left | f ( x ^ {k} ) \right | $ 이라 하자. $ g ( x) $가 실수 전체의 집합에서 미분가능하도록 하는 모든 자연수 $ n $의 값의 합을 구하시오. [4점][2015학년도 수능] 정답 및 풀이를 보려면 아래를 클릭하세요. ...더보기 정답 39 $ \left | f ( x) \right | = { \begin {cases} -..

수학문제입력연습 [내부링크]

두 함수 $ f \left ( x \right ) =2x+a $, $ g \left ( x \right ) =bx+5 $에 대하여 $ f ^ {-1} \left ( 2 \right ) =-1 $, $ \left ( f ^ {-1} \circ g \right ) ^ {-1} \left ( 2 \right ) =3 $ 를 만족시키는 상수 $ a,~b $에 대하여 $ a+b $의 값은?[$ 3.7 $점] ① $ 1 $ ② $ 5 $ ③ $ 9 $ ④ $ 12 $ ⑤ $ 17 $ https://youtu.be/wXc5rTNvDFQ

글쓰기 연습 [내부링크]

함수 $ y= \frac {2} {x} $의 그래프에 대한 설명으로 옳지 않은 것은? [$ 3.3 $점] ① 점근선은 $ x $축과 $ y $축이다. ② 직선 $ y=-x $에 대하여 대칭이다. ③ 함수 $ y=- \frac {2} {x} $의 그래프와 $ y $축에 대하여 대칭이다. ④ 함수 $ y= \frac {3} {x} $의 그래프보다 원점에서 멀리 위치한다. ⑤ 정의역과 치역은 $ 0 $을 제외한 실수전체의 집합이다. https://youtu.be/OPF74jWWLZg